You are on page 1of 320

CHEMISTRY

. . . .
.

1437 2016 /

9

19 :
20 1-1
21 2-1
21 3-1
22 4-1
24 5-1 ( )
25 6-1
26 7-1
27 8-1
27 9-1
29 10-1
31 11-1
32 12-1
37 13-1
42 14-1
44 15-1
49 16-1
54 17-1
56
56
59

60 :
66 1-2
68 2-2
68 3-2
69 4-2

3

70 5-2
75 6-2 Keq
79 7-2 Kc Kp
80 8-2
84 9-2
85 10-2
87 11-2
91 12-2
93
93
95

103 :
104 1-3
104 2-3
108 3-3
117 4-3
120 5-3
120 6-3
129 7-3
131 8-3 ( )
139 9-3
146
147
149

:
153
154 1-4
154 2-4
156 3-4
159 4-4

4

160 5-4
161 6-4
164 7-4
178 8-4
180 9-4
183 10-4 ()
184
185
186

191 :
192 1-5
193 2-5
195 3-5
203 4-5
204 5-5
206 6-5
209 7-5
215 8-5
217
217
218

221 :

222 1-6

222 2-6 ()

5

225 3-6
227 4-6
230 5-6
236 6-6
250
250
252

257 :
258 1-7
260 2-7
265 3-7
271 4-7
275 5-7
281 6-7
285 7-7
287 8-7
291
293

297 :
298 1-8
298 2-8
302 3-8
305 4-8
306 5-8 ()
306 6-8
308
310
6


- -

.
-:
:
. . .
:
.
: . ..

.
:
.
.
: ...
...

..
:
.
:
..

7

.
.
.
:

.


.
.
.
.
.
.

.
.
.

....

8
Preface

:
.

.
.

.

.

9



.

.


: ()SI

(. )SI
1


()kg ()kilogram () mass

()m ()meter ()length

()s ()second ( )time

()K ()Kelven ()temperature

()mol ()mole ()quantity

()A ()Ampere ()electric current

()gram) (g
( )centimeter) (cm .

:
-1 (Molar mass (M
( )M

( .)g/mole
.
:
10
1
Na2CO3
.Ca3)PO4(2 :
1 Na = 23 ; C = 12 ; O = 16 ; Ca = 40 ; P = 31

KMnO4 : NaOH :
K2Cr2O7 H2SO4 M )Na2CO3( = 2 23 + 1 12 + 3 16 = 106 g/mol
.MgCl2
M [Ca3)PO4(2] = 3 40 + 2 1 31 + 2 4 16
. = 310 g/mol

(Number of moles )n -2
( )n
.

(( )m)
( ) =
( ( ) M \ )

:
(m )g
= ( n )mol
(M )g/mol

2
K2CO3
22 g .

:
K2CO3
M )K2CO3( = 2 39 + 1 12 + 3 16 = 138 g/mol

( )n n
(m )g (22 )g
= ( n )mol = = 0.16 mol
(M )g/mol (138 )g/mol
11

:
(m)g( = n )mol( M )g/mol 2

3 Na2SO4
0.15 mole .NaCl 14.21g .
: 0.11mol :
NaCl
M )NaCl( = 1 23 + 1 35.5 = 58.5 g/mol 3
:

m
-
(m )g( = n )mol( M )g/mol
H3PO4
m )g( = 0.15 )mol( 58.5 )g/mol( = 8.78 g
721g .
- -3 (Molarity *)M
0.751mole
. ()M . Na3PO4
. :) 0.731mol
) 1231g
()
( /) =
()
!
* ( )M :
()M
(n)mol
= (M )mol /L
(V)L
( ) n :
(m)g
= (n )mol
(M)g/mol

L
mL :
(m )g
(M )g/mol
= (M )mol /L
(V )mL
(1000 )mL/L
12
:
(m)g(1000 )mL/L
= (M )mol /L
(M)g/mol( V )mL

4
53 g Na2CO3

! . 5 L .
:
Na2CO3
( )L ( )mL M )Na2CO3( = 2 23 + 1 12 + 3 16 = 106 g/mol
( )cm3
: ()n
1 L = 1000 mL
(m)g (53)g
1 L = 1000 cm3 = ( n )mol = = 0.5 mol
(M)g/mol (106)g/mol
1 mL = 1 cm3
:

4 (n)mol (0.5 )mol


= (M )mol /L = = 0.1 mol/L
151g (V )L (5 )L
CH3COOH mole/ L M
1501mL :
M )mol/L( = 0.1 mol/L = 0.1 M
. 5
. Ca)OH(2
0.51M : 0.1 M . 250 mL
:
Ca)OH(2
M [Ca)OH(2] = 1 40 + 2 16 + 2 1
= 74 g/mol
M :

(m)g(1000 )mL/L
= (M )mol /L
(M)g/mol( V )mL

:m
13
(M )mol /L( M)g/mol( V )mL
= (m)g 5
(1000 )mL/L
41g
NaOH
(0.1 )mol /L( 74)g/mol( 250 )mL
= (m)g M= 40 g/mol
(1000 )mL/L

m)g( = 1.85 g .0.051mole/L

-4 .
2 L :
NA ( )
:
NA = 6.023 1023 atoms or molecules / mol
( )atoms ( ) molecules .
( n )mole:
= ( )mol () atoms or molecules / mol

(No. )atoms or molecules( = n )mol( NA ) atoms or molecules / mol

6
0.1 mole H2
:
:
(No. molecules = n )mol( NA )molecules/ mol
(= 0.1 )mol( 6.023 1023 )molecules/ mol 6

= 6.023 1022 molecules
0.16 mole
2 .O2
: 0.96 1023 molecules :
No. atoms = No. molecules 2 1.92 1023 atoms
No. atoms = 6.023 1022 2 = 12.046 1022 atoms

14
-5

( ) . ( )10 .
log10 y . log y :
log y = x y = 10 x

log10 7 : log 7 log10 0.05


log 0.05
10 log 10n = n ::

log 107 = 7 .... log 104 = 4


.
log 0.01 = log 10-2 = -2 log 0.00001 = log 10-5 = -5
.....
.

lne y e 2.718 :
ln y = x y = e x

:
-1 :

log Z Y = log Z + log Y

log 5 6 = log 5 + log 6

-2
Z
= log Z - log Y log
Y
8
= log 8 - log 3 log
3
-3
log ZY = Y log Z .

log 29 = 9 log 2


( )Calculator
.
15

:
-1 ()log
log .

7
:
() ( log 7) ( log 13) ( log 0.08) log 1.5
7
:
() 7 log 0.84509804 )log 26.7
log 7 = 0.84509804 ) log 0.89

() 13 log 1.11394335 ) ln 93
) ln 0.02
() 0.08 log - 1.096910013
.
() 1.5 log 0.176091259

( )2 ()ln
ln

8 8
: y
() ln 1.5 () ln 0.08 () ln 13 () ln 7
.
: ) log y = 2.6
() 7 ln 1.94510149 ) log y = -1.2
() 13 ln 2.564949357 ) ln y = 0.7
) ln y = -3.9
() 0.08 ln
2.525728644 -
() 1.5 ln 0.405465108

16
:
-1
() 2ndF
( )... log .

9
:
() 0.84509804
() 1.113943352
() 1.096910013 -
() 0.176091259
:
() 0.84509804 2nd F log
7
() 1.113943352 2ndF
log 13 12.99999999
() -1.096910013 2nd F log
0.08
() 0.176091259 2nd F log
1.5

-2
(.)1

10
:
() 3.22
() -1.78

:
() 3.22 2nd F ln 25
() -1.78 2ndF ln
0.17
17

)m (g
= ) n (mol 11 ()n
)M (g/mol
)n(mol
= )M (mol /L 12 ()M
)V(L

14

)No. (atoms or molecules) = n (mol) NA ( atoms or molecules / mol

(Molar mass )M
1
.

(Molarity )M
.

)Avogadros Numder (NA


. 6.023 1023 mol-1

18

Thermodynamics
1

:
.
.

.
.
.


.

.

.

.
19
1-1


...
.

.


.
:
-1 .
-2
.
-3
.
-4
.


( )

.

( )Potential Energy (.)Kinetic Energy

.

.

.

.

20
2-1

( )SI ( )Joule ( )J :

1 J .
1 J = 1 kg . m2/s2

kg ( )kilogram m ()meter
s (.)second
( )KE
! }) {m (kg }). {v (m/s

1
( )m ()m m . v2 = KE
2

2 kg 1 m
1 s ( )KE .1 J
1
(2 kg) . (1 m/s)2 = 1 kg . m2/s2 = 1 J = KE
2


( )K C
:
T (K) = t (C)+ 273

3-1
()System
( )

( )Boundaries
( .)Surronding
( )Universe :

.
= +
21

:

-1 Open System

.

() .

-2 Closed System

.


( ) .

-3 Isolated System



.



.

Heat 4-1



( )q
.
.
() .
:
22
T=Tf -Ti

Tf ( f final ) Ti
( i initial )
( )q :
1-1
q T ()1
()25C
T :C
( ) J/g .C

q = C. T ()2
4.18 ()
2.44
2.03 () ( )Heat capacity
2.01 ()
m }( {m )g
1.83
1.023
( .)J/C
0.90 ( )Specific heat( ) :
0.65
0.45 C=m ()3
0.30
0.24
0.20
0.13 ( )1 g
0.13 (.) J/g .C C ( )3
( )2 :

! (q )J( = )J/g .C( m)g( T)C

( )
( )1-1 .
()Zeta

1-1
870 g
5C 95C .0.45 J/g .C

23
:

T = )Tf -Ti(C = )95-5(C = 90C

: q
(q )J( = )J/g .C( m)g( T )C

q )J( = 0.45 ) J/g .C( 870 )g( 90 )C( = 35235 J


1-1

kJ:
10 g 25C
) ( ) ( (1 )kJ 45C
q kJ = q J .205 J
(1000 )J .
(1 )kJ 1.03 J/g .C :
= 35.2 kJ (q )kJ( = 35235 )J
(1000 )J

5-1 ) ( Enthalpy

.
( )
.

( )Enthalpy of reaction ( )H
H
:
H = qP

qP ( P Pressure
).

.
( Hr r reaction ) :
24
Hr ( )Hr < 0 : Hr 
( )H = -
r
(.)Exothermic reaction
Hr ( )Hr > 0 Hr 
( )Hr = +
(.)Endothermic reaction
:

(Hr= H )Products( - H )Reactants



( )Products ( )Reactants .

State Function 6-1



( )Initial state ()Final state

.
.


] (.[)1-1




1-1
.


() . .
:
.
H=Hf -Hi
f final i initial .
25
7-1
:
-1 ( :)Extensive Properties

...
-2 ( :)Intensive Properties

....

. 2 mole
1 mole .

8-1 Thermochemistry
( )Absorbed heat
() ( )Evolved heat

. Hr
.


: !

)2H2(g) + O2(g 2H2O(l) + Energy
( )Energy
.
)H2O(g )H2O(l + Energy

( )Energy

.
:

2HgO(s) + Energy )2Hg(l) + O2(g

H2O(s) + Energy )H2O(l


26




H < 0 H > 0
H = - H = +


9-1
(

)

( .)Calorimeter
.


( )
.
( )2-1

.


2-1
.

.

27
2-1
( )2-3 3 g
( )C6H12O6 ()180 g/mole
.
1200 g (
) 4.2 J/g.C
. 21.0C
.25.5C
kJ 1 mole .
.


T = )Tf -Ti(C = )25.5-21.0(C = 4.5C


.
(q )J( = )J/g .C( m )g( T)C

q )J( = 4.2 ) J/g


. C( 1200 )g( 4.5 )C( = 22680 J


( .)- 22680 J
1 mole
3 g .
( )n ( )m (.)M

(m )g (3 )g
= 0.017 mol = = (n )mol
(M )g/ mol( 180 )g/ mol

1 mole :

28
-22680 J
= - 1334118 J/ mol = q
0.017 mol


:
2-1 H = qP = - 1334118 J/ mol
3 g
J/ mol . kJ J (( )N2H4
)32 g/mole kJ :
1000 g
(1 )kJ
( ()4.2 J/g.C (H )kJ/mol( = H )J/ mol
(1000 )J
24.6C
.28.2C
(1 )kJ
(H )kJ/mol( = - 1334118 )J/ mol
(1000 )J
1 mole = - 1334 kJ/mol
kJ/mol
: .
- 161 kJ/mol : . - 1334 kJ/mol

10-1
(
) .
:
- 1
.
.
:
(H2O)s (H2O)l H = 6 kJ/mol

6 kJ/mol
25C .1 atm
( ) :
29
H2O(s)+ 6 kJ/mol )H2O(l

.
:

)CH4(g) + 2O2(g CO2(g) + 2H2O (l) H = -890 kJ/mol

() 890 kJ/mol
1 mole 25C .1 atm


:

)CH4(g) + 2O2(g CO2(g) + 2H2O (l) + 890 kJ/mol

H = + 6 kJ/mol H = -890 kJ/mol

r r

H > 0 H < 0
H = + H = -

- 2
.
( s solid )
( l liquid ) ( g gas ) ( aq
aqueous ) .

:
30
1
) O2(g H2(g) + )H2O(l H= -286 kJ/mol
2

1
) O2(g H2(g) + )H2O(g H = -242 kJ/mol
2

- 3 ( )
!
( )3 .
. )H2O(s )H2O(l H = 6 kJ/mol

)H2O(l )H2O(s H = -6 kJ/mol

- 4
.
)H2O(s )H2O(l H= 6 kJ/mol

)2H2O(s )2H2O(l H = 2 6 kJ/mol = 12 kJ/mol

11-1 Standard Enthalpy of Reaction



)298 K( 25C 1 atm
.H
r


()Standard Temperture and Pressure
( )STP )273 K( 0C .1 atm

)C5H12(l) + 8O2(g 5CO2(g) + 6H2O(l) H


r
= -3523 kJ/mol

:
C5H12 8

31
5 6
3523 kJ
25C .1 atm

12-1

Standard Enthalpy of Formation 1-12-1


f( H formation f

) ( )

25C .1 atm
.

25C
.1 atm

( )graphite ( )diamond
()Srhombic
( )Sorthorhombic .
(
H ( f
)IUPAC
) .

H (Element) = 0 kJ/mol
f

( )Element .


H (O ) = 0 kJ/mol
f 2
( )graphite ( )diamond
H . f
(C(graphite)) = 0 kJ/mol
:

32
1 (H O )= - 286 kJ/mol
) O2(g H2(g) + )H2O(l H )2 (l
2 f

P H +2O + 13 H3PO4(s) H (H PO ) = - 1279 kJ/mol


)2 2(g )2(g
)4 4(s f 3 )4(s

)Mg(s) + Cl2(g )MgCl2(s H (MgCl ) = - 642 kJ/mol


)2(s
f
H
f

)C(graphite) + O2(g )CO2(g H (CO ) = - 393.5 kJ/mol
f )2(g

2-1 H . f
( )2-1

H f
.
H .
f

()kJ/ mol
-32 )Ag2S(s H f

-1219 . )BaCO3(s
-795 )CaCl2(s .H f
H r

-75 CH H .-72 kJ r

)4(g

-239 )CH3OH(l
-393.5 )CO2 (g )H2(g) + Br2(L 2HBr(g) H r
= -72 kJ
+227 )C2H2 (g
-125 )C4H10 (g HBr .
-770 -36 kJ )CuSO4 (s
-391 HBr )KClO3 (s
-1278 )MgSO4 (l HBr :
-286 )H2O(l
-242 )H2O(g H H = -72(HBr) = 1 kJ/mol = -36 kJ/mol
f
2 r
2
-173 )HNO3 (l
-36 )HBr(g 2
1 1
+26 )HI(g )H + Br2(g )HBr(g
2 2(g) 2

33
3-1
C6H6
H )C H ( = 49 kJ/mol
f 6 6 3-1
) (
H .Hf C6H6

r
:

C6H6
H ( )H SO
25C .1 atm f 2 4

( . )- 8 1 1 1 k J / m o l

(6C)graphite( +3H2)g (C6H6 )l

H
r
H = H )C6H6( = 49 kJ/mol r
f
( . H)H SO
f 2 4
4-1
Hr
4-1
H . f

H
r
()1( 4Fe)s( + 3O2)g 2Fe2O3)s( H= -1625 kJ
r
()HF
H)HF( = -271 kJ/mol
()2( C)graphite( + O2)g (CO2)g H = - 394 kJ f
r
(H2)g( +F2)g (2HF)g
1
( O2)g )3( CO)g( + (CO2)g H = -283 kJ
2 r -542 kJ/mol :

:
( )1 5-1
Fe2O3 .2 mole
( )2 :
4Al +3O CO2 CO2 (2Al2O3)s
()s (2)g

. H = ? kJ
r
H ( )3 H ( )CO
f (2)g r
CO.
H ) (
f Al2O3 = - 1670kJ/mol

-3340 kJ:

34
Standard Enthalpy of Combustion 2-12-1
() .

.
.
-2873 kJ :
13
) O2(g C4H10(g) + 4CO2(g) + 5H2O(l) H
r = -2873 kJ/mol
2
.
( .)C8H18
5471 kJ .

C8H18 (l) + 25
) O2(g )8CO2(g) + 9H2O (l H
r = - 5471 kJ/mol
2

.
1
)H2(g) + O2(g H2O(l) H = - 286 kJ/mol
2 r

.
c( H combustion c

)
25C
.1 atm ( )3-1
.

.
.
C6H12O6
.

)C6H12O6 (s)+ 6O2(g 6CO2(g)+ 6H2O(l) H


r = - 2808 kJ/mol

H r

H c

. .

35
5-1
6-1

(( .)C2H5OH)l
H ) ( ( )C3H8
c C2H5OH )l( = -1367 kJ/mol
H ) (
c C3H8 = -2219 kJ/mol
:
(( )C2H5OH)l
O2 .-1367 kJ/mol
(C2H5OH)l( + 3O2)g (2CO2)g( + 3H2O)l
H ) (
c C2H5OH)l( = -1367 kJ/mol

3-12-1 Enthalpy of physical changes



.
( .)vaporization 3-1
vap( Hvap vaporization
H ) .
c
44 kJ :
H c
vaporization kJ/ mol
(H2O)l H2O)g( Hvap = 44 kJ/mol
-5644 (C12H22O11)s

-5471 . (C8H18)l
-2808 cond( Hcond condensation ). (C6H12O6)s
-2219 Hcond (C3H8)g
-891 Hvap . (CH4)g
-394 (C)graphite
condesation
(H2O)g (H2O)l Hcond= - 44kJ/mol
-286 (H2)g
-1367 (C2H5OH)l

-1411 ( )Fusion (C2H4)g
-298 (S)rhombic
-383 fus( Hfus fusion ) . (NH3)g
.6 kJ/mol

fusion
(H2O)s (H2O)l Hfus = 6 kJ/mol

36
() ()crystallization
cryst( Hcryst
crystallization ) .
:

crystallization
(H2O)l H2O)s( Hcryst = -6 kJ/mol

:
Hfus = - Hcryst Hvap = - Hcond
.
: .

6-1
. 23 kJ/mole
.

:
:
7-1

(NH3)l( vap. NH3)g HVap = 23 kJ/mol
()CH COOH
3
:
. 5.11 kJ/mol
.
(NH3)g( cond. NH3)l
-5.11 kJ/mol :
Hcond = - Hvap = - 23 kJ / mol

13-1

1-13-1


H r
.
()Hess law

.
37

H . r
H r
H r

( ) .
.

.

() :
1
) O2(g C(graphite) + )CO(g H ?=
2 r

.CO2
()
.CO2 CO CO2
:
)(1) C(graphite) + O2(g )CO2(g H
r
= -393.5 kJ/mol
1
) O2(g (2) CO(g) + CO2 H = -283 kJ/mol

2 r

CO Cgraphite O2 CO2
CO2 ( )2 :
1
)(3) CO2(g O2 CO(g) + H = +283 kJ/mol
2 r


( )1 ( )3 :
3-1
)(1) C(graphite) + O2(g CO2(g) H r
= -393.5 kJ/mol
1 CO )C(graphite
)(3) CO2(g O2 H CO(g) + = +283 kJ/mol ) O2(g
2 r
.
1
)(4) C(graphite) + O2(g CO(g) H = -110.5 kJ/mol
2 r

) C(graphite ) O2(g CO
H
r

110.5 kJ/mol ( )3-1 .
38
7-1
( CS2)l
.

(C)graphitle(+ 2S)rhombic (CS2)l


H f
)CS ( = ? kJ / mol
(2)l

()1 (C)graphite( + O2)g (CO2)g


H
r
= -394 kJ/mol

()2 (S)rhombic( + O2)g (SO2 )g


H
r
= -296 kJ/mol

()3 (CS2)l( + 3O2)g (CO2)g( + 2SO2 )g


H r
= -1072 kJ/mol

:
H ( )CS
f (2)l

CS2 ( )3
( )3 :

()4 (CO2)g( + 2SO2 )g (CS2)l( + 3O2)g


H r
= +1072 kJ/mol

H 2 ( )CS
f (2)l

S ( )2 2 :

()5( 2S)rhombic( + 2O2)g (2SO2 )g


H
r
= 2 )-296 kJ( = -592 kJ

39
( )4 ( )5 ()1

()1( C)graphite( + O2)g (CO2)g


H
r
= -394 kJ/mol

()4( CO2)g( + 2SO2 )g (CS2)l( + 3O2)g


H = +1072 kJ/mol 8-1
r

()5( 2S)rhombic( + 2O2)g (2SO2 )g ( C2H2 )g

H = -592 kJ/mol .
r

(2C)graphite( + H2)g (C2H2 )g

H )C H ( = ? kJ
(C)graphite( + 2S)rhombic (CS2)l f (2 2)g

H f = ? kJ/mol

:
H ( )1 ( )CS
f (2)l
(C)graphite( + O2)g (CO2)g

H )CS ( = -394 kJ + 1072 kJ + )-592 kJ( = 86 kJ/mol


f (2)l H
r
= -394 kJ/mol

()2
1
( O2)g H2)g(+ (H2O)l
2-13-1 2
H
r
= -286 kJ/mol
H
r
H f

( )3 . :
(2C2H2)g( + 5O2)g
aA + bB gG + hH
(4CO2)g( + 2H2O)l
H :
r

H
r
= -2599 kJ/mol

H = n H ()Products( - n H)Reactants
r f f 225.5 kJ/mol :

n ( )Reactants
( )Products () .
40
H :
r

H = [g H ]()G(+h H)H(] - [a H)A(+b H)B


r f f f f

H ( )Element f

.
H )Element( = 0 kJ/mol
f

8-1
()III
:
(2Al)s( +Fe2O3)s Al2O3)s( + 2Fe)l( H r
= ? kJ

:
9-1 H
f
)Al2O3)s(( = - 1670 kJ/mol
( )C6H6 H )Fe O ( = - 822 kJ/mol
f (2 3)s
H
f
)Fe)l(( = 12 kJ/mol
H
r
.
: :
H
f
)C6H6)l(( = 49 kJ/mol
H
f
)CO2)g(( = -394 kJ/mol :
H
f
)H2O)l(( = -286 kJ/mol H r
= n H f
)Products( - n H f
()Reactants
-3271 kJ/mol :
H
r
=[H
f
)Al2O3)s((+2H f
](()Fe)l
- [Hf)Al)s((+H
f
](()Fe2O3)s

H Al f
)Al)s(( = 0
H f
. (()Fe)l
.

H
r
= [)-1670(+2)12(] kJ/mol-[2)0(+)- 822(] kJ/mol
= - 824 kJ
41
14-1 Spontaneous Processes

.

(.)Spontaneous reaction
:
 .


.

.
 0C 0C
( .)1 atm
 ()
.
 Na NaOH
H2 H2 NaOH .

()
(.)4-1

.
.

( )
()
:
4-1
()
.
()

.
.
)CH4(g) + 2O2(g CO2(g) + 2H2O(l) H r
= - 890 kJ

42

)H+(aq) + OH-(aq )H2O(l H


r
= -56 kJ

 NH4Cl
.

)NH4Cl(s )NH+4(aq) + Cl-(aq H


r
= + 14 kJ

 HgO
.

)2HgO(s 2Hg(l) + O2(g) H r


= 91 kJ

.

)H2O(s H2O(l) Hfus = 6 kJ/mol

 .

)H2O(l H2O(g) Hvap = 44 kJ/mol



.

.
.

43
Entropy 15-1


( )S
.
.

( ) .
( ) . S

.
S = Sf - Si
S ( f final ) S
i f

( i initial ).

S ()
(
) . .
(NH4Cl)s (NH+4)aq( + Cl-)aq H
r
= + 14 kJ

.

.

(.)5-1


.

.

.
.
44


.
.

() () .
.
NaCl
: - ( )
- .

.

. .

)S(g) > S(l) > S(s



5-1


.

45
( )6-1
.


T1 T2

T1 (T2 > T1) T2

6-1

.

46
9-1

S
:

)
10-1
)
:
) 80C .20C
) .
:
)
)
.

) H2 20C (.)S < 0
.80C
)
) . (.)S > 0

)
(.)S > 0

) 80C 20C
(.)S < 0

1-15-1

S
S r

:

S
r
(= n S)Products( - n S)Reactants

n ( )Reactants
( )Products () .

47
: 4-1

aA + bB gG + hH

(S )J /K.mol
S
r
70 (H2O)l
: 189 (H2O)g
152 (Br2)l
S
r
](= [g S)G( + h S)H(] - [a S)A( + b S)B
27 (Fe)s
2 ( C)diamond
( )4-1
6 ( C)graphite
.
186 (CH4)g
(.)J/K.mol
230 (C2H6)g

10-1 43 (Ag)s
67 (Ba)s
S
r 205 (O2)g
25C .1 atm 223 (Cl2)g

(2CO)g( + O2)g (2CO2)g 214 (CO2)g

87 (Fe2O3)s
201 (C2H2)g
S)CO2( = 214 J/K.mol S)CO( = 198 J/K.mol 220 (C2H4)g
66 (CuSO4)s
S)O2( = 205 J/K.mol
11-1

:
S
S
r
(= n S)Products( - n S)Reactants r
25C .1 atm

S
r
](= [2 S)CO2( ] - [2S)CO( + S)O2 (4Fe)s( + 3O2)g (2Fe2O3)s

S = [2 214] J/K.mol - [2 198+205] J/K.mol
r S)Fe2O3( = 87 J/K.mol
S)O ( = 205 J/K.mol
2
S = -173 J/K.mol
r S)Fe( = 27 J/K.mol
-549 J/K.mol :

48
16-1 Gibbs Free Energy
1800 ( )Gibbs
( )H ( )S
.

G
H S .
G :
G = H - T S ( )

( )
. G
.
:
G ( )G > 0
.
G ( )G < 0
( ).
G ( )G = 0
.

1-16-1
Standard Gibbs Free Energy of Reaction
( G r

r reaction )
( 25C .)1 atm
G r

( )Standard Gibbs free energy of formation
( G f formation ) . f



25C .1 atm
49
G r

5-1
:


G = n G ()Products( - n G)Reactants
f f
r
G
f

()kJ/mol
n ( )Reactants
173 (C6H6)l
( )Products () .
-300 (SO2)g
: -137 (CO)g
aA + bB gG + hH -394 (CO2)g
87 (NO)g
: 52 (NO2)g
-348 NaCl

G = [g G ]()G(+h G)H(]-[a G)A(+b G)B -95 (HCl)g


r f f f f
-53 (HBr)g
-51 (CH4)g
( )5-1
-33 C2H6
G .
f 68 C2H4
.
-1576 (Al2O3)S
G )Element( = 0 kJ/mol
f -741 Fe2O3
-110 AgCl
( )Element -318 ZnO
(.)J/mol -229 (H2O)g
-237 (H2O)l
-273 (HF)g
11-1 2 (HI)g
-33 (H2S)g
25C .1 atm -17 (NH3)g
-604 (CaO)s
(2C6H6)l( +15O2)g (12CO2)g( + 6H2O)l

50
G )C H ( = 173 kJ/mol
f (6 6)l

G )CO ( = - 394 kJ/mol


f (2)g

G )H O ( = - 237 kJ/mol
f (2 )l

G
r
= n G
f
()Products( - n G)Reactants
f

12-1
G = [12 G ]()CO (+ 6 G)H O
G r f 2 f 2
r
- [2 G ]( )C H (+15 G)O
25C f 6 6 f 2

.1 atm
G
r
](= [12 )- 394 kJ/mol(+ 6 )- 237 kJ/mol
(2NO)g( + O2)g
]- [2 173 kJ/mol+15 0
(2NO2)g
G = -6496 kJ/mol
r

G
f
)NO( = 87 kJ/mol
)NO ( = 52 kJ/mol .
G
f 2

-70 kJ/mol : .
2-16-1

:
G = H - T S
G
. G
.
:
()
H

:

. S (
) . S
( )-T S S G
.
51

( S > 0 .)H < 0
H S G:
-1 H S G
T S .H T .
-2 H S G
.T
-3 H S G
.T
-4 H S G
T S .H T.
G 1 4
H S . ( )6-1
.

G G = H - T S 6-1
G S H
+ +
)2HgO(s )2Hg(l)+O2(g .
.
G - +
)3O2(g )2O3(g ( )
.
G + -
)2H2O2(l )2H2O(l)+O2 (g
.
- -
)NH3(g)+HCl(g )NH4Cl(s .
.

S
r
H
r

25C 1 atm :

G
= r
H
r - T Sr

52
12-1
:
(C2H5OH)l( +3O2)g (2CO2)g( + 3H2O)l
13-1 :
G r

25C H
f /kJ/mol
S/J/K.mol
.1 atm (C2H5OH)l -278 161
(C5H12)l( +8O2)g (O2)g 0 205
(5CO2)g( + 6H2O)l (CO2)g -394 214
(H2O)l -286 70
H r
:

H )
.H r = - 3536 kJ/mol r
S )
S r r

. S G . )
r = 374 J/K.mol
r
:
-3647.5 kJ/mol :
H )
r
H
r
= n H
f
)Products( - n H
f
()Reactants

]( Hr = [2Hf )CO2( + 3 Hf )H2O 14-1



- [ H )C2H5OH(+ 3H ]()O2
: f f

(HCOOH)l (CO)g(+ H2O)l


H
r
](= [ 2 )- 394( + 3 )-286(] - [ )- 278( + 3 )0
H
r
= - 1368 kJ/mol
16 kJ/mol
S S )
r r
(= n S)Products( - n S)Reactants
.234 J/K.mol S
r


S
r
](= [2 S)CO2( + 3 S)H2O
25C
](- [S)C2H5OH(+ 3 S)O2
.1 atm
S = [ 2 )214( + 3 )70(] J/K.mol
-53.7 kJ/mol : r

- [ )161( + 3 )205(] J/K.mol


S
r
= - 138 J/K.mol

)kJ/K.mol( = S (1 )kJ

S ()J/K.mol
(1000 )J
r r

53
(1 )kJ
()kJ/K.mol( = -138 )J/K.mol
S
r (1000 )J

S )kJ/K.mol( = - 0.138 kJ/K.mol


r
G
r
)
C .K
!
T )K( = t )C(+ 273 = 25 + 273 = 298 K

G H S H
r - T Sr
= r
r r
.
G )
r = - 1368 kJ/mol - 298 K - 0.138 kJ/K.mol
( J/K.mol

G . kJ/K.mol
r = - 1327 kJ/mol

G
r

25C .1 atm

17-1

Hvap Hfus

m( Tm melting )
() b( Tb
boiling ) .

G
. :
G = H - T S
0 = Htr - Ttr Str
:
H
tr = Str
Ttr
( tr transition ) .

( ) .
:
54
Hfus
= Sfus
Tm
( fus fusion ) .
( )
:
Hvap
! = Svap
Tb

( vap vaporization ) .

(
).

Trouten Rule

( )Trouten Svap
( .)Sublimation
( )85 J/K.mol :

. Hvap
= Svap = 85 J/K.mol
Tb

.
13-1
Hvap kJ/mol
.69C
:
15-1 C .K
T )K( = t )C(+ 273 = 69 + 273 = 342 K
: Hvap
(H2O)l( H2O)g = 85 J/K.mol = Svap
T b
Hvap = 44 kJ/mol
Hvap = 85 )J/K.mol( Tb )K( = 85 J/K.mol 342 K
.100C = 29070 J/mol
118 J/K.mol : J/mol kJ/mol:

1 kJ( )
(Hvap )kJ/mol( = Hvap )J/mol
(1000 )J
(1 )kJ
(Hvap )kJ/mol( = 29070 )J/mol
H )kJ/mol( = 29 kJ/mol (1000 )J
55
vap

21 K C
T (K) = t (C)+ 273
23
)q (J) = (J/g .C) m(g) T(C
23
q = C. T
40
H = n H )(Products) - n H(Reactants
r f f

47

S
r
)= n S(Products) - n S(Reactants
49
G = H - T S

50

G = n G )(Products) - n G(Reactants
r f f

55

Hvap
= Svap = 85 J/K.mol
Tb


Joule Unit
(.)J

( )Kelvin ( )T
(.)K

56
System


Surrounding
.


= +
Open System

.
Closed System

.
Isolated System
.
Heat Capacity
( )C ( )m
(. )J/C
specific Heat

(.)J/g .C
State function

.
Extensive Properties

Intensive Properties
.

57
Enthalpy

H .H
Exothermic Reaction
.
Endothermic Reaction
.
Standard Enthalpy of Reaction
H .
r

Standard Enthalpy of Formation
H
f

.
Standard Enthalpy of Combustion
H
c

.
Hess law
.
Spontaneous and Nonspontaneous

. .
Entropy
S .
Gibbs Free Energy
G
.
Standard Gibbs free energy of Formation
G
f

25C .1 atm

58

1-1 :
.

2-1 (.)SI

3-1 : .

4-1 ( ).

5-1 .

6-1 .

7-1 .

8-1()qp(.)H

9-1 360 g ( )86 J/C .


0.24 J/g .C :
10-1 6 g 21C .124C
( .)kJ ( )0.39 J/g.C
0.241 kJ :

11-1 ( )kJ 350 g 77C .12C


(.)0.14 J/g.C
-3.2 kJ :

12-1 34 g 25C .79C


4479.8 J : ( .)2.44 J/g.C

13-1 155 g 25C 40C


2.45 J/g .C : .5700 J .

59
4.5 g 14-1 276 J
.25C .0.13 J/g.C
496.8C :

( 2CO)g( + O2)g G
(2CO2)g r
15-1
:
H ( )
f CO = -110.5 kJ/mol ,
H ) (
f CO2 = -393.5 kJ/mol

S)O2( = 205 J/K.mol , S)CO( = 198 J/K.mol , S)CO2( = 214 J/K.mol


-514.4 kJ :

16-1 2.4 kJ/C


0.12C .
-288 J :

17-1 1.5 g ( CH3COOH = )60 g/mol


750 g ( .)4.2 J/g.C
24C 28C
.
-504000 J/mole :

18-1 ()STP
.

19-1 2.6 g )M= 26 g/mol(C2H2


. 130 kJ :
H ) (
f CO2 = -393.5 kJ/mol ,
H ) (
f H2O = -286 kJ/mol

227 kJ/mol :

H
f
20-1 ()Al2O3
H :
(c )Al
(4Al)s( + 3O2)g (2Al2O3)s H= - 3340 kJ
r

-835 kJ/mole ; -1670 kJ/mole :

60
21-1 .
25C .1 atm
()1( H2)g( + Cl2)g (2HCl)g H = - 185 kJ
r
()2( 2H2)g( + O2)g (2H2O)g H = - 484 kJ
r
H :
r
(4HCl)g( +O2)g (2Cl2)g( +2H2O)g H= ? kJ
r

-114 kJ :

22-1 25C .1 atm


1
( N2)g( + O 2)g ()1 (NO2)g H= 33 kJ
2 r

()2( N2)g( + 2O 2)g (N2O4)g H = 11 kJ


r

(2NO2)g (N2O4)g H = ? kJ H :
r
r

-55 kJ :
23-1 25C .1 atm
()1( FeO)s( +H2)g (Fe)s( +H2O)g H= 25 kJ r

1
( O2)g )2( 3FeO)s( + (Fe3O4)s H = - 318 kJ
2 r

1
( O2)g )3( H2)g( + (H2O)g H = - 242 kJ
2 r

(3Fe)s( + 4H2O)g (Fe3O4)s( + 4H2)g H = ? kJ H :


r r

-151 kJ :

( CaCO3)s 24-1 CaO)s( + CO2)g( :


S 160 J/K.mol H CaCO3 , CaO , CO2 r
f
) -1207 , -635 ,-393.5 ( kJ/mol :
H)1
r
G r
)2
)3
)3 1115.6 K 130.8 )2 178.5 )1 :
61
25-1 H 25C .1 atm
r
(2NH3)g( + 3O2)g( + 2CH4)g (2HCN)g( + 6H2O)g
:
H) (
f NH3 = - 46 kJ/mol
H ) (
f CH4 = - 75 kJ/mol
H) (
f HCN = 135 kJ/mol H ) (
f H2O = - 242 kJ/mol

-940 kJ :

26-1 ( HPO3)s :
(P4O10)s( + 4HNO3)l (4 HPO3)s( + 2N2O5)g H r = - 180 kJ

H :
f
H) (
f P4O10 = -2984 kJ/mol H) (
f HNO3 = -174 kJ/mol

H) (
f N2O5 = - 43 kJ/mol

-943.5 kJ/mole :

27-1 S 25C .1 atm


r
(N2)g( + 3H2)g (2NH3)g

:
S)N2( = 192 J/K.mol S)H2( = 131 J/K.mol

S)NH3( = 193 J/K.mol


-199 J/K.mol :

28-1 . .

29-1 :
) .
) .
) .
) .

62
30-1 . (.)SI

( 2H2)g( + O2)g S J/K.mol 31-1 (2H2O)g


r
:
H) (
f H2O = -242 kJ/mol ,
G ) (
f H2O = -228 kJ/mol

-94 J/K.mol :
S 25C .1 atm r
32-1
(SiH4)g( + 2O2)g (SiO2)s( + 2H2O)l
:
S)O2( = 205 J/K.mol S)SiH4( = 206 J/K.mol

S)SiO2( = 42 J/K.mol S)H2O( = 70 J/K.mol

-434 J/K.mol :

S 137 J/K.mol 25Cr


33-1
(2O3)g (3O2)g 1 atm S)O2( = 205 J/K.mol S .O3

239 J/K.mol :

34-1 .

35-1 CH3OH , H2 , CO
H : r
kJ/mol ( )-727 , -286 , -284
CO + 2H2 CH3OH
-129 kJ :

36-1 H S.

H = +126 kJ/mol S = + 48 J/K.mol A :

H = -12 kJ/mol S = -105 J/K.mol B :

A : B ; 2625 K 114.3 K

63
37-1 H S 25C
.1 atm

H = 11 kJ/mol S = 30 J/K.mol A :

H = 2 kJ/mol S = 113 J/K.mol B :


25C 1 atm
.
A : ; 366.7 K

( ) G = H - T S 38-1
- 1 .
- 2 .
- 3 .
-4 .
-5 II .

64

2
Chemical Equilibrium

:
.
.
Kc Kp .
.
.
.

.

65
1-2
Irreversible and Reversible Reactions

.

.
.

( )Irreversible reactions

) () (




:

- 1
.

)H2SO4(aq) + 2NaOH(aq )Na2SO4(aq) + 2H2O(l

- 2
:CO2

)Na2CO3(s) + 2HCl(aq )2 NaCl(aq) + H2O(l) + CO2(g

:
( )

.

66

( )Reversible reactions



) . (




CO2

CO2 .

:

- 1 :

)CH3COOH(l) + CH3CH2OH(l )CH3COOCH2CH3(l) + H2O(l

- 2 (
):

)H2(g) + I2(g )2HI(g

- 3 CO2
( ):

)CaCO3(s )CaO(s) + CO2(g

- 4 ( ):

)H2O(l )H2O(g

(

).
67
2 - 2

.
H2 N2 .NH3
Rf( Rf
f forward )

H2 N2 Rb( Rb
b backward ) .
( )
( )Rf = Rb
:
kf
)N2(g) + 3H2(g )2NH3(g
kb

kf kb .

[ ]H
[ ]NH
2

[ ]N
2
3

() ().

.


1-2

(.)1-2

3 - 2
(Reversible homogenous
)reactions
(
):
68
(N2O4)g (2 NO2)g - 1
(HCOOH)l( + CH3OH)l - 2 (HCOOCH3)l( + H2O)l

(Zn)s( + S)s (ZnS)s - 3



(
).
(H2O)l (H2O)g
(2 NaHCO3)s (Na2CO3)s( + H2O)g( + CO2)g
(2 Hg)l( + O2)g (2 HgO)s

4 - 2

445C
2 :
(H2)g( + I2)g (2HI)g


%78 HI %11
HI

%78 HI %11
.


.

.

kf
(H2)g( + I2)g (2HI)g
kb
69
Rf Rb
:
[Rf = kf ]H2[ ]I2

Rb = kb ]HI[2

5 - 2 Equilibrium Constant
:

aA + bB gG + hH

A B G H a b g h
.
:

Rf = kf ]A[a ]B[b ()1

Rb = kb ]G[g ]H[h ()2

kf kb .


( Rf = Rb )2 - 2 :

kf ]A[a ]B[b = kb ]G[g ]H[h ()3

2-2
( )3 :

kf ]G[g ]H[h .

= ()4
kb []A[ ]B a b

70
kf kb
( Keq eq equilibrium )
( )4 :
]G[g ]H[h
= Keq ()5
[]A[ ]B a b

Keq
( )kf ( .)kb




.

[ ]M Keq Kc( Kc
c concentration )
. ( )5

Kc :
]G[g ]H[h
= Kc
()6
[]A[ ]B a b

1 - 2
:
1 - 2
Keq kf
(N2)g( + 3H2)g (2NH3)g
4.24 kb
k 0.11 kb .0.02
f
.kf k .0.05 K.
eq b
0.0848 :

:
k0.11
= 2.2 =
f
= Keq
kb 0.05
71


( )P
Keq KP( KP
P Pressure ) .

( )5 :
g h
PG PH
= KP )(7
a
PA PB
b

KP (



)
.





(.)7

(
)
.


)2NaHCO3(s )Na2CO3(s) + H2O(g) + CO2 (g


.
.

72

(2NaHCO3)s (Na2CO3)s( + H2O)g( + CO2 )g

:
[]Na CO [ ]CO [ ]H O
2 3 2 2
= Kc
]NaHCO [2 3

(

).

.
:

[Kc = ]CO2[ ]H2O

KP:
Kp = PCO2 PH2O


:
(CaCO3)s (CaO)s( + CO2)g

3-2 [Kc = ]CO2


CO2 :

KP = PCO2

( CaCO3
)
( CaO
CO2
).
CaCO3 CaO (.)3 - 2

73
2 - 2

:
(2HBr)g (H2)g( + Br2)g ()1

(NO-2 )aq( + H2O)l -


(HNO2)aq(+ OH )aq ()2
2 - 2
(CaCO3)s( + 2HCl)aq
Kc
(CaCl2)aq( + CO2)g( + H2O)l ()3
KP
: .
()1 (2CO2)g
Kc = ]H 2
[[ ]Br2
(2CO)g( + O2)g( )1
]HBr[2
(3O2)g (2O3)g ()2
- ()2
]HNO [ ]OH [
= Kc
2
[]NO-2 (CO)g( + Cl2)g
(COCl2)g( )3

[ ]CO [ ]CaCl )3( H O + C


Kc 2 2
= (2 )g ()s

[]HCl 2 (CO)g( + H2)g ()4

(2ZnS)s( + 3O2)g
(2ZnO)s( + 2SO2)g( )5
3 - 2

( C6H5COOH)aq :
(C6H5COO-)aq(+H+)aq( )6
(3H2)g( + N2)g (2NH3)g( )1
(2NO2)g( + 7H2)g
(2Hg)l( + O2)g (2HgO)s( )2 (2NH3)g(+ 4H2O)l( )7

2 :
PNH 3

= KP ()1
PH PN 3
2 2

1 ()2
= KP
PO 2

74
6 - 2 Keq


.
Keq
.

.

4 - 2
3 - 2 Keq :
Keq 100C (2CH4)g (C2H2)g( + 3H2)g
(2NO2)g (N2O4)g
:
]H [ = 0.143 M ]C H [ = 0.05 M ]CH [ = 0.02 M
4 2 2 2

: :
]N2O4[ = 0.002 M
Keq Kc
]NO2[ = 0.017 M
6.92 : ]C2H2[ ]H2[3 ) 0.05() 0.143(3
Kc =
= =0.37
[ ]CH 2 ()0.02 2
4



(

) .


( )
:
75
5 - 2
:
(H2)g( + I2)g (2HI)g
0.5 mole H2 0.5 mole I2
430C Kc
5.29 .
:

(n )mol
= []M
(V)L

(0.5 )mol
= 0.5 mol/L = []H2[ = ]I2
(1)L

(H2)g( + I2)g (2HI)g

M / []H2 []I2 [2 ]HI


0.5 0.5 0.0

-x -x +2x

0.5-x 0.5-x 2x

[]HI 2
= Kc
[]H2[]I2

[]2x 2
= 5.29
[]0.5-x[]0.5-x

[]2x 2
= 5.29
[]0.5-x 2

x
x = 0.267 mol/L

]H2[ = ]I2[ = 0.5 - 0.267 = 0.233 mol/L
]HI[ = 2 0.267 = 0.534 mol/L
76



.
4 - 2 6 - 2
0.625 mole N2O4 :
5L .
(H2)g(+ Br2)g (2HBr)g
N2O4
0.4 0.025mole/L Kc .
( mole N2O4)g H2 Br2 (2NO2)g
. 425C
:
5 L Kc .0.25
. N2O4
:
(n ) mol( 0.625 )mol
= 0.125mol/L = = (]H2[ = ]Br2[ = 0.32 mol/L M )mol/L
]HBr[ = 0.16 mol/L (V)L (5)L

(N2O4)g (2NO2)g

M / []N2O4 [2 ]NO2

0.125 0.000

-x +2x

0.125-x 2x

[ ]N2O4 0.025 mole/L


0.125 - x = 0.025 x = 0.10 mol/L
]NO2[ = 2x = 2 0.10 = 0.20 mol/L

]NO2[2 ]0.20[2
= 1.6 = = Kc
[]N2O4 []0.025
77
7 - 2
:
(2SO3)g (2SO2)g (+ O2)g

5 - 2
SO3

3 atm
(2CO2)g (2CO)g(+O2)g
4 atm KP
.
0.8 mole
CO2
:

(2SO3)g (2SO2)g( + O2)g . . Kc
0.2 :
atm / 2SO3 2SO2 O2
3.0 0.0 0.0
-2x +2x +x
3.0-2x 2x x 6 - 2
:

( 3H2)g( +N2)g (2NH3)g

.
( PT :
) H2 N2
PT = PSO + PSO + PO
3 2 2
4 = )3-2x( + 2x + x H2
0.3 mole N x
2

x = 1 atm .0.2 mole H2

PSO = 3.0 - 2x = 3.0 - 2 1 = 1 atm N2


3
Kc .200
PSO = 2x = 2 1 = 2 atm
2
]N2[ = 0.3 mol/L :
PO = x = 1 atm ]H2[ = 0.4 mol/L
2

()2(2 )1
2
PSO2 PO2
= KP = = 4
2 )1(2
PSO3
78
7 - 2 Kc Kp
KP

! Kc :
ng

ng KP = Kc RT
:
( )g . -ng
Kc = Kp RT

ng :
!
)ng = ng (Pr oducts) ng (Re ac tan ts
R
Kc KP ng:
0.082 L.atm/K.mol
ng )1 .Kc = KP
ng )2 KP .Kc
7 - 2 ng )3 KP .Kc
Kc
8 - 2
. 500C KP
ng = -1 KC = 4.1 227C KP
1.5 10-5
.
.
:
(3H2)g( + N2)g (2NH3)g C K
0.06 :
T)K( = t)C( + 273 = 227 + 273 = 500 K
8 - 2 ng -1
 KP = Kc RT Kc 1.6 1000C = 4.1 0.082 500  = 0.1

:
9 - 2
(C)s( + CO2)g (2CO)g
. 300K
(NH4HS)s (NH3)g( + H2S)g
CO

.0.3 atm Kp Kc.
CO2 :
.0.6 atm KP = PNH PH S = )0.3( )0.3( = 0.09
3 2

10 atm :
ng = ng (Pr oducts) ng (Re ac tan ts) = 2 - 0 = 2
-ng -2
Kc = Kp RT = 0.09 0.082 300  = 1.5 10-4
79
8 - 2
:
-
- .

1-8-2
:
A B
Kc =100 :
[]B []B
= Kc = 100

[ ]B[ = 100 ]A
[]A []A

[ ]B
[ ]A B
.

Kc = 0.01 .
[]B []B
= Kc 0.01 =

[ ]A[ = 100 ]B
[]A []A
4-2
K
[ ]A
( )K>>1
[ ]B .A
:
.
.1

. (.)4 - 2
.2

. (.)5 - 2 5-2
K .3
( )K<<1


. .

80
10 - 2
Kc .
(2HF)g (H2)g( + F2)g Kc = 1 10-95
(2SO2)g( + O2)g 2SO3)g( Kc = 8 1025
(3H2)g( + N2)g (2NH3)g Kc = 1.1
:
.1 :
H2 F2
.
9 - 2 .2
Kc SO3
SO2 O2 .

.
.
()1 . 3
(2H2O)g (2H2)g( + O2)g ( N2 )H2
( )NH3 .
Kc = 1.1 10-81
()2
(2CO)g(+Cl2)g (2COCl2)g 2-8-2

Kc = 0.99
()3
.
(H2)g( + Cl2)g (2HCl)g
:
Keq = 4.4 1032
.
10 - 4
:
(H2)g( + Cl2)g (2HCl)g
:

]HCl[2
= Kc1
[]H2[]Cl2
:
(2HCl)g (H2)g( + Cl2)g

:
[]H2[ ]Cl2
= Kc2
]HCl[2
81
Kc1 Kc2
:
1
= Kc2
Kc1
:
( ) 10 - 2

2NO2 N2O4

Kp = 0.39

)H2(g) + Cl2(g )2HCl(g 227C Kc

: N2O4 2NO2
.
[HCl]2
= Kc1 0.0625 :
][H2][Cl2
1
:
2 11 - 2
1 H + 1 Cl :
HCl
2 2(g) 2 )2(g )(g
)3H2(g) + N2(g )2NH3(g

:
0.02 M
][HCl NH3 0.1 M N2
= Kc2
[H2]1/2 [Cl2] 1/2 .H2 Kc .
)NH3(g
Kc1 Kc2 Kc2 3 1
H2(g) + )N2(g
1: 2 2
2
0.5 :
)Kc2 = (Kc1 1/2
= Kc1
:


.
:
1 O ][CO !
C(s) + )CO(g = Kc1
2 )2(g
] [O 2
1/2

.

1 O ][CO2
CO(g) + )CO2(g = Kc2
2 )2(g
[CO] [O2]1/2
82
:

C + O 12 - 2 (CO2)g
[]CO2
()s (2)g = Kc3

[]O2
K
c3
K K K = K K : c3 c1 c2 c1 c2

0.2 mole 11 - 2
: 100C 0.36
(H2 + Br2 N2 4)g
2HBr O (2NO2)g
HBr
:
( 2NO2)g 1.0 L (N2O4)g
1 NO (NO2)g
2 (2 4)g

2.0 mole . :
2.4 mole :

]NO2[2
Kc1 13 - 2 = 0.36 =
[]N2O4

2000 K
( 2NO2)g (N2O4)g
:
Kc2 . Kc1
1 1
Br2)g(+ F
2 (2 2)g []N2O4 1
(BrF)g KP = 150 K = = 2.8 =
c2
]NO[2 0.36
1 3
Br2)g(+ (F2)g 1 NO
2 2 (NO2)g
(BrF3)g KP = 2.5 2 (2 4)g


1
2
Kc :
(BrF3)g (BrF)g( + F2)g
1
2

0.366 : []NO2
= = Kc3 0.36 = 0.6
]N2O4[1/2

83
9 - 2 Reaction Quotient Q Kc


( Q( )Q Quotient
)
. Q
:
Kc Q
. Kc
Q
Q :
.1 Kc = Q Kc
.
.2 Q )Q > Kc( Kc

()

().
Kc
.3 Q )Q < Kc( Kc

Q

() ().


12 - 2
3H2 + N2 2NH3 14 - 2
500C 0.06 ( :
.)mole/L (3H2)g( + N2)g (2NH3)g
[]H2 []N2 []NH3
0.002 0.00001 0.001 ()1 Kc . 9.0
0.354 0.000015 0.0002 ()2 )1 2.0 mole
0.01 5.00 0.0001 ()3 NH3, H2, N2
:
1.0 L
Q Kc
?
]NH3[2 )0.001(2 )2 5 )1(
= Q = = 12.510
(]H2[3]N2[ )0.002(3 )0.00001 ?
6 L :
Q Kc :
[ ] .
84
15 - 2
]NH3[2 )0.0002(2
2 L Q = = = 0.06 ()2
(]H2[3]N2[ )0.354(3)0.000015
1.4 mole
Q Kc HBr H2 Br2
. .

(H2)g(+ Br2 )g (2HBr )g ]NH3[2 )0.0001(2


= Q = = 0.002 ()3
[]H2[3]N2 ()0.01(3)5.00

Q Kc . : HBr H2
( ) Br2 KP
0.25 .
.
: 10 - 2 G Q
]H2[ = ]Br2[ = 1.866 mole :
]HBr[ = 0.468 mole aA + bB gG + hH
G G
:
]G[g ]H[h
G = G+ RT ln ()1
]A[a ]B[b

R
( )R = 8.314 J/K.mol T .K
( )1 :

G = G+ RT ln Q ()2

G
:
G = 0
Q
eq( Keq equilibrium ) . ()2
:
G= - RT ln Keq
KP
Kc
.
85

G Keq
. (. )2 - 2
2-2 G Keq G= - RT ln Keq :

G In Keq Keq
>1
( ) 0 0 =1
<1

( )2 - 2 Keq ( ) 16 - 2
G .
KP
13 - 2
G G= - 104 kJ/mol 25C

25C . 1 atm .1 atm


(CCl4 )l( + H2 )g (HCl )g( + CHCl3 )l (2H2O)l (2H2 )g(+O2 )g
Keq .
:
C . K :
T)K( = t ) C( + 273 = 25 + 278 = 298 K G f)H 2O(=-237kJ/mol
R = 8.314 J/K.mol
R G . 25C
( . )J G kJ J 8 10-84: .

1000 ()J
(G)J/mol( = G)kJ/mol

(1 )kJ
(1000 )J
!
= - 104000 J/mol (= -104 )kJ/mol : ( )ln
(1 )kJ
G= - RT ln Keq ( .)e :
Keq
( )ln -104000 )J/mol( = -8.314 )J/K.mol ( 298 )K( ln Keq
Keq Keq = 1.7 1018 : .
.
86
11 - 2 Le Chateliers Principle
( )Equilibrium Postion

.
(

)
(
)
:

1 - 11 - 2
:
)3H2(g) + N2(g )2NH3(g



.



. .

H2 N2
.

( )

H2 N2 .


.
.
:
87
)1 .
)2 .
)3 .
)4 .

( )6 - 2 6-2
:
(N2O4)g (2NO2)g :
N2O4 2NO2

2 - 11 - 2



:
)1 .
:
(2SO2)g( + O2)g (2SO3)g
ng ( )-1
( )
( )
22SO (2)g
mol
+ O
(2)g
1 mol
2SO(3)g
2 mol

SO3 3 mol 2mol

( )
7-2
: SO3 O2

SO2 : .
)2 . .
:
(2NO2)g (2NO)g( + O2)g
88
ng ()1
: (
)

NO2 .

( O2)NO
: .
)3 .
:
(2NO)g( + O2)g (2NO2)g (C)s( + O2)g (CO2)g
2 mol 1 mol 2 mol
3 mol 2mol

ng ( )0
.
8-2


.
. :


.
.
.
.

3 - 11 - 2

:

)1
( )Endothermic reaction
(.) H =+
)2
( )Exothermic reaction
(.)H =-
:
89
)
:
CaCO3)s( + (CaO)s( + CO2)g


( H ) .
()


CaCO3 .

(N2O4)g (2NO2)g


()
NO2 ( 7-2) .
9-2
N2O4 ( 7-2). () NO2
() N2O4
)
:
(2CO)g( +O2)g 2CO2)g( +

(H
) .
CO2

CO2 .


Keq :


( ) Keq ( ) Keq
( ) Keq ( ) Keq

90
4 - 11 - 2

.




Rf Rb

. .

.

12 - 2



: .
Keq .

14 - 2
17 - 2
:
:
(2Hg)l( + O2)g (2HgO)s
(2BrF5)g (Br2)g( + 5F2)g
H 858 kJ
H -181 kJ 298K KP
1000K KP
. 3.2 1020 KP 500K
. 7.4 10-16
298K .
KP 1500K
1000K .
:


KP
. 500K

91
15 - 2

:
(N2F4)g 2NF2)g( H = 38.5 kJ/mol
)1 .
)2 N2F4 .
)3 .
18 - 2
)4 .
:
(3O2)g (2O3)g
:
H = 428 kJ/mol
)1 H ;


.
?
NF2
)1
N2F4 Keq
.

)2 O2

.
.
)3 .

)4 .
;
. 19 - 2
)2 N2F4 C2H4 H2
N2F4 C2H6
: NF2 25C
.N2F4 . :
)3 (C2H4)g( + H2)g
ng ()1 C2H6)g( + 137 kJ/mol

.
)4 C2H6 .

( ) .
92

71
[G]g [H]h
= Kc
[A]a [B]b
79 Kp Kc
ng
KP = Kc RT

79

)ng = ng (Pr oducts) ng (Re ac tan ts

85
G = G+ RT ln Q

85
G= - RT ln Keq


Chemical Equilibrium
() ()

.

Irreversible Reactions

.

Reversible Reactions

.

Reversible homogenous reactions


.
93
Reversible Heterogenous reactions
.

Law of Mass Action


Equilibrium Constant Kc

.

Equilibrium Constant Kp

.

Reaction Quetient


.

Le Chateliers Principle

.

94

1-2 :
(N2)g( + 3H2)g (2NH3)g + 92 kJ - 1 :
.

- 2 Kc 500C 2 10-15 200C


4 10-12 .

(CO)g( + 2H2)g CH3OH)g( + 127 kJ - 3 :


.

- 4
.

(CO2)g( + C)s (2CO)g - 5 :


.CO2

N2)g( + O2)g( + 180 kJ (2NO)g - 6 :


Kc.

-7 :
N2)g( + O2)g( + 180 kJ (2NO)g

(Cl2)g( + H2)g 2HCl)g( + - 8 :


HCl .

( N2O4)g KP - 9 2NO2)g( :
= 3 NO2 3 atm N2O4

(4NH3)g( + 3O2)g (2N2)g( + 6H2O)l - 10 :


Kc = 1 1028 .

- 11 KP Kc
.
95
- 12 Kc
.
- 13
.
- 14 Kc .
- 15 ( )ng = -1 Kc .
- 16 Kc = 4 .
- 17 = ng .
- 18 .
- 19 .
- 20 KP
= Kc
RT
.

2-2 :
( 2SO2)g( + O2)g 727C - 1 Kc 2SO3)g( :
. 4.17 10-2 0.4 M SO3 0.1 M O2 0.02 M
SO2:
- .
- .
- .
- .
( CaCO3)s CaCO3 - 2 CO2)g( + CaO)s( :
:
- CO2 .
- .
- CaO .
- .
( l2)g( + F2)g Kc 1 106 - 3 2IF)g( :
200K 0.2 atm IF 4 10-3 atm F2
I2:
5 104 atm -
1 10-5 atm -
1 105 atm -
- .
96
1
( Li)g( + I2)g 300K 640.3 - 4 KcLiI)g( :
2
( 2LiI)g . Kc2Li)g( + I2)g( :
25.3 -
41 104 -
15.6 105 -
- .
- 5 K2CrO4 HCl :
(2CrO42-)aq( + 2H+)aq (Cr2O72-)aq( + H2O)l

:
-
- HCl
- K2Cr2O7
- .
- 6 NaOH .
- . Cr2O72-
- . H2O
- . Cr2O72-
- .
( N2)g( + 3H2)g KP 25C - 7 2NH3)g( + 92kJ :
5.5 10-5 Kc
22513.3 -
9 104 -
0.03 -
- .
( Br2)g KP 2558.4 4000K - 8 2Br)g( :
Kf 1.56 Kb:
0.2 -
20 -
7.8 -
- .
( H2)g( + I2)g KP = 2 - 9 2HI)g( :
( ) mole/L :

97
- HI 0.1 I2 0.05 H2 0.05
- HI 0.4 I2 0.1 H2 0.2
- HI 0.001 I2 0.0002 H2 0.0025
- .

3-2 :
n . = g
ng (Pr )oducts
()Products ng=< ngn(Re
g (Pr
()Reactants g (Re
n tan ts)
)oducts
ac )tsac tan
- 1
( A)g . - 2 : B)g( +
- 3 .
- 4 Kc= 0.3 . Q = 1
- 5 Kc .
- 6 Kc .
- 7 ( )ng = +1 .
- 8 .
SO2Cl2)g( + (SO2)g( + Cl2)g - 9 :
SO2 .

- 2 - 3 - 1 : 4-2

2NO2 ? 5-2 N2O4


)1 .
)2 .
)3 N2O4 .

( PCl5 )g 6-2 PCl3)g( + Cl2)g( :


92.5 kJ/mole :
- 1 .
- 2 Cl2 .
- 3 PCl3 .
- 4 .
- 5 .

98
( 2SO2)g( + O2)g 2SO3)g( : 7-2
.-198.2 kJ SO2 O2 SO3 .
- 1 .
- 2 .
- 3 SO2 .
- 4 .

8-2 ?
? .

( 2PbS)s( + 3O2 )g 9-2 (2PbO)s( + 2SO2)g


.
)1 .
)2 .

10-2
( CaCO3)s : (CO2)g( + CaO)s
- 1
- 2 CaO .
- 3 CaCO3 .
- 4 CO2 .
- 5 .

11-2 NOCl 240C :


( 2NOCl)g (2NO)g( + Cl2)g
1atm NOCl 0.64 atm:
- 1 Cl2 NO .
- 2 Kc .
4 10-4 -2 0.24 atm ; 0.12 atm -1 :

( N2O4)g 12-2 2NO2)g( :


PNO = 1.56 atm PN O = 0.377 atm 100C :
2 4 2
- 2 . KP - 1 Kc

0.21 ; 6.46 -1 :

99
(A)g 13-2 2B)g( :
. KP Kc

C / (B/)mole/L (A/)mole/L
200 0.843 0.0125 1
300 0.764 0.171 2
400 0.724 0.250 3

( N2)g( + 3H2)g 377C 14-2 (2NH3)g


Kc . 1.96 .

(2NH3 )g (N2)g( + 3H2)g -


1 N + 3 H NH3)g( -
(2 2)g (2 2)g
(NH3 )g 1 N + 3 H -
: 0.51 - 1.4 - 0.7 - (2 2)g (2 2)g

15-2 Kc 4.2 10-7 :


CO2 + H2O H+ + HCO3-
(CaCO3)s Kc 4.7 10-9 CO32- + Ca2+
HCO3- Kc 4.8 10-11 H+ + CO32-
Kc :
CO2 + CaCO3)s( + H2O Ca2+ + 2HCO3-
4.1 10 :-5

2A + B 3mole 16-2 3C
B C , A 6mole C
6mole .A A , C Kc = 1.5

9mole = C , 4mole = A :

17-2 Kc 19.9 2500K :


(Cl2)g( + F2)g (2ClF)g
]Cl2[ = 0.2 M ]F2[ = 1 M . ]ClF[ = 1.2 M

100
( A2)g A2 18-2 2A)g( :
( )STP %1 . A2 Kc? A
0.01 M A2 ?
0.002 M ; 4 10-4 :

( 3A)g Kc 147.6 19-2 aB)g( :


KP 6 27C a .
2 :

(NiO)s( + CO)g 20-2 Ni)s( + CO2)g( :


727C CO 304 Torr
1atm Kc( . .)1 atm = 760 Torr :
1.5 :

(PCl3)g( + Cl2)g 21-2 PCl5 )g( :


PCl3 Cl2
Cl2 1atm KP 1/6 Cl2 PCl3
.
2.8 atm ; 1.4 atm :

( 2HI )g (H2)g( + I2)g 22-2


H2 I2 HI
1 mole HI 2 mole I2 2 mole H2 :
- 1 .
Kc - 2

; 4-2 ]H2[ = ]I2[ = 1.25 mol/L ]HI[ = 2.5 mol/L - 1 :

23-2 :
(N2)g( +3H2)g (2NH3)g

( CO2)g( + H2)g 24-2 CO)g(+ H2O)g( :


CO2 H2 2000K
.3 mole Kc ? 4

]CO[ = ]H2O[ = 1 mol/L ; ]CO2[ = ]H2[ = 0.5 mol/L :

( CaO)s( + CO2)g 25-2 CaCO3)s( :


800C CO2 .0.235atm Kc .
374.8 :
101
26-2 4 g HF 2 L 27C
( 2HF )g kP (H2)g( + F2)g
.1.21 HF .20 g/mole
0.76 atm :
27-2 N2O4 NO2 %20 27C 1atm
. kP( . : .)100
4.92 :
kf
(H2)g(+I2)g (2HI)g
490C kb 28-2 :
kf=0.6256 kb 0.0136
872K 59 HI :
) .H2
) .
) .I2
29-2 KP 1000K:

(C)s( + 1 O2)g (CO)g KP = 2.9 1010


2
(C)s( + O2)g (CO2)g KP = 5 1020
(2CO)g( + O2)g (2CO2)g Kc :
2.4 1022 :
30-2 CO CO2 H2 .700K

Kc . 5.29
]CO[ = ]H2O[ = 0.606 mol/L ; ]CO2[ = ]H2[ = 1.394 mol/L :
( N2O4)g Kc 6 10-3 31-2 (2NO2)g
298K 1.5 10-2 35C
?

( 2SO3)g 32-4 2SO2)g( + O2)g( :


25C ]SO3[ = 0.002 mole/L: ]SO2[= 0.08 mole/L ]O2[= 0.01 mole/L
10C Kc 4 .

33-2 G G . G G ?

34-2 25C 1 atm


(H2O )l .1 10-14 GH+)aq( + OH-)aq( .
79881 J/mol : 102

Ionic Equilibrium
3

:
.
.
Kc
.
.
.
.
.

.
103
1-3


.

( ) .


. .

2-3

( )Solute ( .)Solvent

.

()Electrolytes
[ ( ])1-3

( )None-electrolytes ()[
( ])1-3
.

1-2-3
()
.

:
-1
.
-2
.
-3
( )Cations (.)Anions
-4
.
104
() () ()

1-3

:
.
.
:
() .
(

HCl
) NaCl Na2SO4 KNO3
() H2SO4 NH4Cl
(
( )

) :
H2O
() ( )HCl(g )H(aq
+
)+ Cl-(aq
). H2O
)NaCl(s )Na+(aq) + Cl-(aq
H2O
)Na2SO4(s )2Na+(aq) + SO42-(aq
H2O +
)KNO3(s )K(aq) + NO3-(aq
H2O
)H2SO4(l )2H(aq
+
)+ SO42-(aq

:
( )1 )2( )3(

( )1-3
.

:
105
1-3
Ba2+ OH-

0.03 M .

Ba)OH(2
(Ba)OH(2)aq Ba2+
()aq
(+ 2OH-)aq
0.03 M ( )
-0.03 M (+ 0.03 M + 2 )0.03 M ( )

0 0.03 M 0.06 M ( )
[ ]Ba2+ 0.03 M
[ ]OH- .0.06 M

.

1-3
() .
CH3COOH
( ) NH3 H2CO3
AgCl ( ) .
() ( HBr 0.25 M) 0.055 M :
+
(CH3COOH)aq (H)aq( + CH3COO-)aq ( KOH) . CaCl2 0.155 M
+
(NH3)g( + H2O)l (NH4)aq( + OH-)aq
+
(H2CO3)aq 2H)aq( + CO32-
()aq
-
(AgCl)s (Ag+)aq( + Cl)aq


(
)
( )Reversible reaction
) . (
( )

( ) .
106
1-3 ()

.
.

( )
+
-
( ( )H)aq ( ()CH3COO)aq
HCl ( )
HNO3 ( )
HClO4
+
H2SO4 ( (H)aq
-
( CH3COO)aq ( )CH3COOH)aq
NaOH ( )1-3
( )
KOH
Kc
Ca)OH(2

NaCl
KNO3 :
K2SO4 []H+[ ]CH3COO-
= Kc
[]CH3COOH

HF [ ]
HCN Kc .
H2CO3 2-3
HCOOH Kc .
(NH3 )g( + H2O)l (NH4+)aq( + OH-)aq
NH3 :
CH3NH2

AgCl
CaF2 ()

CH3OH .Kc
C12H22O11 ( = 1 kg/L
CH3COOCH3 )1kg = 1000 g :
107
(M)H2O = 21 + 116 = 18 g/mol
M
(m )g
= (M )mol/L
(M )g/mol( V )L

(1000 )g
=
(18 )g/mol( 1 )L
= 55.55 mol/L !
]
[NH4 [ ]OH-
+
Keq =
[]NH3[ ]H2O Kc

Kc = Keq ]H2O[ = Keq 55.55 [ ]H2O


+ .
[]NH4 [ ]OH-
Kc =
[]NH3

3-3
-
:
+
HCl H2SO4 CH3COOH
NH4+ C6H5NH3+ HSO4-
.H2PO4-
-
:
+
( NH3
C6H5NH2 ]Fe)H2O(5)OH(2[+ CH3COO- .)OH-


.

.
( )HCl
[ ]HCl ( ):
(HCl)aq( + H2O)l (H3O+)aq( + Cl-)aq
108

( ) .
( )
( )
:
)CH3COOH(aq)+H2O(l )H3O+(aq)+CH3COO-(aq

1-3-3 Weak Acids




.
:

)HCOOH(aq) + H2O(l )H3O+(aq) + HCOO-(aq


[]H3O+] [HCOO-
= Keq
[]H2O] [HCOOH

]Ka = Keq [H2O

][H3O+] [HCOO-
= Ka
[]HCOOH
( Ka a acid )
.
( )HA
:

)HA(aq) + H2O(l )H3O+(aq) + A-(aq

][H3O+] [A-
= Ka
[]HA


.
109

( )
.


.
3-3
+
H 0.1 M
()aq


.1.810-5
: !


.
+ -
(CH3COOH)aq (H)aq( + CH3COO)aq

[]H+[ ]CH3COO-
= Ka
[]CH3COOH

[ ]
.

: .
+ -
(CH3COOH)aq (H)aq( + CH3COO)aq
0.1 M 0 0 ( )
)0.1 - x( M xM xM ()


( )x mole/L
( )x mole
( )x mole
.
:

()x( )x
= 1.8 10-5
()0.1 - x
110
:
(x2 = )1.8 10-6( - )1.8 10-5 x

()
Ka
( )x
:
)0.1- x( mol/L 0.1 mol/L
:
()x( )x
= 1.8 10-5
0.1
! x2 = )0.1( )1.8 10-5( = 1.8 10-6
=x1.8 10-6 = 1.3 10-3

]H+[ = 1.3 10-3 mol/L
% 5
(
x ( )1.3 10-3 mole/L
) ( )0.1 mole/L

% 5 ( ) (
10-7 10-6 10-5 ) Ka
. ( 10-2 10-3 10-4 )

( )a x2 + b x + c a b .
c x2 :
x
.
( ) =
:

(()-b )b2 - 4ac


=x .100
2a


% 100 ( ) =

= 100
111
(:)3-3


=

1.3 10-3
= 0.013 =
0.1

% 100 ( ) =

1.3 10-3
% 100 ( ) =
0.1
( ) = % 1.3
% 5 .

4-3
HF
( )Ka = 6.8 10-4 () 0.1 M ()
0.01 M .
:
() 0.1 M
2-3
(HF)aq (H )aq( + F )aq
+ -


[]H+[ ]F-
= Ka
[]HF ( )Ka= 4.9 10-10
Ka Ka
.0.2 M
% 5
1 10-5 M :
.
( : )
:
(HF)aq (H+)aq( + F-)aq
0.1 M 0 0 ( )
)0.1 - x( M xM xM ( )

()x( )x
= 6.8 10 -4
()0.1 - x
112

)0.1- x( mol/L 0.1 mol/L
:

()x( )x
= 6.8 10-4
0.1

x2 = )0.1( )6.8 10-4( = 6.8 10-5


=x6.8 10 -5
= 8.25 10-3
]H+[ = 8.25 10-3 mol/L

=


= 0.0825
= 8.25 10-3
0.1
=


% 100

% 100
= 8.25 10-3
0.1
= % 8.25
% 5
10-5 .

(x2 = )6.8 10-5( - )6.8 10-4 x
x2 + )6.8 10-4 x( - )6.8 10-5( = 0
(()-b )b2 - 4ac
=x
2a

-)6.8 10-4( ()6.8 10-4(2- 4 )-6.8 10-5


=x
113 2

(
).
[x = 7.91 10-3 mol/L = ]H+

(
)
.

=

7.91 10-3
= 0.079 =
0.1

% 100 =

7.91 10-3
% 7.91 = % 100 =
0.1
() 0.01 M
:
()x( )x
= 6.8 10-4
()0.01 - x
3-3
: x
x = 2.29 10 mol/L
-3

)Ka=1.3 10-10( C6H5OH
=
() 0.2 M ()
2.29 10-3 .
= 0.229 =
0.01 :0.51 10-5 M -

% 100 = 0.51 10-6 M -

2.29 10-3
% 22.9 = % 100 =
0.01
114

( )
( 0.079 .)0.229 -

( )
()
( ) ()
.


()Monoprotic weak acids

( ) .

()Polyprotic weak acids
( )H2C2O4
(

! )
Ka1 Ka2 :
*
+
. )H2C2O4 (aq) + H2O(l )H3O(aq) + HC2O4-(aq
**
][H3O+] [HC2O4-
Ka =Ka1 = 5.37 10-2
][H2C2O4
.

- +
)HC2O4(aq) + H2O(l H3O(aq) + C2O42-
)(aq

][H3O+] [C2O42-
=Ka2 = 5.1 10-5
][HC2O4-
( )H3PO4
( Ka1 Ka2 :) Ka3
+
)H3PO4(aq) + H2O(l )H3O(aq) + H2PO4-(aq

[]H3O+] [H2PO4-
= Ka1 = 7.11 10-3
[]H3PO4
115
- +
(H2PO4)aq( + H2O)l H3O)aq( + HPO42-
()aq

[]H3O+[ ]HPO42-
= Ka2
[ ]H2PO4 -
= 6.32 10-8 !

HPO 2-
(4)aq
(+ H2O)l H3O +
()aq
+ PO 3-
(4)aq
:
[]H3O+[ ]PO43-
= Ka3 = 4.80 x 10-13 Ka1 > Ka2 > Ka3
[ ]HPO4 2-


2-3-3 Weak bases

( )
b( Kb base .

) . :
+ -
(NH3)g( + H2O)l (NH4)aq( + OH)aq
!
[]NH4+[ ]OH-
= Kb = 1.8 10-5
[]NH3

[ ]H2O .Ka
B : .
+ -
(B)aq( + H2O)l (BH)aq (+ OH)aq

[]BH+[ ]OH- Kb:


=Kb
[]B
5-3
[ ]OH-
0.2 M
.Kb )NH3( = 1.8 10-5.
:


:

116
(NH3)aq( + H2O)l (NH4+)aq( + OH-)aq
( 0.2 M ) 0 0
( )0.2 - x( M ) xM xM

[]NH4+[ ]OH-
= Kb
[]NH3
()x( )x
= 1.8 10 -5
()0.2 - x

x2 = 0.2 1.8 10-5 = 3.6 10-6


x = ]OH-[ =1.9 10-3 mol/L


=
4-3


1.9 10-3
C6H7N = 0.0095 =
0.2
( )Kb = 3.8 10-10
.0.1 M =
( )
6.2 10-5 : % 100

1.9 10-3
% 0.95 = % 100 =
0.2

4-3 Self-Ionization of Water


(
Autoionization of water
)Autodissociation of water
H3O+
OH-
.
( ) .
117

.
-
)H2O(l) + H2O(l )H3O+(aq) + OH(aq

:
+ -
)H2O(l H )(aq + OH )(aq


.Kw
] Kw = [H3O+][OH- ]Kw = [H+][OH-

H3O+
H+ OH-
()
25 C:
[H+] = [OH-] = 1.0 10-7 mol/L

Kw :25 C
Kw = [H+] [OH-] = (1.0 10-7) (1.0 10-7) = 1.0 10-14
Kw
.25 C
( )

( = ][H3O+
)[OH-] = 1.0 10-7 M
( )
:
+ -
)H2O(l )H(aq) + OH(aq

Kw
( )1.010-14 H+
OH- .
( )
OH- .H+
H+ OH- :
118
25 C

[]H+[ > ]OH- ]H+[ > 1.0 10-7 M , ]OH-[ < 1.0 10-7 M

[]H+[ = ]OH- ]H+[ = 1.0 10-7 M, ]OH-[ =1.0 10-7 M

[]H+[ < ]OH- ]H+[ < 1.0 10-7 M , ]OH-[ >1.0 10-7 M

6-3
H+ OH- 0.05 M
(.)HNO3
:
( )
H+ NO3- .
Kw = ]H+[ ]OH-[ = 1.0 10-14 OH- 5-3
.
+ -
(HNO3 )aq H (+ NO3)aq
()aq

0.05 M 0 0
:
0 0.05 M 0.05 M
() 0.01 M ().2.0 10-9 M

]H [ = ]NO3 [ = 0.05 mol/L :1 10-12 M -


+ -

5 10-6 M -
[ ]OH-
.Kw
H2O (H+)aq + (OH-)aq
( ) 0.05 M
( ) )0.05 + x( M xM

(Kw =1.0 10-14 = ]H+[ ]OH-[ = )0.05 + x( )x

( x H+ )
H+
HNO3 ( )x + 0.05
.0.05 :
119
(1.0 10-14 = )0.05( )x

= x
1.0 10-14
= 2.0 10-13 mol/L = 2.0 10-13 M
!
0.05 [ ]H+ []OH-


.
( x
)OH- .0.05

5-3 The hydrogen-ion Exponent - pH


( )
H+ OH-
( )Sorensen 1909
( ) pH
:
1
pH = - log ]H+[ = log
[]H+

]H+[ = 10-pH


H+ OH- 0 .14
:

1
pOH = - log ]OH-[ = log
[]OH-

]OH-[ = 10-pOH
6-3
( )pH 7-3
( :) 6.0 10-4 pH H+
mol/L ( ) .0.05 mol/L
:
0.03 M .
pH
[ ]H+ : :3.22 -
]H+[ = 0.05 M 1.22 -
pH = - log ]H+[ = - log 0.05 = 1.3
120
8-3
! pH 3.301 H+
:
p .pH = - log]H+[ pH
( H+ )OH- [.]H+
[pH =- log]H+
p [3.301 = - log]H+
( :
.)p pH ]H+[ = 10-3.301 = 5.0 10-4 mol/L
H+
pOH pH pOH
OH- pK 25C:
.K ]H+[]OH-[ = Kw = 1.0 10-14

( pKw log ]H+[ + log ]OH-[ = log )1.0 10-14 .pKw = - log Kw

( )- 1
7-3
()- log ]H+[ ( + )- log ]OH-[( = -log )1.0 10-14
pH
3.32 pH + pOH = 14
4.79 10-4 M :
pH pOH
14 14 8-3
.
. pH pOH
pH pOH
25C:

[]H+ []OH- pH pOH
0.15 M HI
25 C
0.06 M RbOH
0.02 M Ba)OH(2 pH < pOH pH < 7 < pOH
0.0003 M HClO4

pH = pOH pH = 7 = pOH

pH > pOH pH > 7 > pOH


121
9-3
[ ]H+ pH [ ]OH- pOH
pH
.0.015 M

:
HCl [ ]H+
.pH
[ ]OH- .pOH
(HCl )aq (H+)aq( + Cl-)aq


:
]H+[ = 0.015 mol/L

pH = - log]H+[ = - log)0.015( = - )- 1.82( = 1.82

pH + pOH = 14:

pOH = 14.00 - 1.82 = 12.18

]H+[ ]OH-[=1.0 10-14 []OH-

]OH-[ = 1.0 10 = 1.0 10


-14 -14
= 6.7 10-13 mol/L
[]H+ 0.015
pH

Solvolysis 6-3 .pH

CO2
( : ) (.)hydrolysis .


9-3
H+ .OH-
[ ]H+ pH []OH-
(
pOH Ca)OH(2
) ( )A- ( )HA
.0.015 M
()

H+ OH-
:
122
)A-(aq )+ H2O(l )HA(aq + )OH-(aq

OH-

CH3COO- :
)CH3COO-(aq) + H2O(l )CH3COO H(aq)+OH-(aq
( )
.
NH4+(aq) + H2O )NH3(aq) +H3O+(aq



.

:
.1 .
.2 .
.3 .
.4 .
.

1-6-3

.

. ( NaCl
NaOH ) HCl
:

H2O
)NaCl(s )Na+(aq) + Cl-(aq

)H2O(l )H+(aq) + OH-(aq

123
Na+ Cl- H+
.OH- Na+
Cl-
.

( )
OH- H+.

2-6-5

(
) .OH-
( CH3COONa
NaOH )CH3COOH :

H2O
)CH3COONa(s )Na+(aq) + CH3COO-(aq
)H2O(l )OH-(aq) + H+(aq OH-

CH3COOH
CH3COOH
H+ .CH3COOH H+
H2O
OH- .
:
)CH3COO-(aq) + H2O(l )CH3COO H(aq) + OH-(aq

h( Kh
hydrolysis ) :

][CH3COOH] [OH-
= Kh
][CH3COO-

( )HA:
124
[]HA[ ]OH-
= Kh
[]A-
[ ]HA [ ]OH-
[ ]A- ( )

. Kh
[ ]H+:

[]HA[ ]OH- []H+


= Kh
[]A- []H+
[]HA []OH[ ]H+
= Kh
[ ]A [ ]H
- +
1
1 Kw Kw
= Kh =
(Ka)HA 1 (Ka)HA

10-3

Ka)CH3COOH( = 1.8 10-5
( .)Kw = 1.0 10-14 : Kw
.
:
NaOH
CH3COOH
Ka Kw :
Kw 1.0 10 -14
=Kh = = 5.6 10-10
(Ka)CH3COOH 1.8 10 -5


:
Kw
= Kh
Ka

125
pKh = pKw - pKa

Ka
pKa pKw pKh Ka Kw Kh
. H+
HA OH-

[ . ]HA[ = ]OH- :c M

[]HA[ ]OH- ]OH-[2 Kw


= Kh = =
[]A- c Ka


Kw
c
= []OH- Ka Kw
= [ ]H+
[ ]OH -


Kw Ka
= [ ]H
+
c
!
pH = 1
[ ]pKw +pKa+ log c
2

11-3
[ ]H+ pH
pH 0.01 M
.
25C .Ka)CH3COOH( = 1.8 x 10-5
:
pH : 10-3
1
[ ]pKw +pKa+ logc = pH
2

pH = 1
[ ]-log Kw -log Ka+ log c
2 Ka )HCN(= 4.9 10-10 KCN
pH = 1 .0.1 mole/L [ ]-log )1.0 10-14( - log )1.8 10-5( + log 0.01
2
1
)14 + 4.74 - 2( = 8.37 = pH
2 1.43 10-3 M : .

126
3-6-3

( )
.H+
H+ OH-
]]H+ .
( NH4Cl NH3
)HCl :

H2O
)NH4Cl(s )NH4+(aq) + Cl-(aq
H2O(l) + )OH-(aq) + H+(aq H+

NH3 + H2O

( NH4+ )NH3
OH- NH3 .H2O
OH-
H2O H+
.

NH4+:

NH4+ + H2O NH3 + H3O+

][NH3] [H3O+ Kw
= Kh =
][NH4+ Kb
Kw
= Kh Kb
Kb
.
127
:
M+ + H2O MOH + H+

[]MOH[ ]H+ Kw
= Kh =
[ ]M
+ Kb

[ ]MOH [ ]H+
[ ]M+ (
) . [ ]MOH [ ]H+

:c M

]H+[2 ]H+[2 Kw
= Kh = =
[ ]M + c Kb
:


Kw
= [ ]H
+ c
Kb

:
[ ]pKw pKb log cpH = 1
2

Kb .

12-3
pH
0.2 mole/L
.Kb)NH3( = 1.8 10-5
:
pH Kb
Kw pH
:
[ ]pKw - pKb- log cpH = 1
2
128
1
[ ]-log Kw + log Kb- log c = pH 11-3
2
pOH
1
[ ]-log )1.0 10 ( + log )1.8 10 ( - log 0.2 = )pKb )NH3( = 4.74 ( pH
-14 -5

2
.0.5 M
9.22 : )14 - 4.74 - )- 0.7(( = 4.98pH = 1
2

7-3 Common Ion Effect


( )


( )
.
.
.





.



.

13-3
)0.1 mole( 8.2 g
CH3COONa
0.1 mole/L H+ 25 C
.Ka )CH3COOH( = 1.8 10-5
:
( )
( )
129 :
(CH3COOH)aq (H+)aq( + CH3COO-)aq
0.1 M 0 0 ( )
)0.1-x( M xM xM ( )
(CH3COONa)s (Na+)aq( + CH3COO-)aq
0.1 M 0 0 ( )
0 0.1 M 0.1 M ( )


( )
( ) .
.
H+
0.1 M :

[]H+[ ]CH3COO- ()x( )x


= Ka = 1.8 10 -5
=
[]CH3COOH ()0.1 - x

()x( )x
= Ka = 1.8 10-5
()0.1

x2 = )0.1( )1.8 10-5( = 1.8 10-6

=x 1.8 10 -6 =1.3 10-3 12-3


26.75 g
]H [ = 1.3 10 mol/L
+ -3
( )0.5 mol

H+ (
0.1 M
) :


( ) .Kb )NH3( = 1.8 10-5

)0.1 + x( M x :

H+ .360

.
130
[]H+[ ]CH3COO-
= Ka = 1.8 10 -5
[]CH3COOH
()x( )0.1+ x ()x ( )0.1
= =
( )0.1 - x ()0.1
[x = 1.8 x 10-5 mol/L = ]H+
[ ]H+
( )
1.3 10-3 mole/L 1.8 10-5 mole/L

( ).

8-3) ( Buffer Solutions


pH
pH 10-4 M
4 ( )

. pH
1 M
7 ()

1 mL HCl 1.0 M
pH NaCl pH = 3.0 pH
CH3COONH4 .


:
.1
( CH3COOH .)CH3COONa
.2
( NH3 .)NH4Cl

131



( ) pH
()
.
CO2

pH pH
()
( ) pH CO2
pH .
H+
( )Buffer action pH.

(
H+ ) ( ).


.

.
(
) ( )
pH
HCl
:

H+ pH

H+
CH3COO-
( )
pH .

132
NaOH
OH-
pH .
( CH3COOH )CH3COONa
OH-
(
) pH .


pH HCl

pH . NaOH

pH ().

( HA ) MA:
)HA(aq )H+(aq) + A-(aq
)MA(s )M+(aq) + A-(aq

:
] Ka = [H ] [A
+ -

][HA

][H+] = Ka [HA
][A-
( ) [ ]H+
(
)
[ ]HA ()
[ ]A- .
:
][acid
[H+] = Ka
][salt

[ ]acid [ ]salt
:
133
[]acid []salt
pH = pKa- log = pKa + log
[]salt []acid

[]base
]OH-[ = Kb
[]salt

[]base []salt
pOH = pKb- log = pKb + log
[]salt []base

[ ]base [ ]salt .

14-3
H+ pH 0.1 M
0.2 M .
:

H3O+ pH 13-3
:
[]acid
]H+[ = Ka ( )pH NH3
[]salt
0.15 mole/L NH4Cl
0.3 mole/L Ka )CH3COOH( = 1.8 10 ]acid[ = 0.1 M
-5

pH ]salt[ = 0.2 M :
0.1
]H+[ = 1.8 10-5 = 9.0 10-6 .0.15 M .pKb = 4.74
0.2
11.22 ; 8.96 :
pH :
pH =- log ]H+[ =- log 9.0 10-6 = 5.04
:
[]salt
pH = pKa + log
[]acid
134
pKa = -log Ka :

0.2
pH = - log 1.8 x10-5 + log
0.1

pH = 4.74 + log 2 = 4.74 + 0.30 = 5.04

H+ pH
( )
0.1 M
( )]H+[ = 1.35 10-3 M ( )pH= 2.87 .

15-3

( )pKb= 4.74 0.1 M pH 9.0
:
pH + pOH =14
pOH = 14 pH = 14 9 = 5
pH 9.0 pOH
.5.0

14-3

:
[]salt
pOH = pKb + log
[]base
0.3 mol/L
[]salt
5.00 = 4.74 + log pH 4.31
0.1
. Ka = 1.8 10-5
[]salt
0.81 M : log = 5 - 4.74 = 0.26
0.1
[]salt
= 100.26 = 1.82
0.1

]salt[ = 0.1 1.82 = 0.182 mol/L

135
16-3
( )pH 1 mL
10 M
0.1 M .0.1 M
( :
) . .pKa = 4.74
:

:
- +
(CH3COOH )aq (CH3COO)aq( + H)aq
- +
(CH3COONa )s (CH3COO)aq( + Na)aq

.1 ( )
( ) :
( = MCH COOH x V )L
3
= 0.1 mol/L x 1 L = 0.1 mol
( = MCH COONa x V)L
3
= 0.1 mol/L x 1 L = 0.1 mol

.2 HCl :
(HCl)aq (Cl-)aq( + H+)aq

H+
( = MHCl x V)L H+
1L
= 10 mol/L 1 mL = 0.01 mol
1000 mL

H+ HCl
.

.3
:
136
H+ + =

15-3 = 0.1 mol + 0.01 mol


()pH = 0.11 mol
() (n )mol (0.11 )mol
= [ ]CH3COOH 0.1 M = = 0.11 mol/L
(V )L (1 )L
( 0.1 M)
1 mL H+ - =

10 M
= 0.1 mol - 0.01 mol
pH . = pKb = 0.09 mol
( 4.74
(n )mol (0.09 )mol
= [ ]CH3COO -
= = 0.09 mol/L
(V )L (1 )L
).
.4 pH :
: ; 9.26 -9.16 -
[]salt
pH = - 0.1 pH = pKa + log
[]acid
0.09
pH = 4.74 + log
0.11
pH = 4.74 + )-0.087( = 4.66

9-3 Solubility and Solubility Product



( )


.

( )
( )
137

( )AB

)AB(s) + H2O(l )A+(aq) + B-(aq


( )Keq : .

][A+[ ]B- )(NH4)2S(aq)+ Cd(NO3)2(aq


= Keq
][AB] [H2O
)CdS(s) + 2NH4NO3(aq


[]AB

[ ]H2O
:

]Ksp = [A+[ ]B-

]Ksp = Keq [AB] [H2O

Ksp
( )

.
(
) (
)s
. s

( ).
138
AgCl
BaSO4 PbSO4 :
Ksp = s s = s2

CaF2 Zn)OH(2
:
Ksp = s )2s(2 = 4s3
Ca3)PO4(2
:
Ksp = )3s(3 )2s(2 = 108 s5

.

17-3
PbSO4
.Ksp = 1.6 10-8
:
:PbSO4
(PbSO4)s Pb2+
()aq
+ SO42- ()aq

PbSO4 s mole/L
[ s = ]Pb2+ [ s = ]SO42-
PbSO4 Pb2+
.SO42-
Ksp :
(PbSO4 )s Pb2+
()aq
+ SO42- ()aq

(s )mol/L (s )mol/L
[Ksp = ]Pb2+[ ]SO42-
Ksp = s s = s2 = 1.6 10-8
= s1.6 10-8 = 1.26 10-4 mol/L
139
18-3
Ksp
0.0025g BaSO4.
:
BaSO4
.Ksp
( )0.0025 g/L ()s mole/L 16-3
.
( ) Ag2CrO4
() : ()M = 332 g/mole
(BaSO4)s Ba2+()aq
+ SO42- ()aq 0.0215 g
[Ksp = ]Ba2+[ ]SO42- .
( )s )M = 233 g/mole( BaSO4 1.09 10-12 :
:
= (s )mol/L

1)mol( BaSO4
()g/L 17-3
233 )g( BaSO4

1 )mol( BaSO4 ( )g/L AgCl


(s BaSO4 )mol/L( = 0.0025 )g/L ( )M =143.5 g/mol
233 )g( BaSO4

s BaSO4 = 1.1 10 mol/L
-5
.Ksp )AgCl( = 1.8 10-10
BaSO4
1.3 10-5 mol/L :
Ba2+ SO42-:
1.93 10-3 g/L
(BaSO4 )s Ba2+
()aq
+ SO42- ()aq

1.1 10-5 M 1.1 10-5 M 1.1 10-5 M


]Ba2+[ = ]SO42-[ = 1.1 10-5 M
Ksp:
Ksp = ]Ba2+[ ]SO42-[ = )1.1 10-5( )1.1 10-5( = 1.2 10-10

140
Ksp

.


)
(

()

.

.

19-3
F- .2 10-2 M

.)K = 4.9 10-11 ( CaF2
sp
:
18-3
( )pH :
( )III -
(CaF2)s Ca2+
()aq
(+ 2F )aq
2 10-10 M

( )III
( ) ( Ksp ()III

K . 5 10-38
sp
4.8 :
) .

.2 10-2 M :
Ksp =)x M() 2 10-2 M (2
4.910-11 = )x()2 10-2 M (2
x = 1.23 10-7 M
141
1-9-3
(
)
.pH
.1


()

( ).

.
.2

( )


( ).
20-3
Ba)IO3(2
(( )Ksp= 1.57 10-9) ( )
KIO3 0.02 mole/L .
:
() Ba)IO3(2
Ksp
.

= s Ba)IO3(2
(Ba)IO3(2 )s (Ba2+)aq( + 2IO3-)aq
sM sM 2s M
Ksp = ]Ba2+[ ]IO3-[2 = )s( )2s(2 = 1.57 10-9

142
( s ) :
s = 7.3 10-4 mol/L

Ba)IO3(2
.7.3 10-4 M
() KIO3
IO3- :
(KIO3 )s (K+)aq( + IO3-)aq
0.02 M 0.02 M 0.02 M
= y Ba)IO3(2 KIO3
.0.02 M
(Ba)IO3(2 )s (Ba2+)aq( + 2IO3-)aq
yM yM 2y M
IO3- []IO3-
( 0.02 mole/L
19-3
)KIO3 ( 2y mole/L

)Ba)IO3(2 )0.02 + 2y( mole/L:
K = ]Ba2+[ ]IO3-[2 = )y( )0.02 + 2y(2 = 1.57 10-9
sp MgF
2

( .Ksp = 6.5 10-9)



y
( )
IO3- Ba)IO3(2

( KIO3
( NaF )
) 2y << 0.02
0.1 mole/L .
.)0.02 + 2y( 0.02 mol/L
Ksp = ]Ba2+[ ]IO3-[2 = )y( )0.02(2 = 1.5710-9
:1.18 10-3 M -
y = 3.9 10-6 mol/L
6.5 10-7 M -
Ba)IO3(2
KIO3 0.02 mole/L .7.3 10-4 M
Ba)IO3(2
( KIO3 )
:
143

= 187 7.3 10-4 mol/L
1 3.9 10-6 mol/L

Ba)IO3(2
KIO3 0.02 mole/L 187
.

.3
H+

Mg)OH(2
pH
.

(Mg)OH(2 )s (Mg2+)aq( + 2OH-)aq

( )H+
H+

OH- (
) . ( )OH-
.

21-3
()Ksp = 1.8 10-11
.pH= 10.5
:
() H+
pH =10.5 .
pH = - log ]H+[ = 10.5
]H+[ = 10-10.5 = 3.2 10-11 mol/L

144
Kw
1.0 10-14
= ]OH [ = +
-
= 3.1 10-4 mol/L
[ ]H 3.2 10-11


3.1 10-4 mole/L
Mg)OH(2 s
Mg)OH(2 .10.5

(Mg)OH(2 )s (Mg2+)aq( + 2OH-)aq

sM sM 3.1 10-4 M

Ksp
20-3 :
Ksp = ]Mg2+[ ]OH-[2
]Mg2+[ ]OH- [2 = )s( )3.1 10-4(2 = 1.8 10-11
() pH= 6() pH = 9.0
1.8 10-11
= s = 1.9 10-4 mol/L
( )3.2 10
-4 2

Ksp )Zn)OH(2( = 1.2 10-17


1.9 10-4 M Mg)OH(2 .

:0.12 M - .pH= 10.5

1.2 10-7 M -

145

pKa = - log Ka pKa

120 pH

]pH = - log [H+


120 pOH
]pOH = - log [OH-
121 pH pOH
pH + pOH = 14
118 Kw
[H+][OH-] = Kw = 1.0 10-14
126
Kw
= Kh
Ka
128
Kw
= Kh
Kb
126 pH
] [pKw +pKa+ log cpH = 1
2
128 pH
] [pKw pKb log cpH = 1
2
134 pH

][acid ][salt
pH = pKa- log = pKa + log
][salt ][acid

pOH 134
][base ][salt
pOH = pKb- log = pKb + log
][salt ][base

146

Electrolyte

.

Weak Electrolyte
.

Weak Acid
.

Weak Base
.



. .



( ).



H3O+ OH-

pH

1 M
.pH

Solvolysis
.

147
Hydrolysis
.

Buffer Solution
( )
( )
( )pH .

Solubility s
(
).

Solubility Product Ksp


( )
.

148

: 1-3
(2H2O)l (H3O+)aq( + OH-)aq
- .
- 25C

PH : 2-3
1mL .1 HCl .10M
1mL .2 NaOH . 10M
pH = 5 )2 pH = -5 )1 :

3-3
() ( NH4Cl) ( Na2SO4) ( CH3COOK) ( CaF2) ( MgSO4) .KCl
1 10-14 :

4-3 )M = 60g/mol( CH3COOH 250 ml


pH 2.7 pka = 4.74
3.3g :

: 5-3
.1 ( M= 487 g/mole )Ksp = 1.57 10-9
150 mL :
() 34.4 mg
() 44.4 mg
() .53.4 mg
.2 Na+ SO42- 0.4 M:
() ]SO42-[ = 0.4 M ]Na+[= 0.4 M
() ]SO42-[ = 0.4 M ]Na+[= 0.2 M
() ]SO42-[ = 0.4 M .]Na+[= 0.8 M
149
.3 pH pOH 0.05 M :
() pH = 1.3 pOH = 12.7
() pH = 7.0 pOH = 7.0
() pH = 12.7 .pOH = 1.3
.4 pOH 0.5 M:
() .4.78 () 9.22 () 7.00

6-3 PH 10ml HCl 0.1M:


10mL .1 NaOH .0.1M
15mL .2 NaOH .0.1M
10mL .3 NH3 .0.1M
15mL .4 NH3 .0.1M
pH = 12.3 )2 pH = 7 )1 :
1.8 10-5
pH = 8.96 )4 pH = 5.28 )3
: 7-3

pOH pH []OH- []H3O+


0.15 M HI
0.15 M HF
0.06 M RbOH
0.05 M Ca)OH(2
0.75 M NH4OH
1.8 10-5 6.5 10-4

: 8-3

[]H3O+ pH []OH- pOH


3.84 1
12.61 2
2.90 3
9.47 4

150
: 9-3

pKsp Ksp
6.0 10-50 ()mole/L Ag2S
4.06 ()g/L MgC2O4
(1.35 10-19 )mole/L Sb2S3
0.02 ()g/L ZnSO4

10-3 Ka C2H5COOH 1.3 10-5


% 0.45 : 0.65 M

11-3 [ ]NH3 ]NH4+[ = 0.01 M


6.67 10-3 M : ]OH-[ = 1.2 10-5 M 1.8 10-5

12-3 ( )M= 82 g/mole


0.125 M pH ( .4.74 :
10.25 g : ) . 1.8 10-5

( 13-3) (Ka)HNO2( = 4.5 10-4 )HNO2


0.12 M NaNO2 0.15 M () pH
3.62 ; 3.45 : 1.0 g ( )M = 40 g/mole .

14-3 pH = 9.0 NH3 .NH4Cl


1.82 : pK =4.74
[]NH4+

b
[]NH3

15-3 BaSO2 1.6 10-10 = ksp 1ml


10M . 1.26 10-5 mol/L , 1.6 10-8 mol/L :
H2SO4

16-3 pH [ ]OH- :
( 0.1 M )1 .NaCN
( 0.25 M )2 .NH4NO3
( 0.5 M )3 .NaNO3
Ka)HCN( = 4.9 10-10 1.8 10-5
151
17-3 0.01 M .%4.2 .
1.76 10-5 :
18-3 ( )M= 53.5 g/mole 500 mL
0.15 M pH .9.0 1.8 10-5
7.22g :
19-3 ( )mole/L ( )g/L Ag2SO4
( M = 314 g/mole )pKsp = 4.92 () ( ) 0.15 M .K2SO4

: 4.396 g/L ; 0.014 M -1.38 g/L ; 4.4 10-3 M -


20-3 )M = 332 g/mole( Ag2CrO4
2.161 10-3 g : 100 mL .Ksp = 1.1 10-12

21-3 BaCrO4 BaCl2


( ) 0.1 M .Ksp)BaCrO4( = 1.2 10-10
1.2 10-9 M :
22-3 pH 20 mL 0.2 M NaOH
50 mL 0.1 M CH3COOH .Ka)CH3COOH( = 1.8 10-5
5.34 :
23-3 25 mL 0.2 M NaOH 50 mL0.1 M

8.78 : .Ka)CH3COOH( = 1.8 10-5

24-3 pH 26 mL 0.2 M 50 mL
11.42 : 0.1 M .

25-3 ( )M = 56 g/mole 200 mL


0.0112 g : pH 11

26-3 0.1 M HCN % 0.01


1 10-9 : .
27-3 ( s
) ( )g/L .)M=99.4 g/mol( Zn)OH(2
.K ()Zn)OH ( = 1.2 10 -17

1.42 10-4 g/L ;1.43 10 M :
-6 sp 2

28-3 1mL 13.6 M .


1.866 :
29-3 ( )M = 40 g/mole 0.1 g/L
110-7 M )pKsp = 8.64( CaC2O4
: 152

Redox Reactions and Electrochemistry
4

:

.

.
:

.

.
.

.

.
E cell Ecell
G . Keq
153
1-4



.

. .


.
()

.


.

2-4 Oxidation Number




.
( ) .
:
-1 ( ) .
Na, Be, K, Pb, H2, O2, P4 = 0
-2 .
Li+, Li = +1, Fe3+, Fe = +3, O2-, O = -2
-3 ( )+1 ()-1
H2O ( )+1
NaH (.)-1
-4 ( )-2 ()-1
H2O ( )-2
H2O2 (.)-1
154
-5
- ( )IA ()+1
Cr+2
Cr+3 Li, Na, K, Rb, = +1
- ( )IIA ()+2
Be, Mg, Ca, Sr = +2
- ( )IIIA ()+3
B, Al, Ga, In = +3
-6 ( )VIIA ( )-1
F, Cl, Br, I = -1

:
Cr+6
:
Cr+4 .
Na = +1 Cl = -1 :NaCl )+1(+) -1 ( = 0
:
.
H = +1 P = P O = -2 H2PO4- :
{2)+1(} +)P(+{4)-2(} = -1
1-4 P = +5 :

] ( .[)1-4 1-4
. .
1-4
:
F2 BaOPO43 Mg2+ KCl SO2
:
F = 0 ()1 :F2
1-4
:BaO Ba O =

Ba = +2 -2 Ba )Ba( + )-2(= 0

:PO43 P O =
:
-2 P )P( + {4)-2(}= -3 P = +5
NaIO3 K2Cr2O7 MnO41- Mg = +2 :Mg2+ ()2
HPO42- H2CO3 H2SO4 Cl = -1 :KCl K = +1 ( 5 )6
:SO2 S O =
-2 S )S( + {2)-2(}= 0 S = +4
155
1-4

3-4 Redox Reactions


.

]
(.[)2-4
.


:
( :)Oxidation
.


:
(2Na)s( + Cl2)g (2NaCl)s
156

.
:
Na0 Na+ + e-
( )0
( )+1
(.)+1
( :)Reduction

()

( )-1 :
Cl20 + 2e- 2Cl-1

()
( )-1
.
2-4 .
) .

(CH4)g(+2O2)g
CO2)g(+2H2O)l(+Energy

) ( )
.
2Na + Cl2 2Na+ + 2Cl-
(4Fe)s(+3O2)g

2Fe2O3)s(+Energy
. .
.
.

.
.
:
P+5Cl5-1 + 4H2+1O-2 H3+1P+5O4-2 + 5H+1Cl-1

.
:
Cu0 + 4H+1N+5O3-2 Cu+2)N+5O3-2(2 +2N+4O2-2 +2H2+1O-2
157

. ( )0
( )+2
( )+5 ( )+4
.

.

: ()
() :
2Na0+Cl20 2Na+ + 2Cl-
:
Na0 Na+ + e- ()
Cl20 + 2e- 2Cl- ()
!
2 :
2Na0 2Na+ + 2e- (
Cl20 + 2e- 2Cl- )

(
:
).
2Na0+Cl20 2Na+ + 2Cl-
( )+1 .

( )-1 .
.
2-4

.
Cl2 + 2I- 2Cl- + I2 ()1
Mg + Fe2+ Mg2+ + Fe ()2
:
Cl2 + 2I- 2Cl- + I2 ()1

2I- I20 + 2e- ()


Cl20 + 2e- 2Cl- ()
158

:
2-4 Cl20 + 2I- 2Cl- + I20
( -1 ) .

( )-1 .
.
.
Mg + Fe2+ Mg2+ + Fe ()2
Zn+Cu 2+
Zn +Cu
2+
()1
Mg+2HCl (MgCl2+H2 )2 Mg0 Mg2+ + 2e- ()
Fe2+ + 2e- Fe0 ()


:
Mg + Fe2+ Mg2+ + Fe

( )+2
.
( 2+ ) .
.

4-4
1-4-4 Reducing Agent
.

.


Cu+2)NO3(2 + 2N O2 + 2H2O
+5 +4
Cu0 + 4HN O3

0 . +2

2-4-4 Oxidizing Agent


.



+5 +4
. :


159
3-4
:
(Zn)s(+CuSO4)aq (ZnSO4)aq( + Cu)s ()1
(Cu)s(+2AgNO3)aq Cu)NO ( +2Ag (3 2)aq ()s
()2
:
(Zn)s( +CuSO4)aq (ZnSO4)aq(+Cu)s ()1
:Zn Zn Zn2+ + 2e-
3-4
:Cu2+ Cu2++ 2e- Cu

:
(Cu)s( + 2AgNO3)aq (Cu)NO3(2)aq( + 2Ag)s ()2( 2C)s(+O2)g( 2CO)g ()1
:Cu Cu Cu2++ 2e- (Mg)s(+Cl2)g (MgCl2)s( )2
(Mg)s(+Fe2+)aq
:Ag . +
Ag + e
+ -
Ag Mg2+
()aq (+ Fe)s( )3

5-4Electrochemical Cells


.
(Mg)s( +2HCl)aq (MgCl2)s( +H2)g


.
? ?

( )Anode

( )Cathode

( ) .
.
160
:
( ).
6-4

.

( )Luigi Galvani
( )Alessandro Volata .
() ( )Battries .

: Zn
:
( )


] ( [)3-4 :

Zn)s( + Cu+2 SO4-2


()aq
(Zn+2 SO4-2)aq( + Cu)s
SO4-2
(
) :
(Zn)s( + Cu)aq2+
(Zn)aq
2+
(+ Cu)s


()II

3-4


.

161
()Danil Cell
( .)4-4
Zn ZnSO4 Cu
.CuSO4 Zn
Zn2+ Cu2+ Cu
.
( .)Electrodes

. ( )4-4 Zn Cu
ZnSO4 CuSO4 .

4-4

( )
( )Anode
. ()Cathode

( )
Cu2+
.
:
)Zn(s )Zn(aq
2+
+ 2e / ( Zn) ()
)Cu(aq
2+ + 2e )Cu(s / ( Cu) ()
Zn ( )
Zn2+ Cu2+
( ) .Cu

162
)
(
:
(Zn)s( + Cu)aq
2+
(Zn)aq
2+
(+ Cu)s

( )Cu2+

.

.
( )Salt bridge U

(()Ager

) .
KCl KNO3 .K2SO4


( )
Zn2+
. SO42-
(
) K+
(
) .
( )
( ) .

4-4

:
(Cu)s( +2Ag)aq+
(Cu)aq
2+
(+ 2Ag)s
- .
163
-
.KNO3

+ -

:
-
(Cu)s (Cu)aq
2+
+ 2e- /
(2 Ag)aq
+
+ 2e- (2Ag)s /

- ( )
( ) . K+
( )NO3-
.

7-4 Galvanic Cell Potential



.

.
:
-1 (:)Oxidation potential
.
-2 (:)Reduction potential
.
()Cell potential
( .)E
( )emf
( )Electromotive force
.
164
(] )Voltmeter ( .[)5-4

.

. cell( Ecell ) .
.
() () . 5-4
() ox( Eox oxidiotion )
(
red( Ered reduction ) . )

:
Ecell = Eox + Ered ()1

Eox Eanode
Ered .Ecathode ( )1
:
Ecell = Eanode + Ecathode ()2
25oC 1 atm

( 1 mol/L )1 M
o
( )Standaed cell potential E cell

o
Eocathode . E anode ( )2
:
o o
E cell = E anode + E cathode ()3
o

( )Volt (.)V

1-7-4
o
E cell
.
?
( )Reference electrode
.
.
165
2-7-4
(
) .
. Pt

1 atm 25 C H+
o

HCl ( .)1 M


.


: Pt

- .
6-4
- .
( )6-4 .
( )SHE
(.)Standard Hydrogen Electrode
( )IUPAC
(.)E H2 = 0.0 V
o


() :
+
(H2 )g 2H)aq( +2e- E anode = 0.0 V
o

() :
+
2H)aq( +2e- (H2)g E cathode = 0.0 V
o

3-7-4



. ( )SHE
.

.
( )2-4
.
166
) V(
2-4
.1 M C
o

F2 (g) + 2e- 2F(aq)


-
+2.87
H2 O2(aq) + 2 H+(aq) + 2e- 2H2O +1.77
Au (aq) + 3e-
3+ Au (s) +1.50
Cl2 (g) + 2e- 2Cl-(aq) + 1.36
O2(g) + 4H+(aq) + 4e- 2H2O + 1.23
Br2(l) + 2e- 2Br (aq)
-
+ 1.07
2Hg2+(aq) + 2e- Hg2+
2(aq) + 0.92
Hg 2(aq) + 2e-
2+
2Hg(l) + 0.85
Ag+(aq) + e- Ag (s) + 0.80
Fe3+(aq) + e- Fe(aq)
2+ + 0.77
I2 (s) + 2e- 2l-(aq) +0.53
O2(g) + 2H2O + 4e- 4OH-(aq) + 0.40
2+ + 2e- + 0.34
Cu(aq) Cu(s)
Sn4+(aq) + 2e- Sn2+(aq) + 0.13


2H(aq)+ + 2e- H2(g) 0.00
Pb(aq)
2+ + 2e- Pb(s) - 0.13
Sn2+(aq) + 2e- Sn (s) - 0.14
Ni2+(aq) + 2e- Ni (s) - 0.25
Co2+ (aq)
+ 2e- Co (s) - 0.28
Cd2+ (aq)
+ 2e- Cd (s) - 0.40
Fe (aq) + 2e
2+ -
Fe (s) - 0.44
Cr3+(aq) + 3e- Cr(s) - 0.74
Zn2+ (aq)
+ 2e- Zn(s) - 0.76
2H2O + 2e -
H2 (g) + 2OH-(aq) - 0.83
Mn(aq)2+ + 2e- Mn(s) - 1.18
Al3+(aq) + 3e- Al (s) - 1.66
Be2+(aq) + 2e- Be(s) - 1.85
Mg(aq)2+ + 2e- Mg(s) - 2.37
Na+(aq) +e- Na (s) - 2.71
Ca2+ (aq)
+ 2e- Ca (s) - 2.87
Ba2+(aq) + 2e- Ba (s) - 2.90
K+(aq) + e- K(s) - 2.93
Li(aq)
+ + e- Li(s) - 3.05

167



+ -
Eocell :
o o
E cell = E anode + E cathode
o

Eo +0.763 V ()SHE
cell
.
o o
E cell = E anode + E cathode
o

= 0.763 V E anode +0.0 V


o

() ()
E anode = +0.763 V
o

+0.763 V
-0.763 V 4-4

(.)SHE
.

.
. .+0.337 V
-1 2-4 .
-2 + -

+2.87 V .
-
F2)g( + 2e- (2F)aq E cathode = +2.87 V
o

-3.04 V .
+
Li ()aq +e -
(Li)s E cathode = -3.04 V
o
() ()

Li+ .
F +0.337 V :

.
-3
( )SHE
.
-4 ()
( ) Eoanode

168
() (
) Eocathode .
-3.04 V
.+3.04 V
+
Li (aq) + e- )Li(s E cathode = -3.04 V
o

+
)Li(s Li (aq) + e- E anode= +3.04 V
o

4-7-4
:
-1 .
.
-2 (
) (
) .

!
.
-3
. .
. 26


. :
-
I2(s) + 2e- )2 I (aq E
o
I2/I-
= 0.53 V
-
2I2(s) + 4e- )4 I (aq E
o
I2/I-
= 0.53 V
-4
.
-5 Eocell

:Eocell
o o
E cell = E anode + E cathode
o


:
E cell = E cathode - E anode
o o o

169
.
-6 ( )Eocell = +
. Eocell ( )Eocell = -
.
5-4

.
E Cu2+/Cu = +0.34 V . EoZn2+/Zn = - 0.76 V
o

(: EoCu2+/Cu).
: + -


} (.{)1
(Zn)s (Zn)aq
2+
+ 2e- E anode = +0.76 V
o

o
(Cu)aq
2+ Ecathode
+ 2e -
(Cu)s = +0.34 V

} ( .{)4
5-4


.

(Zn)s( + Cu)aq (Zn)aq
2+ 2+
(+ Cu)s
Cd 1 M
E cell = E anode + E cathode
o o o
Cr

E cell = )+0.76 V( + )+0.34 V( = 1.10 V 1 M .


o


: : o
E Cd2+/Cd = -0.40 V
E cell = E cathode - E anode
o o o

.
E Cr3+/Cr = - 0.74 V
o


+0.34 V :
:
E cell = 0.34 V - )- 0.76 V( = 1.10 V
o

5-7-4 Types of Electrodes


.

170

.
- 1 /

6-4
( )Zn/Zn2+


:
)Zn(aq
2+
+ 2e- )Zn(s ()
)Zn(s )Zn(aq
2+
+ 2e- ()
- 2
o
E Co2+/Co =- 0.28 V
. E Ni2+/Ni = - 0.25 V
o

E Fe2+/Fe = - 0.44 V
o

. Eo =+1.50 V . 1 atm
3+
Au /Au
.HCl
Ni(s) + Co2+ :
)(aq
+
)Ni2+(aq )+ Co(s )H2(g 2H(aq) + 2e- ()
HCl
)3Fe(s) + 2 Au (aq .NaCl
3+

3Fe2+ )+ 2Au(s :
)(aq
Cl2(g) + 2e- )2Cl(aq
-
()
: ; .
-3

.
Fe2+ ( Fe3+
Sn2+ .)Sn4+
:
)Fe(aq
2+
)Fe(aq
3+
+ e-

6-7-4


. /
:
171
)Zn(aq
2+
+ 2e- )Zn(s ()


:
Zn2+ (1 M)| Zn
:
)Zn(s) Zn(aq
2+
+ 2e- ()


:
)Zn | Zn2+ (1M
()s
( )aq
.
( )11M .

.
:
+
)H2(g 2H(aq) + 2e- ()


( )
.
)Pt | H2 (1 atm) | H+ (1 M


.
Fe2+ Fe3+ :
)Fe(aq
2+
)Fe(aq
3+
+ e- ()
)Pt | Fe2+ (1 M) ; Fe3+ (1 M

Pt
. (;)
.
172

() () .


.
:
(Zn)s( + Cu)aq
2+
(Zn)aq
2+
(+ Cu)s
:
||
Zn | Zn2+ )1 M( || Cu2+ )1 M( | Cu
.

6-4


.

:
(Zn)s (Zn)aq
2+
+ 2e- :
+
2H)aq( +2e- (H2)g
Pt
:

:
+
(Zn)s( + 2H)aq (Zn)aq
2+
(+ H2)g
:
Zn | Zn2+ )1 M( || H+ )1 M( | H2 )1 atm( | Pt

7-7-4 E cell oG Keq


o


. } ( {)J
Eocell ( )V
! ( )Columb( )C .
( = )J ( )V ()C
( )C ( )V
(.)J ( )V
( )C :
1 J = 1V 1C
173

( )n
( )Faraday ( .)F
( .)nF
( )NA ( )F
.
= F ( )e- ()NA

NA = 6.0231023
e- =1.610-19 C : 7-4
F = 6.023 1023 1.6 10-19 (C) = 96478 C/mol.e-
:
)Cl2(g) + 2Ag(s
96500 C/mol.e- .
: )2 Cl-(aq) + 2Ag+(aq


( = )J ( )nF ()E cell
o


= nFE cell
o

.
( )nFEocell
oG:

oG = - nFEocell )(1 8-4


HCl
o
. ( )1 G
Ag ( )1M E cell . E cell
o o +

.
o
)1( . E Ag+/Ag =+0.80 V

( :
Eocell oG :
).

G = - nFE cell
o o
)(2 )Ag | Ag+ (1 M) || H+ (1 M
| H (1 atm) | Pt
2
oG
: .
Keq :
oG = - RT ln Keq )(3
174
oG ( )3 ( )2 :
- RT ln Keq = - nFEocell ()4

( )4 :
RT ln K
= Eocell ()5
eq
nF

! ( )5 )298 K(25oC

R= 8.314J/K.mole F= 96500C/mol.e- :

(8.314 )J/K . mol( 298 )K


lnKeq = E cell
o

. )298 K(25oC
( n 96500 )C/mol.e-

(0.026 )V
3-4
lnKeq = Eocell
G
o
n
Keq E cell
o

n

.
.)298 K(25 C ( )3 - 4
o

G E cell Keq .
o o


o
Keq G
o 7 - 4
E cell

:
(2Ag)aq
+
(+ Pb)s (2Ag )s(+ Pb)aq 2+

Keq > 1

G Keq 25 C
o o



Keq = 1 EoPb2+/Pb = -0.13 V . EoAg+/Ag=+0.80 V

:
() ()
Keq < 1

(Pb)s (Pb)aq
2+
+ 2e- E Pb2+/Pb =+0.13 V
o

(2Ag)aq
+
+ 2e- 2Ag
E Ag+/Ag =+0.80 V
o


(2Ag)aq
+
(+ Pb)s (2Ag)s( +Pb)aq2+

175
.Eocell
E cell = E anode + E cathode
o o o

E cell =)+0.13 V( +)+0.80 V( = + 0.93 V 9 - 4


o

Go
E cell
o

o
(oG = - nFE cell = - 2 96500 )C/mol.e-( 0.93 )V
o
.25 C
= - 181420 J/mol (3Hg22+)aq( + 2Cr)s
Keq : 6Hg (+ 2Cr)aq3+
RT ()l
ln Keq = E cell
o

nF
E Hg22+/Hg =+0.85 V
o

)298 K(25 C n = 2
o

. EoCr3+/Cr=-0.74V
2 :
-920610 J/mol :
(0.026 )V
lnKeq = + 0.93 V
2
Keq
Keq = 2.5 10 31

8-7-4 ) ( 10 - 4


.25oC
( .)1 M
2Fe3+ (+ 2I )aq
-
( .)1 M ()aq
2Fe2+ (+I2)s
Ecell . ()aq


oG o G : EoI2/I- =+0.53 V
E Fe3+/Fe2+=+0.77 V:
o
aA + bB gG + hH
G G
o
- .
: - .
][G] [H
g h

G = G + RT ln
o
()1 - .
][A] [B a b

:+0.24 V -
oG R
-46320 J/mol -
( )R = 8.314 J/K.mol T ( )K
G . 1 108 -
[G]g [H]h
( Q ( )Q Quotient
b
a
][A] [B
).
176
( )1 :
G = oG+ RT ln Q )(2
:
G = - nFEcell )(3

o
oG = - nFE cell )(4
G G ( )3 ( )4 ( )2 :
o

o
)- nFEcell = - nFE cell + RT ln Q (5
( )5 - nF :
RT
ln Q Ecell = E cell -
o
)(6
nF

( )6 ( )Nernest equation
o
Ecell E cell .Q
( )1 M
Q ln Q
o
( )6 . Ecell =E cell

( )6
(298 K)25oC
R = 8.314 J/K . mol F = 96500 C/ mol.e- :
o
0.026 V
ln Q Ecell = E cell - )(7
n

.(298 K) 25oC
Ecell
. :
)Zn(s) + Cu(aq 2+
)Zn(aq
2+ )+ Cu(s

1.10 V
25oC :
0.026 V ][Zn2+
ln Ecell = E cell -
o

n ] [Cu 2+


[Cu] = 1 [Zn] = 1 .
177
8 - 4
o
Ecell 25 C : 11 - 4
2Ag+ (+ Cu)s (2Ag)s( + Cu)aq
2+
()aq

[Cu2+] = 0.01 M .[Ag+] = 0.01 M Ecell 0.73 V
o
E Cu2+/Cu=+0.34 V E Ag+/Ag=+0.80 V
o
.25oC
: [Zn2+] = 0.1 M
() () . 1 atm
o
(Cu)s (Cu)aq
2+ + 2e- Eanode = - 0.34 V
o .H+
2Ag+ + 2e- (2Ag )s Ecathode = + 0.80 V o
()aq .E = -0.76 V
Zn2+/Zn

(2Ag)aq
+
(+ Cu)s (2Ag)s( + Cu)aq
2+ .
+ -
o
. E cell
o
E cell = Eanode + Ecathode
o o

E cell = )- 0.34 V( + )+0.80 V( = 0.46 V


o

Ecell () ()
0.1 mol/L :
0.026 V ] [Cu 2+
ln Ecell = E cell -
o

n ] [Ag + 2

0.026 V ][0.01
2 ln Ecell = 0.46 V - 12 - 4
2 ][0.01 o
Ecell Ecell G
Ecell
Ecell = 0.40 V :
Mg | Mg2+ )0.05 M( || Sn2+
)0.04 M( | Sn
8 - 4 Electrolyte Cells


o
E Sn2+/Sn= -0.14 V
. o
.E 2+ =-2.37 V
( .)G = + Mg /Mg

. +2.23 V ; +2.227 V :

; -429811 J/mol

. :
178
1 - 8 -4

.
Na+
Cl- . ( )7 - 4

.
Na+
: Na
Na++e- Na

NaCl
Na


Na
NaCl

7-4 Cl-



.Cl2
:
2Cl- Cl2 + 2e-
() 2

:
2Na+ + 2e- 2Na ()
2Cl- Cl2 + 2e- ()

2Na+ + 2Cl- 2Na + Cl2




NaCl .

179
2 - 8 - 4

( )AgNO3 ] [Au)NO3(3

. ( )8 - 4
8-4
.

.

Faraday Laws 9 - 4
( )Faraday
.
:
:

.
:


.

.
:
- .
-
H2.
:
- ( )
. !
-
Cl2 .O2 Q
.
:
-1 ( )I ( )Ampere
(.)A
180
-2 ( ) ( )Coulomb
.C ()
( F )F = 96500 C/mol.e- .
( I (A ( t (s :
) (C) = I (A) t (s
.96500 C/mol.e-

) (C) = I(A) t(s


)I (A) t (s
= ) (mol . e-
) 96500 (C/mol.e-
( )mol . e- .
= )I (A) t (s

96500
.
( )1 mol . e-
n = 1
mole
96500 C


.
( ) mol . e-
.
V(L) 1 mole
=n ( )1-6:
22.4 L -1
( )mol . e- :
)I(A) t(s
= n = ) (mol . e-
) 96500 (C/mol.e-
- 2
( Q )mol . e-

= n .
-3
:
n = 1
mole

=

- 4 ( )n mole M g/mol
1-4
:

. )m (g) = n (mol) M (g/mol
:
PV = n RT
181
9 - 4
(Mg)aq
2+ + 2e- (Mg )s :

25 A ( .)1 hr
.24 .
:
:
(3600 )s
(t )s( = t )hr
(1 )hr
3600 )s( = 3600 s
(= 1 )hr 13 - 4
(1 )hr

.
AuCl3
(I )A( t )s
= ( )mol.e- 200 s 3 g
(96500 )C/mol.e-
.197
(25 )A( 3600 )s
= = 0.9 mol.e- 21.7 A :
( 96500 )C/mol.e -

2 mol.e- 1 mole
14 - 4
:

n = 1
mole
CuSO4 0.2 M

.600 mL

(1 )mole .96.5 A
nMg )mole( = 0.9 mol.e
- = 0.45 mole
(2 )mol.e- 0.03 mol .

: 180 s :
(m )g
= (n )mole
) (
M g/mol
m
(m )g( = n )mole( M )g/mol
m )g( = 0.45 )mole( 24 )g/mol( = 10.8 g
=
= (NA )atoms / mol( n )mol
= (6.0231023)atoms/mol(0.45)mol
= 2.71023 atoms

182
10 - 4 )( Battries and Fuel Cells
:
.

.
.

1 - 10 - 4 Lead - Storage Battrey



.
( .)2 V



1.2 g/mL 1.3 g/mL

6 V
12 V ] (. [)9 - 6




(Pb)s( + SO4)aq
2-
PbSO4)s( + 2e-
9-4

PbO2 :

(PbO2)s( +4 H )aq
+
(+SO4)aq
2-
+ 2e- (PbSO4)aq( + 2H2O)l

:
(Pb)s( + PbO2)s( +2H2SO4)l (2PbSO4)s( + 2H2O)l



.

H2SO4 PbO2 . Pb

PbSO4 ().

183
Dry Cell 2 - 10 - 4 ) (
MnO2

() NH4Cl

( .)10 - 6 : ZnCl2 NH4Cl

(Zn)s(+ 2 OH)aq -
Zn)OH(2)s( + 2e- MnO2

MnO2 :

2MnO2)s( + 2H2O)l( + 2e- (2MnO)OH()s( +2 OH)aq


- 10-4
( - )
:
(Zn)s( + 2MnO2)s( + 2H2O)l (Zn)OH(2)s(+ 2MnO)OH()s

: 1.481V
.


o
E cell = E anode + E cathode
o o
165

G = - nFEcell 169

Go = - nF Eocell 173

RT
ln Keq = E cell 174
o

nF

(0.026 )V
o
E cell = lnKeq 174
n
25oC
RT 176
ln Q Ecell = Eocell -
nF

0.026 V
ln Q 176 Ecell = Eocell -
n 25oC

184

Oxidation
.
Reduction
.
Reduction Agent
.
Oxidizing Agent
.
Anode
.
Cathode
.
Electrochemical cells


.
Galvanic cells

.
Electrolyte cells

.
Electromotive force
( )emf ( )Ecell
.
Nernest equation
( )Ecell ( )Eocell
.
185

: ( ) ln x = 2.303 log x
1-4 : .

. 2-4

3-4 .
Eo Eo 3+ =+1.50 V Eo + =+0.80 V =+1.07 V
Ag /Ag Au /Au Br2/Br-
EoCd2+/Cd= -0.40 V .Eo 3+ = -0.74 V
Cr /Cr
Au3+ ) Br2
Ag+ ) H2
Cr3+ )Cd2+

4-4 Go Keq Eocell .

5-4 25oC 1 atm


Ce4+ Ce3+ Fe ? Fe Go Keq .
2+ 3+

Eo 4+ 3+=+1.61 V . Eo 3+ 2+=+0.77 V
Fe /Fe Ce /Ce
1.1 1014 ; -81060 J/mol :
6-4 .

7-4 Eocell Ecell G :


(Mg )s( + Sn2+)aq (Mg2+)aq (+ Sn )s

[Sn2+] = 0.04 M [Mg2+] = 0.05 M


(3Zn )s( +2Cr3+)aq (3Zn2+)aq( + 2Cr)s

[Zn2+] = 0.001 M [Cr3+] = 0.01 M


ESn EoZn2+/Zn=-0.76 V
o
2+/Sn=-0.14 V E Mg2+/Mg= -2.37 V
o
- 429811 J/mol ; + 2.227 V ; + 2.23 V : .Eo 3+ = -0.74 V
Cr /Cr
- 40530 J/mol ; + 0.07 V ; + 0.02 V
8-4 Eocell Zn/Zn2+ . SHE Ecell
. [H+] = 1.8 M , PH2 = 1 atm , [Zn2+] = 0.45 M
+0.79 V ; +0.76 V : . Eo 2+ =-0.76 V
Zn /Zn
186
( ) . 9-4

10-4 :
+
(2H2O)l (O2)g(+4H)aq + 4e-

0.08 L O2 25oC 7551mm.Hg


(.)1atm = 760 mm.Hg :
0.012 mol.e :

11-4 3.75 hr
AgNO3 .CuCl2 2 g .
) .
: , 0.64 g -0.14 A - ) .

12-4 .
Eo 3+ =-1.66 V Eo 3+ = +1.50 V?
Au /Au Al /Al
: .
13-4 ?
.Eo EoFe3+/Fe2+=+0.77 V =+1.07 V
Br2/Br-
Pt |Fe2+ )1 M( ; Fe3+ )1 M( || Br- )1 M(| Br2 )1 atm( | Pt
:.
14-4 H2 25oC 1atm pH = 1
-0.0592 V :

15-4 :
AgNO2 + Cl2 +2KOH AgNO3 +2KCl + H2O

16-4 ? EoCu2+/Cu=+0.34 V
:. .Eo + = -2.70 V
Na /Na

17-4 CuSO4 ?
? Eo 2+ = +0.34 V .Eo 2+ = -0.24 V
Ni /Ni : . Cu /Cu

187
18-4 ( )A 5 g
2.01 A : (? )+3

19-4 ? .

20-4 ?

21-4
( )-0.82 V .Eo 2+ =-0.76 V
0.01 mol/L : Zn /Zn

22-4 ( )0.1 M
+1.07 V : ( )0.01 M .25oC (?)1.101V

23-4 :
(1 Al2O3 + 6HCl 2AlCl3 + 3H2O
(2 2Na + Cl2 2NaCl
(3 2KClO3 2KCl +3 O2
(4 SiBr4 +3H2O H2SiO3 + 4HBr

(5 Zn + CuSO4 ZnSO4 +Cu

24-4 .25oC
(Cu)aq
2+ )0.01 M( + Cd (Cu)s( + Cd)aq
2+ ()0.1 M
()s

-137030 J/mol : ( .)+0.74 V .

25-4 :25oC
Mg | Mg2+ )1 M( || Br- )0.1 M( | Br2 )1 atm( | Pt
.Eo EoMg2+/Mg=-2.37 V =+1.07 V
Br2/Br-
675500 J/mol :

26-4 25oC Keq = 2.3 104


:
(2H)aq( )1 M( + Pb)s
+
(H2)g( + Pb)aq( )0.01 M()s
2+

-36515 J/mol :
188
27-4 25oC :
(Sn)aq
2+
(+ Ni)s Sn)s( + Ni2+
()aq
( .)+0.17 V Ni
2+

Eo 2 =-0.14 V . Eo 2+ = -0.25 V
Ni /Ni Sn +/Sn
0.01 mol/L :
28-4 :
Al | Al3+)1 M( || Cd2+ )1 M( | Cd
( )1.26 V .Eo 2+ = -0.40 V
Cd /Cd

-1.66 V : ?

29-4 25oC 1 atm


? 0.01 M ( )pH
-48.25 kJ/mol ? -0.251V
1 :
30-4 :
Sn | Sn2+)? M( || Ag+ )1 M( | Ag
25oC 0.9992 . ( )Sn2+
.Eo Eo 2 = -0.14 V =+0.80 V
Ag+/Ag Sn +/Sn
0.01 mol/L :

31-4 10 A 965 s
= ?63
0.3 1023 atoms ; 3.15 g :

32-4 0.648 g 3 A
3 min 13 s . ?
108 g :

33-4 ( STP :
STP ?)22.4 L
48.16 1023 e :

2 hr 520 s 34-4
36.12 1021 ?
1 A :

189
35-4 10 A
9.65 s % 75
0.05 g : ? =.197

36-4 0.2 mol.e- 0.448 L


.STP ? = . 63
5.04 g :

37-4 :
(Sn)s( + Pb)aq
2+
(Sn)aq
2+
(+ Pb)s
EoSn2+/Sn=-0.14 V EoPb2+/Pb= -0.13 V
2.16 :

38-4 STP 3 13
. 0.0672 L
.
2A:

39-4 0.0592 V
. .
0.01 M :

Eo
Ag1+/Ag
40-4 EoZn2+/Zn= -0.76 V = 0.8 V
:
-1
? 1M

-2
. 0.1M
-1 :
-289500J -2

190

Coordination Chemistry
5

:
.
.
.
.
.
.
. IUPAC

.
.

191
1-5


.

.
B12.



.
IIA
IIIA ] ( [)1-5
d f
. :
-1 d

.
-2 f

.
:
-1
.
-2
d f
.
-3 .
-4 .

192
1 18
IA VIIIA
1 2 13 14 15 16 17 2
H IIA IIIA IVA VA VIA VIIA He
3 4 5 6 7 8 9 10
Li Be B C N O F Ne
11 12 3 4 5 6 7 8 9 11 12 13 14 15 16 17 18
10
Na Mg IIIB IVB VB VIB VIIB VIIIB IB IIB Al Si P S Cl Ar
19 20 21 22 23 24 25 26 27 28 29 30 31 32 33 34 35 36
K Ca Sc Ti V Cr Mn Fe Co Ni Cu Zn Ga Ge As Se Br Kr
37 38 39 40 41 42 43 44 45 46 47 48 49 50 51 52 53 54
Rb Sr Y Zr Nb Mo Tc Ru Rh Pd Ag Cd In Sn Sb Te I Xe
55 56 57 72 73 74 75 76 77 78 79 80 81 82 83 84 85 86
Cs Ba La Hf Ta W Re Os Ir Pt Au Hg Tl Pb Bi Po At Rn
87 88 89 104 105 106 107 108 109 110 111 112
Fr Ra Ac Rf Db Sg Bh Hs Mt Uun Uuu Uub
58 59 60 61 62 63 64 65 66 67 68 69 70 71
Ce Pr Nd Pm Sm Eu Gd Tb Dy Ho Er Tm Yb Lu
1-5 90 91 92 93 94 95 96 97 98 99 100 101 102 103
Th Pa U Np Pu Am Cm Bk Cf Es Fm Md No Ir

2-5
(
( ))II

( .)Addition compound :
(NH4)2SO4 + FeSO4 + 6H2O FeSO4. (NH4)2SO4. 6H2O

CuSO4
:
CuSO4 + 4NH3 CuSO4. 4NH3

[Cu(NH3)4]SO4

:
- Double Salt

.
( )Mohr,s salt
Fe2+ NH4+ SO42-
.
193
- Coordination Compound

.
.
CuSO4.4NH3
( Cu2+ )SO42- SO42-
Cu2+ [Cu(NH3)4]2+
:

CuSO4. 4NH3 [Cu(NH3)4]2+ + SO42-

SO42-
Cu2+
.[Cu(NH ) ]SO
3 4 4
[Cu(NH ) ]2+ ( .)SO 2-
4 3 4

]K3[Fe(CN)6 3K+ + [Fe(CN)6]3-


[Co(NH3)6]Cl3 [Co(NH3)6]3+ + 3Cl-



[ ]Ni(CO)4
][Co(NH3)3Cl3

194
1-5
Fe)NH4(2)SO4(2
] K3[Fe)CN(6 ( )

:
Fe)NH4(2)SO4(2
( )II :
)NH ( SO + FeSO FeSO4.)NH4(2SO4
4 2 4 4


Fe2+ NH4+ .SO42-
.
] K3[Fe)CN(6 :
]K3[Fe)CN(6 [Fe)CN(6] 3- + 3K+

Fe3+
[Fe)CN(6] 3-
K+ Fe3+ .CN-
Fe)NH4(2)SO4(2 ]K3[Fe)CN(6
.

3-5
()III
CoCl3. 6NH3 ( )1798 .

. CoCl3

.

.
()Chain Theory

195

.
( )III
.
{ CoCl3. 6NH3( })I


.
CoCl3. 5NH3 ( )II

.

Cl NH3+Cl-

Co- NH3-NH3-NH3-NH3+Cl- Co- NH3-NH3-NH3-NH3+Cl-

NH3+Cl- NH3+Cl-

()II ()I

CoCl3. 4NH3 ( )III



.
CoCl3. 3NH3 (.)IV
Cl Cl

Co- NH3-NH3-NH3+Cl- Co- NH3-NH3-NH3-NH3+Cl-

Cl Cl
()IV ()III

( )IV
.CoCl3.4NH3
.
.
196
1-3-5 Werner,s Coordination Theory


:
-1
( )------ (Oxidation
)state ()

(. )Coordination number
-2


.
-3
.
( )Coordination sphere
.
.1913


CoCl3. 6NH3 ( )V
.]Co(NH3)6]Cl3
() ( )+3
( ) .

( )III
NH3
NH3 Cl (.)6

NH3
Cl Co
( .)Ligands NH3
NH3
NH3 Cl
()V .
.
197
CoCl3.5NH3 ( .)VI
Cl
() ---------- NH3 NH3


Cl Co Cl

. [Co(NH3)5Cl]2+ NH3 NH3
( )+2 Co3+ + Cl- = (+3) + (-1) = +2 : NH3
()VI
CoCl3. 5NH3 [Co(NH3)5Cl]Cl2
.

CoCl3. 4NH3 ()VII Cl


NH3
NH3

Cl Co

. NH3
NH3
]Co(NH3)4Cl2[+ Cl- Cl
()VII
.[Co(NH3)4Cl2]Cl

CoCl3.3NH3 Cl
)VIII( NH3 NH3
[ .]Co(NH3)3Cl3
Co

Cl . Cl
NH3

. ( )VIII

198
2-5
( ) ( )
:
K4[Fe)CN(6] -
[Cr)NH3(6])NO3(3 -
( : CN-
].) K4[Fe)CN(6

:
- +1
[Fe)CN(6]4- ( )4-:
Fe)x(+ )CN-(6 = -4
x + 6 * )-1( = -4
x = +2
+2 6
.

- ( )1-
[Cr)NH3(6]3+()3+:
Cr + )NH3 (6 = +3 1-5
()x 0


x + 0 * 6 = +3
] K3[Fe)CN(6
x = +3
.6 ; +3 :
+3 .6

2-3-5
( )CH4 ( )NH3
:
H H
H C H H N
H H

199



.
(
)
( )Coordination bond
) (
.
H H H
][ H N H ][H NH
+ + +
H N + H
H H H


:
Ag+ + 2 :NH3 [H3N:Ag:NH3]+

NH3
:

Cu2+ + 4 :NH3 [H3N :Cu : NH3]2+


:

NH3
NH3

H
3 N NH 3 3+
: : NH3

Co3+ + 6 :NH3 [ Co ]
: :
: :

N NH
H3 3




.
.

.
200


:

-1

.

( )Monodentate
( )Bidentate
(. )Multidentate

-2
()Acceptor

. .

-3 Coordinate Complex

.

-4 Coordination Number
( )

. 2 4 6
.

.
[Fe(CN)6]4- ( )+2
( )6
(( )-4
) )2+( (II ( )CN- (.)6-
201
-5 Complex Ion

.

. :

[Co(NH3)6]3+ [Ni(CN)4]4- [Fe(CN)6]4-


[Co(NH3)5Cl]2+

-6 Neutral Complex
.
:
][Co3+(NH3)3Cl3 ; ][Ni0(CO)4
][Pt2+(NH3)2 Cl2 ; ][Ni2+(dmg)2

-7 Coordination Sphere

[ ] .
( )Inner sphere
(
) ( )Ionisation sphere
( )Outer sphere .
[Co(NH3)5Cl]Cl2 Co3+

Cl- Cl- .



. :
202
[Co)NH3(5Cl]Cl2 [Co)NH3(5Cl]2+ + 2Cl-

2Ag+ + 2Cl- 2AgCl


: Cl-
( )
( )AgNO3 Cl- .
-8 Coordination Chemistry

.

4-5

-



Cl- F- Br- CN- ()NH3
RNH2 ( )C5H5N .H2O
.


-

(
) C2O42-
. NH2-CH2CH2-NH2

-

.
( .)EDTA
.
203

( . )Chelating ligand
( )1-5 .
1-5


NO CO
NH3 H2O
C5H5N CH3NH2
N3- CH3COO-
}{(NH2)2CO -CN
Br- Cl-

NH2NH2 NH2CH2CH2NH2 ()en

CO32- C2O42-

NO3-

5-5 (The Effective Atomic Number Rule )EAN


( )Sidgwick

.


( 36Kr 54Xe
.)86Rn

. .
204
3-5
[Co)NH3(6]3+
( )EAN .27

2-5
:

:

Co = 27 e-

Co3+ = 24 e-
; [Pd)NH3(6]4+ ; [Fe)CN(6]3-
6NH3 = 12 e-
[Ni)en(3]2+
----------------
; 35 : ; ; 54 ;
[Co)NH3(6]3+ = 36 e-
; 38 .

36
!
.


4-5
[CoCl4]2- ( )EAN
. .27

3-5 :
Co = 27 e-
[Ni)NH3(6]2+ [Ag)NH3(4]+ Co2+ = 25 e-
( )EAN 4Cl- = 8 e-
28 -----------------
.47 [CoCl4]2- = 33 e-
; 38 : ; ; 54. 33

.

205


[ ]M(CO)x :
r = 24 e-
C Fe = 26 e- Ni = 28 e-
CO = 12 e-6 5CO = 10 e- 4CO = 8 e-
-------------- ------------------ ---------------
[Cr(CO)6] = 36 [Fe(CO)5] = 36 [Ni(CO)4] = 36
. 4-5

[ ) ]Re (CO
2 10
36 54 86
( )EAN
.
Re . 75
( )Dimer ( )Polymers
; 86 : .
[ ]Mn2(CO)10 [ . ]Co2(CO)8
:
CO CO CO CO
OC CO
CO
OC
OC Mn Mn CO Co Co
CO
OC OC
CO
CO CO CO CO
n = 25 e-
M Co = 27 e-
Mn Mn = 1 e- Co Co =1 e-
CO = 10 e-5 4CO = 8 e-
------------------- -------------------
[Mn2(CO)10] = 36 [Co2(CO)8] = 36

6-5

( )IUPAC
:
-1
(
) .
206
NaCl ()Sodium chloride
[Cr(NH3)6]Cl3
()III
(.)Hexaammine chromium (III) chloride

-2

! :


[Cr(H2O)4Cl2]Cl

)(III

dichloro chromium(III) chlorideTetraaqua

.
-3 () (

( ) )o
.
( )aqua ( )ammine
m

m . ( ) ()
( .)ium ( )2 - 5
:

2-5

Cl- Chloro
CN- Cyano
CH3COO- Acetato
SCN- Thiocyanato
NH2CH2CH2NH2 Ethylenediamine
NH2.NH3+ Hydrazinium

207
-4 ( )di ( )tri ( ......)tetra


( )bis ( )tris
( )en
( )EDTA :

Co(en)2Cl2]2SO4[
( ) ()III
Dichloro bis (ethylenedi amine) cobalt (III) sulphate

-5
. 5-5
( .)0 : :
[]Ni(CO)4
[Co(NH3)5H2O]Cl3
()0
Na[Co(NH3)4 Cl2]
)Tetracarbonyl nickel(0
K2[PtCl6]

-6 [Cr(NH3)6](NO3)3 () ( )ate
( [Cu(en)2] Cl2
..) . ](NH4)2[Cr(SCN)6
. :

] a2[Fe(CN)6
C
( )II
) alcium hexacyano ferrate(II
C

]Fe (H2O)6]SO4
()II
exa aqua iron (II) sulphate
H

[]Ni (dmg)2
( ) ()II
) is(dimethyl glyoximato) nickel(II
B

208
7-5



:

(Valence Bond Theory (VBT -1


(Crystal Field Theory (CFT -2
-3
(Molecular Orbital Theory (MOT

1 - 7 - 5


.
() ()
.
( )
.
2 3 4
.

( [Ag(NH3)2]+ )I

:

209
Ag [Kr]36
47
4d10 5s1 5p0







Ag+ [Kr]36 4d10 5s0 5p0








[Ag(NH3)2]+ [Kr]36

4d10 5s 5p 6-5
: : VBT










NH3 NH3 [Cu(CN)2]-

-
-spsp.
- (.)Linear
- .

( ]HgI3 ]- (II

:
80
Hg [Xe]54 4F14 5d10 6s2 6p0




Hg2+ [Xe]54 4F14 5d10 6s0 6p0











[HgI3]- [Xe]54 4F14 5d10 6s 6p


: : :









I I I

210
- .
- sp2 s p .
(.)Trigonal planar -
- .

( ]Ni(Cl)4]2- (II
:

28
Ni [Ar]18 3d8 4s2 4p0


Ni2+ [Ar]18
3d8 4s0 4p0







[Ni(Cl)4]2- [Ar]18
3d8 4s 4p
: : : :







Cl Cl Cl Cl

- .
- sp3 s p .
- (. )Tetrahedral
- .

([Ni(CN)4]2- )II

:

28
Ni [Ar]18 3d8 4s2 4p0




211
[Ni2+] [Ar]18 3d8 4s0 4p0









[Ni(CN)4]2- [Ar]18
3d8 4s 4p
: : : :







CN CN CN CN

- .
- dsp2 d s
p .
- (.)Square planer
- .

d
[Ni(Cl)4]2- [Ni(CN)4]2-
CN-
Cl-
.

. ( )3 - 5 :

3-5


CN- I-
NH3 Br-
NH2CH2CH2NH2 Cl-
NO2- F-
CO OH-
C5H5N H2O

212

( ) dsp2
.

[PtCl4]2-
:

78
Pt [Xe]54 4F14 5d8 6s2 6p0


Pt2+ [Xe]54 4F14 5d8 6s0 6p0







[PtCl4]2- [Xe]54 4F14


5d8 6s 6p
: : : :







Cl Cl Cl Cl

- Cl-
- dsp2 d s
p .
-
- .

7-5

] ]NiCl4
2-

. ( ) ] ]PtCl4
2-

. :

(B.M) = [e(e+2)]1/2

= e B.M
( .)Bohr Magneton
. 5-5
213
5-5
( )II [Co)L(4]2+ L
. :
:
:
-1

:
27
Co [Ar]18
3d7 4s2 4p0





Co2+ [Ar]18
3d7 4s0 4p0
.



[Co)L(4]2+
3d7 4s 4p
: : : :



8-5
L L L L

( )VBT 3
:
= [e)e+2(]1/2 [PdCl4]2- [Co)H2O(4]2+

= [3 ) 3 + 2 ( ]1/2 = 3.87 B.M .

;dsp : ; -2
2

: .

[Co)L(4]2+ ;sp3 ;

3d7 4s 4p .
: : : :






L L L L

214
1
:
= 1) 1 + 2 (1/2 = 1.73 B.M


3.87 B.M sp3
1.73 B.M .dsp2

8-5


. 2 9
4 .6 2 4
.
-1 2
2 [Ag)NH3(2]+
.
.[H3N-Ag-NH3]+
( )I ( )I ( )I :

I [NC-Ag- CN ]- ; [Cl Au Cl]- ; [CN Cu CN]-


-2 3

Hg [HgI3]- .
I I
(. )Trigonal Planer
I
4 -
[ ]
-3 4

Hg

I I
( .)Square planer
( )Tetrahedral
][HgI3 -
[CoBr4]2-
. [FeCl4]-
215
- 2-
Cl Br

Fe Co

Cl Cl Br Br

Cl Br


(]Pt(NH3)2Cl2] (II

Ni2+ Cu2+ Pd2+ Pt2+ . Au3+



] [PtCl4
2-

DNA
[Ni(CO)4]2+ . [Pd(CN)4]2-

.DNA
2+ 2-
()
Co Co CN CN

Ni Pd
DNA
Co Co CN CN .
[Ni(CO)4]2+ [Pd(CN)4]2-

2-
Cl Cl

Pt

Cl Cl
[PtCl4]2-

216

(B.M) = [e(e+2)]1/2 213




( )Doble Salt ( .)Coordination Compound
.
,
.
Chain Theory


.
,
Werner s Theory
,
.

.
Ligand
.
()monodentate
( )bidentate
(.)multidentate
())EAN) Effective Atomic Number Rule

Kr Xe . Rn
.

())VBT) Valence Bond Theory


() ()
. .
( )
.


. 2
3 (Trig o
.)nal Planer
4
( )Tetrahedral (.)Square Planer
217

1-5

2-5

3-5 FeSO4 )NH4(2SO4 1:1


Fe2+ CuSO4 4 : 1
.Cu2+

4-5 :
[PtCl6]2- -1
[Pt)NH3(6] -2
[FeCl4]- -3
[Cr)NH3(6]3+ -4
50 )5 ; 33 )4 ; 31 )3 ; 90 )2 ; 86 )1 : [Ag)NH3(2]+ -5

5-5 ( ) :
[Fe)CO(5] -1
[Fe)C2O4(3]3- -2
K3[Fe)CN(6] -3
[Fe)H2O(5)NO(]SO4 -4

)+2( )4 ; )+3( )3 ; )+3( -)2 ; )0( )1 :

6-5 :
[Fe)H2O(5)NO(]2+ -1
Na2[Fe)CN(5)NO(] -2
[Co)N3()NH3(5]SO4 -3
K4[Ni)CN(4] -4
[Cr)H2O(4Cl2]+ -5
[Ni)en(2Cl2]2+ -6
[Co)NO2(3)NH3(3] -7
K2[PtCl6] -8

218
7-5 :
- ( ) (. )III
- ( )0 .
(.)III
( )II .
( )II .
(. )III
- (. )0
( ) (. )III
(. )III
(. )II

8-5 :
[Cr)H2O(4Cl2]Cl.2H2O [Cr)H2O(6]Cl3 [ Cr)H2O(5Cl]Cl2.H2O :
- ( )
-
-

9-5 :
-1 ( ) [Cr)H2O(4Cl2]+ :
5 - 6 - 1 - 3 -

-2 ( ) [Pt)C2H4(Cl3]- :
4 - 3 - 2 - 1 -

-3 ( () ( )II :
[Cu})NH2(2CO{2]Cl2 -
[Cu})NH2(2CO{Cl]Cl -
[CuCl2})NH2(2CO{2] -
.

219
-4 [Pt)NH3(3Br)NO2(Cl]Cl IUPAC :
- (. )IV
- (. )IV
- (. )IV
- (. )IV

10-5 ( )VBT :
; [PtCl4]2- ; [pd)CN(4]2- ; [Ni)dmg(2] ; [CoCl4]2- ; [Zn)CN(4]2- ; [Co)CN(4]2-
][ZnCl2)NH3(2
-
-
-

11-5 ( )VBT
( ) [Ni)H2O(4]2+ ; [Ni)NH3(4]2+

12-5 ( ) [MnL4]2- 5.9 B.M Mn II


L .
.
; sp : .
3

13-5 II [NiL4]2- L :
)1 .L
)2 .
)3 (.)

14-5 :
.

220

6
Chemical Analysis

:
.
.
.
.

.

.

.

.
.

.
221
1-6

.


.
.

.

.
.
( )
( ).

2-6 () Qualitative Analysis




( ) .

( )




.



(
)
( )
(.)1-6

( ) ( )I
222
1-6
.

AgCl Hg2Cl2 PbCl2 Ag+ Hg22+ Pb2+ HCl I
HgS CuS Bi2S3 Hg2+ Cu2+Bi3+ II
CdSPbS As2S3 Cd2+ Pb2+As3+ HCl
Sb2S3 SnS Sb3+ Sn2+
Al(OH)3 Cr(OH)3 Al3+Cr3+Fe3+ A III
)Fe(OH 3

( NH OH )NH Cl
4 4

NiS ZnS CoS MnS Ni2+ Zn2+ Co2+ B


Mn2+ NH4OH NH4Cl
CaCO3 BaCO3 Ca2+ Ba2+ Sr2+ )NH ) CO 4 2 3
IV
SrCO3 NH4OH NH4Cl

Mg2+ Na+ K+ NH4+ V


( )IV
(Ag+
! ) Hg22+ Pb2+

(
I II
)

) (Hg2+Cu2+Bi3+Cd2+ Pb2+Sn2+ As3+ Sb3+



IIIA .
HCl.


.
( )I
:
223
1-2-6
(Ag+ Hg22+
)Pb2+ ( AgCl Hg2Cl2
)PbCl2 :
-1 PbCl2 AgCl Hg2Cl2
PbCl2

K2CrO4
PbCrO4
:
PbCl2 + K2CrO4 PbCrO4 + 2KCl

-2 ( AgCl )Hg Cl
2 2
AgCl

Ag(NH3)2Cl
HNO3
KI :
AgCl + 2NH3 [Ag(NH3)2]Cl

[Ag(NH3)2]Cl + 2HNO3 AgCl + 2NH4NO3


[Ag(NH3)2]Cl + KI AgI + KCl + 2NH3


( Hg2Cl2 )I
:

Hg2Cl2 + 2NH3 Hg(NH2)Cl + Hg + NH4Cl


( )3HCl + HNO3
( )HgCl2
( )SnCl2( ) II
:
224
2HgCl2 + SnCl2 SnCl4 + Hg2Cl2

Hg2Cl2 + SnCl2 SnCl4 + 2Hg


1-6
(?)III
:
Ag+
Cd2+ ( Fe3+ )III
A
1-6 :
-1 ( HCl) :
Hg22+ + HCl AgCl
( )III .
HCl .
Bi3+ + H S2

-2
( )III
Al3+ + NH4OH NH4Cl CdS .
-3 ( )III

NH4OH ( Fe)OH(3 )III
Mn2+ + H2S
NH4Cl .

3-6 Quantitative Analysis



( )
.
( )


.
225


.

.

. :


. .

.
(

)
() .

.

1-3-6
Quantitative Chemical Analysis
:
- : .
- : .

2-3-6 Instrumental Analysis


.
( )
.

226
4-6 Gravimetric Analysis
(
)
.


.
.
( )
:
-1 .
-2 .
-3 .
-4 .

.

1-4-6 Volatilization Methods


(
) :
- ( )
(.)1-6
-
.



.
( )BaCl2.2H2O
:


BaCl2.2H2O BaCl2 + 2H2O
( )

227
1-6
.

.
( )BaCl2
.

CaCO3 .

CaCO3 + 2HCl CaCl2 + CO2 + H2O


( )

(
CO2
)
( ) CO2
.

228
2-6
1.4511g
( ) BaCl2.2H2O
. 125 C

.1.2361g .

:

%100 =


BaCl2.2H2O BaCl2 + 2H2O
( ) ( ) ( )

mH2O )g( = m BaCl2.2H2O - m BaCl2

mH2O )g( = 1.451 )g( 1.236 )g( = 0.215 g

:
(mH2O )g
= % H2O 100%
m BaCl2.2H2O ()g

(0.215 )g
= 100% = 14.81%
(1.451 )g

229
3-6

. 15.24 mg
CO2 CO2 .22.361mg
.
:

% 100 = 2-6

(
CO2
:
)
C CO2
1.4gg .
C CO2:

(MC )g/mol
(mC = m CO2 )mg .2.2mmg
(M )g/mol
CO2
.
(12 )g/mol
) (
mC = 22.36 mg = 6.1 mg % 0.043 :
(44 )g/mol

mC (6.1 )mg
=%C = 100 % 100 % = 40%
m )
15.24 mg (

5-6 Precipitation Methods



(
) :
-1 .

.
-2
() (

) .

-3
.
230
-4 : (
)


( :)
( ) ( )
.
-5 : (
) (
)100C (
)1000C
. :

130C
CaC2O4.H2O CaC2O4 + H2O
( ) ( )

130C
Al2O3. xH2O Al2O3 + x H2O
( ) ( )

( ) :

CaC2O4.H2O CaC2O 4 + H2O
130-225C 400-490C

840C
CaCO3 + CO CaO + CO2

-6 ( ) .


:
-1 (
)
.
231
-2

(
)
.
-3 ( )

.


1-5-6


100C



.


.

2-5-6


( )

( 1000C
)10-4 10-6 mm

.

:
-1 :
BaSO4
AgCl .
-2 :
( ) .
232
-3 :


.
-4 :
( )
.

.

3-5-6



.
( )
.BaSO4
( BaSO4 ):


Ba2+ + SO42- BaSO4 BaSO4


Ca2+ + C2O42- CaC2O4 CaO + CO + CO2



.

( ) :
233
( )
=
( )


() Gf
( G gravimetric f factor )
( )
( )
:

( )g/mol a M
= Gf
b ( )g/mol M 3-6
:
a b

Gf
. :
AgI I
4-6 Ni)C4H7N2O2(2 Ni

( ) Fe2O3 Fe3O4

( )M = 35.5 g/mole (AgCl ) AgI MgI2


Al2O3 NH4Al)SO4(2
(.)M = 143.5 g/mole
: 0.967 ; 0.203 ; 0.54 :
4.647 ; 0.592
AgCl
:
a M ) g/mol (
Cl
= Gf
(b MAgCl )g/mol 4-6


( )M = 56 g/mole
a b
.)M =160 g/mole( Fe2O3
Gf :
0.7 :
)
35.5 g/mol (
= Gf = .
(143.5 )g/mol

234
5-6
1201mg
( )
:
( )g )g( = Gf m m ()1


.
:
)M = 35.5 g/mole( Cl
( )g m
= % ( 100 % )2
( )gm
.1531mg
% 31.5 : m ( )1 ( )2 :
6-6 ( )g Gf m
100 %
% =
0.764 g ( )gm
Al2O3.xH2O
5-6
.NH4OH 3.164g
.
. Al2O3 :
.1
.
() .2 CaC2O4 ( + CO2 + CO) CaO
? )M= 56 g/mole( CaO
. .3
)M= 128 g/mole( CaC2O4
)M= 102 g/mole( Al O
2 3 a = 1 ( b=1 ).

a (MCaO )g/mol (56)g/mol
= Gf = = 0.4375
(b MCaC O )g/mol( 128 )g/mol
2 4
.0.127 g% 16.62 : CaO
.4 CaC2O4 :
( )M= 27 g/mole . (mCaO )g( = Gf m CaC2O4 )g
% 8.8 :
:
mCaO )g( = 0.4375 3.164 )g( =1.384 g
235
6-6 Volumetric Analysis

(
) (
) .

( ....) .

.

(
)
.


.

:
.1 .
.2 (
) .
.3
.
.4 (
).
.5 .
.6 .
1-6-2 Titration
- -
( )
.

( )Burete
[ ( .])2-6
236


(.)Equivelent point
(
)
( )Indicators


(
) ( )End point

( ) .




2-6 .
-
. :
- .1
.pH 10 .
.2 ( ).
.3 (
) .
.4

.

:
.1 ( ) :
(
)
( )

.
237
.2 :
(
!

)


.
.3 : ( H+ )OH- .
.

Ag+ + Cl- AgCl
.

.4 : ( H+ )OH-
( )
:

2 CN- + Ag+ [Ag(CN)2]-


2-6-6

.

:
()M

:
)m (g
( ) )n(mol )n (mmol )M (g/mol
= M mol/L = =
)V(L )V(mL )V(L

n M V
.
() ()N


() .

:
238
!
)m (g
)Eq (eq )Eq (m eq )EM (g/eq
= )N(eq/L = =
)V(L )V(mL )V(L
) EM (g/eq
Eq EM .
M ).M (g/mol
\
( )g/mole EM
( )


\ (.)g/eq
()
(
)
.

3-6-6 ()EM

:
.1

( 1.008 g )
() :


= ()H+

Ma
= EM
()H+

a acid .

( 17.0081g
).

239
Mb
= EM
( )OH-

b base .




:

Na2CO3 + 2HCl 2 NaCl + H2O + CO2


Na2CO3 Na2CO3
= = Na2CO3
2
( )H+
MNa2CO3
= EMNa2CO3
2
.2

(
1.008 g ) :

=

M
= EM

(
)
:

M
= EM

240
.3

1.008 g
8.000 g

:
KMnO4
K2Cr2O7
:
2KMnO4 K2O + 2MnO + 5O
K2Cr2O7 K2O + Cr2O3 + 3O

5
:

2
= KMnO4
10
: K2Cr2O7


= K2Cr2O7
6

! ( ) ( )
(
( ) (). ) ( ) .1
.

241

(M)g/mol M
= EM = ()g/eq
( )eq/mol
M ()mg/meq
=

6-6
:
!
(1. Pb)NO3(2)aq( + 2KI)aq ( )
+
(PbI2)s(+2K)aq( +2NO3 )aq
-

(2. H2SO4)aq(+2NaOH)aq (.)n


+
(2H2O)l(+2Na)aq( + SO42-)aq
2+ - +
(3. 5Fe)aq( + MnO4 )aq( +8H3O)aq
3+ 2+
(5Fe)aq( + Mn)aq( + 12H2O)l

(4. AgNO3)aq( + 2KCN)aq


([ Ag)CN(2])aq( + 2 K)aq( + NO3 )aq
- + -

:
.1
.
MPb)NO3(2
= EMPb)NO3(2

Pb)NO3(2
.
]M [Pb)NO3(2] = 1207+[2114]+[2316
= 331 g/mol


(Pb)NO3(2)aq(+2KI)aq
+ -
(PbI2)s(+2K)aq( +2NO3 )aq

Pb)NO3(2
PbI2 :
242
= 2 1= 2 eq/mol =

MPb)NO3(2 (331 )g/mol


=EMPb)NO3(2 = =165.5 g/eq
(2 )eq/mol
KI
M)KI( =139+1127=166 g/mol
=11=1 eq/mol =

MKI (166)g/mol
= EMKI = = 166 g/eq
)
1 eq/mol (

.2 -:
(H2SO4)aq( + 2NaOH)aq
+ 2-
(2H2O)l( + 2Na)aq( + SO4 )aq


M )H2SO4( = 2 1 + 1 32 + 4 16 = 98 g/mol

= 2 eq/mol =
MH2SO4 (98 )g/mol
= EMH2SO4 = = 49 g/eq
(2 )eq/mol

M )NaOH( =1 23 + 1 16 + 1 1= 40 g/mol
=1 eq/mol =

MNaOH (40)g/mol
EMNaOH = = = 40 g/eq
(1 )eq/mol
.3 :
2+ - +
(5Fe)aq( + MnO4 )aq( +8H3O)aq
3+ 2+
(5Fe)aq( + Mn)aq( + 12H2O)l

( Fe2+
).
M )Fe( = 56 g/mol
243
= 1 eq/mol = 7-6
M EM Fe2+
Fe3+ .
MFe )
56 g/mol (
= EMFe = = 56 g/eq AgNO3 - :
(1 )eq/mol + -
(Ag )aq( +Br )aq (AgBr )s
( MnO4- 170 ; 1 :
) BaCl2 - :
M )MnO4( =155+416 = 119 g/mol 2+
(Ba)aq( +SO4 )aq
2-

(BaSO4)s
= 5 eq/mol =
104 ; 2 :
Fe2)SO4(3 - Mn2+ MnO4-
:
MMnO4 (119)g/mol 2+
= EMMnO4 = = 23.8eg/eq (Fe2)SO4(3)aq( +3Pb)aq
(5 )eq/mol 3+
(3PbSO4)s( +2Fe)aq

.4 : 66.7 ; 6 :
Na2CO3 - ( +
(AgNO3)aq( + 2KCN)aq
- + -
) :
([ Ag)CN(2])aq( + 2 K)aq( + NO3 )aq +
(Na2CO3 +2H)aq
+
(2Na)aq( +CO2)g( + H2O)l
AgNO3
53 ; 2 :
M)AgNO3( =1108+114+316 = 170 g/mol
BaI2 - :
= = 2 eq/mol 2+
(2BaI2 +Hg)aq
2- 2+
MAgNO3 (HgI4 )aq( +2Ba)aq
(170 )g/mol
= EMAgNO3 = = 85 g/eq 195.5 ; 2 :
(2 )eq/mol
Na2S2O3 -
KCN
:
M)KCN( =139+112+114 = 65 g/mol 2- 2-
(2S2O3 )aq (S4O6 )aq

= 1 1 = 1 eq/mol = 158 ; 1 :

MKCN (65 )g/mol


= EMKCN = = 65eg/eq
(1 )eq/mol
244
7-6
8-6
= 2
K2Cr2O7 .eq/mol .0.231mol/L
( )M = 294 g/mole :
2 L 0.12 N
.
?
Cr2O72-+14H++6Fe2+ ( )eq/mol
2Cr3+ + 7H2O + 6Fe3+
:
11.76 g :
(N )eq/L( = )eq/mol( M )mol/L
:
9-6
N = 2 )eq/mol( 0.23 )mol/L( = 0.46 eq/L.

NaOH 4-6-6
500emL
?0.2 M ( )L
4g : ( )mL
( .)1L = 1000 mL
( )
10-6
:

.1 : .
:
.2 : .
350emL .1 0.125eM .3 :
. (.)3-6
250emL .2 0.1N 5-6-6
Na2B4O7.10H2O
()M = 381 g/mole
:
(
B4O72- + 2H3O+ + 3H2O
)
( )
4H3BO3
.
; 4.76 g .2 7.44 g .1 :
()

245

.

Ba(OH)2
) (M= 171 g/mol :
.1
251mL (
).
.2 201mL
(
) .
.3

pH
.
.4 (
) (
0.0981M )0.0981N

. 22.41mL
( )
( ).

3-6

246
.5:
( )
( ) .
(
... ) ( )mL (
mol/L eq/L ) :

) (mL) M (mmol/mL (mmol) = V



) (mL) N (meq/mL (meq) = V


:
=
( )meq = ( )meq

(N V)HCl = (N V) Ba(OH)2

NHCl VHCl = NBa(OH)2 VBa(OH)2

)0.098 (meq/mL) 22.4 (mL) = NBa(OH)2 20 (mL

. Ba(OH)2
)0.098 (meq/mL) 22.4 (mL
= NBa(OH)2 = 0.1097 meq/mL = 0.1097 eq/L
)20 (mL
(

! )251mL
:
mmole/mL

mole/L
Ba(OH)2 + 2HCl BaCl2 + 2H2O
. M
= 2 eq/mol:

EMBa(OH)2 )171 (g/mol


= )EMBa(OH = = 85.51g/eq
2
)2 (eq/mol

247
(m )g
(Eq )eq (EM )g/eq 11-6
= = (N )eq/L
(V )L (V )mL
(1000 )mL/L KMnO4

:
(N)eq/L( V )mL( EM )g/eq MnO2
= (m )g
(1000 )mL/L

(0.1097 )eq/L( 25 )mL( 85.5 )g/eq .0.05 M
= (m )g
(1000 )mL/L 0.15 eq/L ; 3 :

m )g( = 0.235 g
8-6 12-6

)M = 90 g/mol( H2C2O4
:
0.1743 g
Ni2+ + 4CN-
39.82 mL 0.09 M
Ni)CN(42-
160 mg . .
:
38.3 mL KCN :
0.137 N
2NaOH + H2C2O4 Na2C2O4 + 2H2O
.
Ni2O3
()M=165 g/mole .
:
% 67.5 :

1
= )mmol( H2C2O4 )mmol( NaOH
2

( )mmol :
( )mL( M )mmol/mL)mmolNaOH( = V

(mmolNaOH = 39.82 )mL( 0.09 )mmol/mL


= 3.584 mmol
248
NaOH
! :


( )


=

.

= ()mmol H2C2O4
2
3.584 mmol
: =1.79 mmol = 1.79 10-3 mol
2
N1 V1 = N2 V2

(m )g
(M )g/mol
= (M )mol/L
. (V )mL
:
13-6
(m )g( = M )mol/L( V )L( M )g/mol
0.958 g
CH3COOH
( )L ( )M=60 g/mol

( )mol/L (.)mol
0.225 N
m )g( = 1.79 10 )mol( 90 )g/mol( = 0.16 g.
-3


. 33.6 mL
:
.
% 47.34 : (m H2C2O4 )g
=% H2C2O4 100
( )gm

(0.16 )g
= 100 = 91.8 %
(0.1743 )g

249

(g/mol) 234 a M
= Gf
b ) (g/mol M

)n(mol 238
= )M (mol /L
)V(L

)Eq (eq 239


= )N(eq/L
)V(L


()

.

.


.


.


.
Gravimetric Factor
()
( ) .

250
Titration
( )

.
Equivalent Point
()
( ).
End Point
()
() .

() .
Indicator

.
Nutralization Reaction

.

()
Redox Reaction
.
Salt
( )H+ ( OH- )O2-
Equivalent Mass
( ) 1.008 g 17.008
g 8.000 g
.

251

1-6 ?

2-6
.

3-6 :
K2CrO4
PbCl2
NH3 HNO3
AgCl
HCl NH3 SnCl2 SnCl2
Hg2 2+

4-6 Hg2+ Hg22+

5-6 :
-1 Cr3+ ....... ........ . ........
-2 ........ ....... . ......
-3 Al2)SO4(3 0.3N = . .....
-4 0.2M 1 eq/L = . ......
-5 5.7mg 15.675mg
. CO2 = . .........

6-6 KIO3 1.67 g ?Pb)IO3(2


1.283 g :

7-6 5.71mg 14.41mg 2.5 mg


. .
% 4.9 ; % 68.9 :

8-6 ? 36.7 mL
43.2 mL 0.236 M ?
0.278 M :
252
9-6 9.5 g 2 L
-?
0.054 eq/L ; 0.027 M :

10-6 :
. 10 mL 0.15 M 10 mL ?
. 10 mL 0.15 M 10 mL 0.3 M ?
: ; 0.075 M .0.225 M .
11-6 0.2 M ( ) 40
mL 0.1 M ( )II ? :
10FeSO4+2KMnO4+8H2SO4 5Fe2)SO4(3+2MnSO4+K2SO4+8H2O
4 mL :
12-6 :
-1 ( Fe2)SO4(3 )III :
() .4 eq/mol
() .5 eq/mol
() .6 eq/mol

-2 Cu2+ Ca2+ :
() HCl.
() H2S NH4OH NH4Cl .
() H2S HCl .

-3 ( C14H9Cl5 )DDT 0.74 g


0.253 g AgCl:
() . % 17
() . % 19
(). % 21

-4
:
() .
() .
() .

253
-5 :
() .
() .
() .

-6
( ) :
() .
() .
() .

-7 :
() .
() .
() .

-8 13 g K2Cr2O7 500 mL :
() . 0.53 mole/L
() . 0.53 eq/L
() . 3.18 eq/L

13-6 )M = 368 g/mole( Na5P3O10 )M= 222 g/mole( Mg2P2O7


1.11 :

14-6 50 mL )M = 176 g/mole(HIO3


0.145 N
45.8 mL:
. .HIO3
. .
HIO3 + 4FeCl2 + 5H3O+ + 6Cl- 4FeCl3 + ICl2- + 8H2O
: ; 0.13 N .0.52 N .
15-6 NaOH 25 mL
H2SO4 0.08 M .47.1 mL
NaOH 500 mL .
6 g ; 0.3 M :
254
16-6 ( Bax2.2H2O x )

0.266 g 200 mL H2SO4

) M= 137 g/mole ( Ba ) M= 233 g/mole( BaSO4

0.254 g x ?

:
17-6 0.5 g ) M= 150 g/mole( NaI

AgNO3 0.744 g

. ) M= 235 g/mole( AgI .


% 95 :

18-6 4.29g NaCO3 x H2O


250mL 25mL 15mL HCl

0.2N . ( )x ?
NaCO3 .10 H2O :
19-6 6.0 M H3PO4
( ) M= 98 g/mole :
-
3-
(H3PO4)aq( + 3OH-)aq (PO4 )aq( + 3H2O)l
-
2- +
(H3PO4)aq( +2NH3)aq (HPO4 )aq( + 2NH4)aq
-
(H3PO4)aq( + F-)aq (H2PO4-)aq( + HF)aq

: ; 18 N ; 32.7 g/eq . ; 12 N ; 49 g/eq .6 N ; 98 g/eq .

20-6 2.5 g ( M CO3 M ) 100 mL

. 0.6 N 50 mL

NaOH 0.2 N . .
Ca ; 40 g/mol :

255
21-6 20 mL KMnO4 0.3 N
KI I2
) M= 158 g/mole( Na2S2O3 :
I2 + 2Na2S2O3 2NaI + Na2S4O6
25 mL .:
- . Na2S2O3
- 1L .
: ; 0.24 N .37.92 g .

22-6 ) M = 233g/mol( BaSO4

) M = 208g/mol( BaCl2 100mL ( .) M = 98g/mol

20mL 16mL NaOH 0.10M.


0.932g :

256

Organic Chemistry for Substituted
Hydrocarbons 7

:
.
( )IUPAC
( ).

.
.

.
.

257
1-7 H
H C H

H
()
.
H H

C C
H
H
()


( )
H C C H
. ()

()


.
()
1-7
()
- - 2 ( )
.C4H10 1-7
:
H
H C H
H H H H C5H12 .
H H
H C C C C H H C C C H
H H H H H H H 1 - 7
- - 2



.


() () ()

IUPAC ( )1 - 7 . - - C1

C2
2-7
C3
CH3
: IUPAC
C4
CH3 CH CH CH2 CH3 CH3 CH3
CH3 CH2 C CH2 CH CH3 C5
CH2
()1 CH3
CH3 C6
1
CH2 2
C CH3 ()2
3
CH2 4
CH3 C7
()3 :
C8
( - 3 ) 1 2 -
C9
( - 442 ) 2
C10
( - 2 ) 3 - 1 -
258
2-7

. ()IUPAC
)(1 CH3
CH3 CH CH2 C C H

)(2 CH3 CH3
CH3 CH CH CH CH CH3

CH3
)(3 .
CH3 C CH3
CH3 ().


2 - 7
()


()


CH3Br R X
CnH2n+1X X

)x = (Cl, Br, I
CH3CH2 OH
CnH2n+2O OH R OH


CH3 O CH3 OR
CnH2n+2O

R O R

O O
O
CH3 C H CnH2nO C
R C H


O O
O
CH3 C CH3 CnH2nO C
R C R

.
O O
O
CH3 C OH CnH2nO2 C O H
R C OH
( )2 - 7
O O
O
CH3 C OCH3 CnH2nO2 C OR
+ R C OR

H
R N H

H
CH3CH2 NH2 .
CnH2n+3N NH2
R N R

R
R N R

259
2 - 7 Alkyl Halides

. ( )3 - 7
H- .
)(R-H R-
3 - 7
.

( ) R - ( ) X -

CH3 CH4
C2H5 C2H6
CH3 - CH2 CH2 C3H8

CH3 CH CH3
- C3H8
.
()
= R R X C X

Cl , Br , I = X :
H H H
H C Cl H C C Br
H H H

Methyl chloride Ethyl bromide
( )1 ()2
( )3
.


!
mono
)1( primary
R R H
R C X R C X R C X
R H H di

()3 ()2 ()1 )2( secondary


tri
)3( tertiary
:
260
Cl Cl Cl
Cl C Cl H C Cl H C Cl
Cl Cl H
() ( )
tetrachloromethane trichloromethane dichloromethane
I chloroform
H C I 1 - 2 - 7
I ( ) IUPAC
:
()
- 1
Triiodomethane

Iodoform .
- 2
Br Br
( ) -
H C C H
( ) ( ) -
H H

- 2 1
1,2 dibromoethane .
.4-7

. 4 - 7

!

- 2 Cl
2-chlorobutane 4
CH3 3
CH2 CH
2
CH3
1

Br
- 22
3
CH3 2
C 1
CH3
2,2-dibromopropane
Br

. Cl Br
- 2 - 3 - 4
CH3 3
C C
2 1
CH3
2-bromo-3-chlorobutane
H H

- 1 - 3 - CH3
1-bromo-3-methylbutane 4
CH3 3
CH 2
CH2 1
CH2 Br

261
2 - 2 - 7 3-7
- 1 :
:
- 11 -
- 2 - - 2 -

- 2 - - 2 -
HX ( HBr HCl )HI
- 2

:
CHCl3 -

( ) . Br Cl
- . CH2 CH CH3
-
- 2:

CH2 CH2 HBr CH2 CH2



H Br

Br
CH3 CH CH CH3 HBr CH3 CH CH2 CH3
- 2 - 2

:
- 1 ( )H+ (.)Br-
H Br H+ Br-:
!
- 2 ( ) ()
()Carbonium ion

H+ CH2 CH2 CH3 CH2 (
carbonium ion ) .


- 3 ( )
( ) ( )
.
.
CH3 CH2 Br CH3CH2 Br

262
(
)

.
CH2 CH2 HBr BrCH2CH3

CH2 CH2 HBr CH3CH2Br

(
) . .
Br
A A
CH3 CH CH3
- 2
CH3 CH CH2 HBr


B B CH3CH2CH2Br
4-7 - 1

HCl ( ) A
-1 -2. - 2 - 1 .
.
( )Valdimir Markovnikov :

( )

.
( )3
( )2 ( . )1
A .
:
H Br H Br
CH3 CH CH3
()2
CH3 CH CH2 H
CH3CH2CH2
()1

Br
CH3 CH CH3 Br CH3 CH CH3
- 2
263
3-7
:
5-7
) 1 .

- 2 ) 2 .
: :
- 1 ( - 2 )
1( CH2 CH2 HCl CH3 CH2 Cl
.
2( CH3 CH CH2 HI CH3 CH - 2 CH3
HCl ( - 2 ).
I
- 2

3 - 2 - 7
- 1
CH3Cl CH3Br CH3CH2Cl

C18 ()C18
. CCl4

. .
.
- 2
.




. . D.D.T

( ) .
H
: Cl C Cl
Cl C Cl
H H Cl

Nu:
R C X

R C Nu X:
D.D.T

H H

:
- :KOH
()
( )-OH :
264
H2O
CH3I + KOH CH3OH + KI

H2O
CH3CH2Br + KOH CH3CH2OH + KBr

- KOH:

. HX .
H Br :
C H OH
CH2 CH2 KOH 2
CH2 CH2
5
KBr H2O

C2H5OH
CH3 CH2 CH2 Br KOH CH3CH CH2 KBr H2O

6-7 -
Mg

( ) (
:
) . :
- 1 CH3I + Mg CH3MgI
KOH
- 2 - 2 - . ( )
- 2
CH3CH2Br + Mg CH3CH2MgBr
KOH
- 1 - 3 - . ( )

3 - 7

( )-OH ()
R - OH (.)CnH2n+2O
CH3
CH3 C OH
:
CH3
- 2 -2 -

C OH
CH3 CH CH3
OH CH3CH2OH
CH3OH
- 2
265

( )1

( )2 (. )3



R R H
R C OH R C OH R C OH
R H H :
()3 ()2 ()1 CH2 OH

1 - 3 - 7 CH2 OH

IUPAC ( )
CH2 OH
: CH OH
- 1 CH2 OH

( )
.
- 2
(). 7-7
- - 3
: :
CH3 OH CH3 CH3
CH3 CH2 CH2 OH CH3 CH CH3 CH3 C OH
CH3 C CH CH CH3
- 1
OH CH3
CH3
- 422 - 3 -
OH
CH3
CH3 CH CH CH2 CH3
CH3 CH OH Cl CH2 CH2 CH2 OH
CH3
- 3 - 1 -
- 2
- :

2 - 3 - 7 - 2 )1 - 2 -
- 22 )2 - 1 -
: - 3 )3 - 3 -



(
)
:
266
OH
H2SO4
CH3 CH CH2 H OH CH3 CH CH3

- 1 - 2

:
OSO3H

CH3 CH CH2 H OSO3H CH3 CH CH3
- 1

OSO3H H OH

CH3 CH CH3 O H CH3 CH CH3 H2SO4
- 2

! 4-7
74 g/mol
OH-
O = 16 , C = 12 , H = 1
OSO3H-
:
( ) CnH2n+2O
.
(CnH2n+2O = )n * 12( + ]) 2n+2( * 1[+)1 * 16

CnH2n+2O = 12n + 2n + 2 + 16 = 74 g/mol
.
14n = 74 -18 n = 4
C4H10O :

8-7
CH3

CH3 CH2 CH2 CH2 OH CH3 CH CH2 OH

- 1 ()1 - 2 - 1 -
88 g/mol
()1 ()1

OH OH
9-7
CH3CH2CH CH3 CH3 C CH3
:
- 2 )1 - 1 - 2 ()2 CH3
()2 - 2 - 2 - ()3
)2
()3

267
3 - 3 - 7
- 1


.
[ (.])1 - 7
( ) C3 - C1
.

H O H O H O H O H O H O
1-7
H R H R R R -
() () -

- 2 : .

:
-

2CH3CH2OH + 2Na 2CH3CH2O-Na+ + H2

- ( )PCl3
( )PCl5 :

CH3CH2-OH + PCl5 CH3CH2-Cl + POCl3 + HCl


3CH3-OH + PCl3 3CH3-Cl + H3PO3 !

OH
-
HX .
OH Cl .
CH3CH-CH3 + HCl CH3CH - CH3 + H2O
-2 -2
268

.
:
HI > HBr > HCl

(
)


(
) 5
.
5 - 7
- 1 - 2 - 2 - 2 -
:
ZnCl2/HCl
- 2 ( ) 5
- 2 - 1 ( )
. - 2 2 - ( )
.
ZnCl2
CH3 CH2 CH2 OH HCl N.R
- 1

CH3 CH3
ZnCl2
CH3 CH OH HCl CH3 CH Cl H2O
- 2 - 2

CH3 CH3
ZnCl2
CH3 C OH HCl CH3 C Cl H2O
CH3 CH3
- 2 - 2 - - 2 - 2 -


:
- 1
(. )3
269
- 2 5 - 2
(. )2
- 3 ( )1
.

-
()170C
.
H2SO4
CH3-CH2-OH CH2=CH2 + H2O
()170C

:
:
- 1 :
: :
: :

CH3 CH2 O H H CH3 CH2 O H


H
- 2
:

H2O
: :

CH3 CH2 O H CH3 CH2


H 10 - 7

- 3 :
- 2 - 2
H H H H
- 1 .
H C C H C C H H
H H

OH
OH

H+
.

270
11 - 7 -

. :
- 1 )1
KMnO4/H2SO4 K2Cr2O7/H2SO4
( )1
- 2 )2
:
- 2 )3 - 2 -
O O
][O ][O
RCH2OH R C H R C OH

O O
K2Cr2O7/H ][O
CH3CH2OH CH3 C H CH3 C OH
][O

() () ( )


OH O
K2Cr2O7/H
CH3 CH CH3 ][O
CH3 C CH3

- 2 () ()


OH .
12 - 7

CH3
K2Cr2O7/H

CH3 C OH NR
. CH3
- 2 -2 - ()
.
NR ( )NO Reaction .

Ethers 4 - 7
( ) R - O - R
( .)CnH2n+2O

R = R
R R .
271

()

C - O - C CH3 O CH2CH3
CH3 O CH3

1 - 4 - 7

IUPAC :
- 1 ( )
( )
.
- 2 .
- 3 RO :
.
- 4 :



RO RO

( ) RO -:
13 - 7
)1 CH3O -
)2 CH3CH2O - -

)3 CH3CH2CH2O - :
CH3 CH3 O CH3
CH3 CHO -2 )4
CH3
6 - 7 CH3 O C CH3
: CH3
- :
Cl OCH3
:
CH3O 1
CH2 2
CH CH3
3 3
CH3 CH
2 1
CH3
- 22 )1
- 2
- 2
OCH2CH3
- 2 )2 - 2 -
1
CH3 2
CH 3
CH2 4
CH3

- 2

272
2 - 4 - 7

:

( )Williamson 14 - 7
- 1
. :
ROH + Na RONa + 1 H
- 2 : 2 2

CH3 CH2 CH2-Br+ CH3-ONa



R-ONa + R-X ROR + NaX




:
CH3-ONa + C2H5-Br CH3OC2H5 + NaBr

C2H5-ONa + C2H5-Br C2H5OC2H5 + NaBr




.
3 - 4 - 7
- 1



. .

(.)dryness


:
273
R R R R R R

::
::
O O

::
::
CH2 CH2 O O
R R H H
R R


. . .

R O
CH2 O R R O

R H H O H H O H H

R
H

- 2

.
:
15 - 7
- :

H2SO4
CH3CH2-O-CH2CH3 + H2O . : 2CH3CH2OH



HCl
.
.
.

. :
H

::

CH3 CH2 O CH2 CH3 H2SO4 CH3 CH2 O CH2 CH3 HSO4
:

- PCl5
PCl5
.

RO-R + PCl5 R-Cl + R -Cl + POCl3
:


CH3CH2-O-CH2CH3 + PCl5 2CH3CH2-Cl + POCl3

274
5 - 7 Aldehydes and Ketones

O

R C H
O

C
CnH2nO



O
.
R C R
1 - 5 - 7

IUPAC :
-
- 1 1
-
.
- 2
() .
()
- 3 .
.

.
:


O O

CH3 C H H C H
Ethanal Methanal
:
Cl O O
.
CH3CH C H CH3CH2 C H
- 2
2 - chloropropanal propanal

275
-
:
- 1
.
- 2
() .
- 3 :
O O CH3 O
CH3CH2 C CH2CH3 CH3CH2 C CH3 CH3CH C CH3
- 3 - 2 - 3 - 2 -
3 - pentanone 2 - butanone 3 - methyl - 2 butanone

2 - 5 - 7

: 16 - 7

- 1
:
( )1 ( )2 O
CH3 CH CH2 C H ( )K2Cr2O7 ( )KMnO4
. : Cl
H H
K2Cr2O7/H CH3
R C O H ][O
R C O H2O O CH2
H CH3 C CH CH3

O - 2
K2Cr2O7/H
CH3CH2OH CH3 C H H2O :
][O
- 3 )

- 2 )

( )
.
H O O
K2Cr2O7/H K2Cr2O7/H
R C O H ][O
R C H R C O H
H
276
(. )2
!
H R
K2Cr2O7/H


R C O H ][O
R C O
R


:
OH O
. K2Cr2O7/H
CH3 CH CH3 ][O
CH3 C CH3
- 2


.

7 - 7
:

- 1 )1
- 2 )2

:
()1 O
][O ][O
(). CH3CH2CH2 OH CH3CH2 C H H2O
- 1

O
CH3CH2 C O H

17 - 7



:
()2
)1
- 2 )2 OH O
][O
CH3CH2CH2 CH CH3 CH3CH2CH2 C CH3
- 2
- 2

277
3 - 5 - 7
- 1
(
() )


.


O : .

CH3 CH3 H C H CH3 OH



M = 30 , bp = -89C M = 30 , bp = -21C M = 32 , bp = 64.5C

CH3 O OH
CH3 CH CH3 CH3 C CH3 CH3 CH CH3


M = 58 , bp = 56C M = 60 , bp = 82.5C
M = 58 , bp = -12C
( )bp ( )M .

- 2

:

C O C O C O

( ( ) )Nu:
(
).


::

C O
278

:
:

.
Nu
Nu: O: O:

::
C C

: ( )
.H+
Nu18 - 7 Nu
: O:

::
C
::
C H O H

:
O
][Red
-
C2H5 C CH3
H2/Pt H2
Ni Pt .
O
][Red :
C2H5 C H O
H2/Ni Ni
CH3 C H H2 CH3CH2OH
][Red

O OH
Ni
CH3 C CH3 H2 CH3 CH CH3
][Red
- 2 ()2

[ ]Red Reduction .
-

( )Clemmensen reduction
( -)
:
O
Zn/Hg
R C R R CH2 R
HCl

O
Zn/Hg
CH3 C C2H5 CH3CH2 C2H5
HCl
- 2
279
O
Zn/Hg
R C H R CH3
HCl
O
Zn/Hg
CH3 C H CH3 CH3
HCl 19 - 7
-
- 1 .
( )NH2NH2
- 2
.
H H .

CH3C=O + H2N-NH2 CH3C=N-NH2 + H2O

CH3 CH3

CH3-C=O + H2N-NH2 CH3- C=N-NH2 + H2O


( )
.
-


.


()

.

:

- 1 Tollens Reagent

( )
.



. :
O O
R C H 2Ag(NH3)2OH R C O NH4 2Ag H2O 3NH3

( )
280
6 5 4 3 2 1
O
R C R 2Ag(NH3)2OH NR
- Fehlings Solution

( )II

( )II
( )I .
( )I .
.
O O
-
R C H 2Cu2+ 5OH- R C O Cu2O 3H2O

20 - 7 O
R C R 2Cu2+ 5OH- NR

:
- .
- . 6 - 7 Carboxylic acid
()
O
) ( )-COOH (
C OH
: .
O
R R C OH

CnH2nO2

1 - 6 - 7
() IUPAC
:
- 1
281

O (.)1
C OH
1
- 2 .
- 3 .
- 4

()
:
CH3COOH HCOOH 21 - 7

- 1

Br Cl O
3
CH3 2
CH 1
COOH CH3 CH2 C C OH
- 2 CH3
CH3 CH3 - 2
6
CH3 5
CH2 4
CH 3
CH 2
CH2 COOH
1
:
- 43 ) - 2

2 - 6 - 7 ) - 3

( )1 .

CO2
:


R - X + Mg RMgX

O O
H2O/H
RMgX CO2 R C O MgX R C O H

O O
H2O/H
CH3MgBr CO2 CH3 C O MgBr CH3 C O H

282
22 - 7
: .
)1
) 2 3 - 6 - 7

- 1
O ( )C10

R C O
H


.
O HO
R C C R
OH O


.
- 2
-


.
O O
R C O H NaOH R C O Na H2O

-

CO2
23 - 7 .
RCOOH + NaHCO3 RCOONa + CO2 + H2O
. NaHCO3 - ()

( H2SO4 ) HCl
( )Reversible
( )Esterification :
283
O O
H
R C O H H OR R C O R H2O

O O
H
CH3 C O H H OC2H5 CH3 C O C2H5 H2O
( )

-

LiAlH4
H2 / Ni :
O
LiAlH4
CH3 C OH CH3CH2OH



8 - 7



O O
CH3 C H CH3CH2 C OH

:

: .
O
R C H + H2
Pt
R-CH2OH
PCl3
R-CH2-X Mg COOH

CO2 HX O
RCH2MgX RCH2COOMgX RCH2COOH + MgX2
O O C CH3
Pt
CH3 C H + H2 CH3CH2OH

3CH3CH2OH + PCl3 3CH3CH2Cl + H3PO3




CH3CH2Cl + Mg CH3CH2MgCl
24 - 7

CH3CH2MgCl + CO2 CH3CH2COOMgCl

CH3CH2COOMgCl + HCl MgCl2 + CH3CH2COOH



284
25 - 7 9 - 7
.
:

:


CH3-CH2-Cl + Mg CH3CH2-MgCl

1( CO2
CH3CH2MgCl CH3CH2COOH
2( H2O/H+

)Vitamine C( C




.
C
7 - 7 Esters

O
( )R C O

R

.
O
R C O R CnH2nO2

R = R R R R H .R

:

.


285
1 - 7 - 7

O
R C O R
O
R R C OH OH .
:
26 - 7
- 1 - 1 :
( )R . )
O
- 2
CH3 CH C O C2H5
. :
CH3
Cl O O )
CH3 CH C O C2H5 CH3 C O CH3 O
- 2 - H C O CH3
- 2 :
2 - 7 - 7 - 32 -

( 27 - 7
- 1 . ) .
- 2
HCl : .
O O - 3

R C Cl H OR R C O R HCl .

CH3COCl + CH3OH HCl + CH3COOCH3

3 - 7 - 7
- 1 .

O

R C O
. R

.
286
- 2
:
-

( H2SO4
)HCl
O O
H
R C O R H2O R C OH R OH
:

O O
H
CH3 C OC2H5 H2O CH3 C OH CH3CH2OH

-
28 - 7
NaOH

()Saponification .
:
O O
H2O
CH3 C OCH2CH3 NaOH
CH3 C O Na CH3CH2OH

8 - 7 Amins
NH3

( )1
RNH2
( )2
R2NH ( )3
R3N :
H R R
R N H R N R
R N H
. ()1 ()2 ()3
287
:
CH3 CH3 H
CH3 N CH3 CH3 N H CH3 N H

()3 ()2 ()1

1 - 8 - 7
-
- 1
.
:
CH3 - NH2 CH3CH2 - NH2 CH3CH2CH2 - NH2
-
- 2
.
CH3 :
CH3
CH3 N H CH3 N CH3

- 3
.
H CH3
CH3 N CH2CH3 CH3CH2 N CH(CH3)2

- IUPAC:
- 1
( )- NH2 .
.
288
CH3 - 2
CH3 C CH3 :
NH2
- 2 - 2 - NH2 CH3
1
CH3 2
CH2 CH
3 4
CH2 5
CH 6
CH3
CH3 CH COOH - 3 - 5-
NH2
- 2
2 - 8 - 7
:
- 1
NH3

:
:
+ - NaOH
CH3I + NH3 CH3NH3 I CH3NH2 + NaI + H2O
()1


29 - 7 .
- 2
.
Al2O3 ( )400C .


.

CH3OH + HNH2 CH3 - NH2 + H2O
30 - 7 Al2O3


. 3 - 8 - 7
- 1



(.)2 - 7

289
H
H O: H N: H O:
H R H

H H H
H N: H N: H N:
R R R

2-7

-
.
. -
- 2 .

( )
.
HCl :
R - NH2 + HCl R - NH3+Cl-

:
R N H

R
.NaOH
NaOH
R - NH3+Cl- R - NH2 + NaCl + H2O
.
()
:
O H O H
CH3 C Cl H N CH3 CH3 C N CH3 HCl

- N

290

Organic Chemistry

.

Hydrocarbons


.

Isomers

.

Functional Group
.
.

Alkyl Halides
( )R- ( )-X
R-X CnH2n+1X
() .C-X

Markovnikov Rule
( )

.

Electrophyl Reagent

O R+ X+ NO2+
=

-C-
AlCl3

291
Nucluphyl Reagent

H- X- OH- R-
NH3


R-OH
CnH2n+2O . C-OH


R-O-R
CnH2n+2O .C-O-C

O
=

- C -
CnH2nO O
=

R -C- H .


O

=

R -C-R
CnH2nO .


-COOH R-COOH
CnH2nO2


O
- C - O -
=


O
CnH2nO2 -R
=

R - C -O - R



RNH2 R2NH R3N R-NH2
CnH2n+3N
292

1-7
.
CH2 CH2 ; CH CH ; CH3OCH3 ; CH3CH2OH ; CH3CH2Cl ; CH3 CH3
O O O O

CH3CH2NH2 ; CH3C CH3 ; CH3C H ; CH C


3 ; CH3C OH
OCH3
2-7-1

3-7 :
. ) - 1 - 1 . )
. ) . )

- 1 - 1 - 1 4-7

5-7

CH3 O

3( CH CH CH CH CH Br 2( CH3 CH2 C CH2OH (1 CH3 )CH2(3 C OH


3 2 2 2 2

CH3
O
5( CH3 )CH2(2 C OCH3 4( CH3CH2CH2CH2NH2

O O O
8( CH3 )CH2(2 C CH3 7( CH3 )CH2(4 C H 6( CH3 CH2 CH2 C CH2CH3

6-7 :
)2 )1 - 32
- 3 )4 - 4 - - 3 )3
- 3 )6 - 2 - - 2 )5 - 2 -
- 2 )8 - 2 - - 3 )7
)10 - 3 - )9

7-7
C4H8O2 C4H8O C4H9OH :
( : (. )1 - 7

293
8-7 :
)1 .
)2 .
)3 .

9-7 CnH2n+2O 60g/moL


. .
.

10-7 :
: - 1 CH3CH2OH
O
) CH3COOH) CH3CH3- CH2 ) CH2 CH3C H )

- 2 - 2 K2Cr2O7 :
) ) ) )

- 3 H2 Ni :
) ) - 1 ) - 2 )

- 4 :
) H2 ) CO ) O2 ) CO2

- 5 ( -) HCl:
) ) ) )

- 6 :
) CO ) O2 ) CO2 ) H2

- 7 ( ) HCl/ZnCl2 :
) . ) . ) .

- 8 C3H6Br2:
)5 )4 )3 )2

294
- 9 Ag)NH3(2OH
.
) ) ) )

- 10 :
O O O O
) ) ) )
CH3 C CH3 CH3C OH CH3C H CH3C OCH3

- 11 - 2 :
) ) ) )

11-7
.

12-7 CH2 CH2


A : E
CH2 CH2 + HCl A

H2O
A + KOH B

K2Cr2O7 K2Cr2O7
B C D
H+ H+

D + NaHCO3 E + CO2 + H2O

13-7 :
- 1 )2 )1
- 2 - 2 - - 1 )4 )3
)6 )5

14-7 :
O
) CH
)
CH3 3

CH3 C CH3 CH3 CH CH2 CH3 C CH3 CH3 C CH3


Cl OH

- 1 - 2 )

295
15-7 :
) - 2 ) - 2 )

16-7 :

OH- K2Cr2O7
C4H9Br C4H9OH C4H8O
A B C
) A : B C ) A : B C
) B ) C B
K2Cr2O7.

17-7 C4H8O2
.
CH3CH3 CH3CH2OH CH3CHO CH3COOH CH3CH2OCH3

18-7 .
) CH3COOCH2CH3 ) CH3COOH
) CH3CH2MgCl ) CH3CH2Cl

19-7 CH2 = CH2 :


) ) )

20-7 E , D , C , B , A A
:
HCl Mg CO2 HCl
A B C D E

21-7 :
)1 .
)2 .

22-7 ( )A ( )B
.()A()C()B(.)A
) A : B C ) A : B C

296

8
Biochemistry

:
.
.
.
().
.

297
1-8


.

.

.
: .

2 - 8 Carbohydrates




.

1 - 2 - 8

.

)CH2O)n Cn(H2O)n

( )Carbon (.)Hydrate
.
( )Complex carbohydrate (Simple
)carbohydrate

.

298
2-2-8 :
:
-1 ( )
( :)Monosacrchaides .
- 2 ( )
( :)Disacrchaides .
- 3 ( )
( :)Polysacrchaides .

- 1

.

- (: )Glucose


.
. C6H12O6 C6(H2O)6

:

H C O
H C OH CH2OH
HO C H O
H
H
H

H C OH C C
H C OH OH H
. OH OH
H C OH H OH

299 H
:
* .
* .




() .

.


.



. O
OH
- ( ) Fructose H C H H
H
HO H HO C OH
H
C6H12O6 OH H

()
() ().
H
H C OH
( C O
O HO C H
) ( )-OH (
C H C OH
) ( )-OH C O C ( H C OH
H C OH
. H
()

O
( ) C
-

-
.

.
300
)2 :

:
(:)Soucrose


.C12H22O11 :

.

.
.
Sucrose Fructose + Glucose

)3 :
:
().
- : Starch



.
()
.
301
- :Cellulose


( )



.

.

.
.

Cellulose Glucose

3-2-8


.

.

3-8 Proteins
( )Pro
( )teins


.






.
302

1-3-8

:
.

2-3-8

:


:
303
- 1
- 2
.R

3-3-8
1-8
( )
() () - ( )
40000000 - 40000 (.)amphoteric
:

R O H R O
H2N C C OH H N C C OH
H H

R O H R O
H2N C C N C C OH H2O
H H


:
H O
H N ( ) C OH

4-3-8 :



HCl
(
)
.NaOH
304

20



.


:

() .
.


. (
)
.
: .

4-8


.

pH
.
(
) .
:
- 1
.
- 2 (
) .

305
() Lipids 5-8





.
( )

.
.


( )Triester of glycerol Fatty acids
( )C24 - C12
.

6-8

( )


NaOH KOH Fat


.
306
O
CH2 O C (CH )14 CH3
2
O CH2 OH O
CH O C (CH )14 CH3 3NaOH CH OH CH3(CH2)14 C O Na
2

O CH2 OH
CH2 O C (CH )14 CH3
2



NaOH

2-8 KOH
.

.
.

307

Biochemistry

.

Carbohydrates

)CH2O)n Cn(H2O)n
( .)Hydrate
.



C6H12O6 C6(H2O)6
.



C6H12O6 C6(H2O)6
.



C12H22O11
() C12(H2O)11
.

308



.



( )Pro ( )teins
.
()
.

Amino Acids

.

Anzymes

.
pH
.

Lipids

.

309

1-8 :
- 1 :
- .
- .
- .
- 2 :
- . - -
- 3 :
- - - -
- 4 () :
- () - -

2-8 :
- .
- .
- .

3-8

4-8 :





()

310

Carbonium ion ()
Carbanian ion
Enthalpy(
Electrophil
Compex ion )
Primary Initial
Mono Hydroxide ion
Ester Hydrogen ion
Anzyme Electron
Reduction
Anode IUPAC
(
Ampere
) Addition
( ) Coordination bond
Monodentate
Ionic equilibrium (
Bohr magneton
Chemical equilibrium )
Protein Cation
Battery Anion
Weak electrolyte
( )
Amphoteric
Exothermic Reaction Hydrogen-ion exponent
Common ion Endothermic Reaction
Quantitative analysis Fusion
Entropy
Qualtitative analysis
( Forward)
Volumetric analysis Isomer
Volatilization Hydrogen bond
Titration Ether
Nutralization Esterfication

311

Di Valancy
Tri Electric current
Solubility product Chemical analysis
)( Homogeneous reaction

Hetrogeneous reaction
Molecule
Salt bridge Concentration
Cell potential Self ionization
Oxidation potential Solvolysis
Reduction potential Hydrolysis
Standard potential Electrolysis
Joule (J) ) ( Vaporiziation
Condensation
)(
Crystalization
Heat Spontaneous
Specific heat Reversible reaction
State
Irreversible reaction
Absorbed heat
Evolved heat
Common - ion effect
Boundary
Volume
Redox reaction
Calculator
Oxidation state )(
Acid
Thermodynamic
Weak acid
Bidentate
Reaction quotieut
Equilibrium constant
Polyprotic acid
Secondary
Strong acid Tertiary

312

)( Carboxylic acid
Amino acid
Tetrahedral )(
Precipitate
)( Extensive property
Intensive property
Time Linear
Backward
)(
Electrochemical cell
cell
Heat capacity
Galvanic cell
Liquid
Electrolyte cell
Centimeter
Voltaic cell
Burret
Fuel cell
Saccharides
Dry cell
)( Mercury cell

Solid )(
Saponification
Indicator
)( Erylnmyer
Temperature
Pressure
Melting point

Standrad Temperature Boiling Point

and Pressure (STP) Lipid ) (

)(
)(
Solubility
Energy Atom
Kinetic energy Molar solibility

313

)( Potential energy
Gibbs free energy
Effective atomic Satandard Gibbs free
number rule energy
Base Satandard Gibbs free
Le Chatelirs Principle energy of formation
Weak Bade )(
Strong base
Conjucated base Normality
Hess law Avogadros number
Oxidiation number
Markovnikov rule
Coordination number
Electromotive force Element
(emf) Oxidiation agent
Reference electrode Reduction agent

Satandrad hydrogem )(

electrode (SHE)
Nonelectrolyte
)( Nonespontaneous
)(
Equivalent mass
Molar mass Oven
Mass Buffer action
Quantity (
Voltmeter
Corrination chemistry )
Graphite (
Volt
Organic chemistry )

314

Coordination Tollens reagent
sphere Carbohydrates
Multidentate Electrochemistry
Acceptor Cathode
Coordination Columb (C) (
complex )
Equilibrium Biochemistry
posttion )(
Diamond
Rhombic Ligand
Orthorhombic Chealating ligand
Fehlings solution Logarethem
Buffer solution ) (
)(
Nernest equation
)( Reactant
Chemical equation
Product Neutral complex
Valence bond Trigonal planer
theory Squar planer
System Surrounding
End point Universe
Werners theory Balance
Nucleuphil Factor
)( Mole
Molarity
Triagonal pyramid Millimeter
Pipte
)(
Salt
Donor

315

254 Es Einsteinium 227 Ac Actinium


167 Er Erbium 27 Al Aluminium
152 Eu Europium 243 Am Americium
253 Fm Fermium 122 Sb Antimony
19 F Fluorine 40 Ar Argon
223 Fr Francium 75 As Arsenic
157 Gd Gadolinium 210 At Astatine
70 Ga Gallium 137 Ba Barium
73 Ge Germanium 247 Bk Berkelium
197 Au Gold 9 Be Beryllium
178.5 Hf Hafnium 209 Bi Bismuth
4 He Helium 11 B Boron
165 Ho Holmium 80 Br Bromine
1 H Hydrogen 112 Cd Cadmium
115 In Indium 40 Ca Calcium
127 I Iodine 249 Cf Californium
192 Ir Iridium 12 C Carbon
56 Fe Iron 140 Ce Cerium
84 Kr Krypton 133 Cs Cesium
139 La Lanthanum 35.5 Cl Chlorine
259 Lr Lawrencium 52 Cr Chromium
207 Pb Lead 59 Co Cobalt
7 Li Lithium 63.5 Cu Copper
175 Lu Lutetium 254 Cm Curium

24 Mg Magnesium 162.5 Dy Dysperosium

316


101 Ru Ruthenium 55 Mn Manganese


150 Sm Samarium 101 Md Mendelevium
45 Sc Scandium 201 Hg Mercury
79 Se Selenium 96 Mo Molybdenum
28 Si Silicon 144 Nd Neodymium
108 Ag Silver 20 Ne Neon
23 Na Sodium 237 Np Neptunium
88 Sr Strontium 59 Ni Nickel
32 S Sulfur 93 Nb Niobium
181 Ta Tantalum 14 N Nitrogen
99 Tc Technetium 253 No Nobelium
128 Te Tellurium 190 Os Osmium
159 Tb Terbium 16 O Oxygen
204 Tl Thallium 106 Pd Palladium
232 Th Thorium 31 P Phosphorus
169 Tm Thulium 195 Pt Platinum
119 Sn Tin 242 Pu Plutonium
48 Ti Titanium 210 Po Polonium
184 W Tungsten 39 K Potassium
238 U Uranium 141 Pr Praseodymium
51 V Vanadium 145 Pm Promethium
131 Xe Xenon 231 Pa Protactinium
173 Yb Ytterbium 226 Ra Radium
89 Y Yttrium 222 Rn Radon
65 Zn Zinc 186 Re Rhenium
91 Zr Zirconium 103 Rh Rhodium

317

1- P.W.Atkins and J. d Paula, Atkins Physical Chemistry 9th Edition, Oxford Uni-
versity Press, Oxford (2012).
2- A.F. Cotton, G. Wilkinson, A.C. Murillo and B. Bochmann, Advanced Inorganic
Chemistry 7th Edition, Wiley-VCH, New York (2008).
3- R.T. Morrison and R.N. Boyd, Organic Chemistry 6th Edition, Pearson Prentic
Hall, New Jersy (2008).
4- K.W. Whitten, R.E. Davis and L. M. Peck, General Chemistry 7th, Edition. Holt
Rinehart and Winston, New York (2010).
5- A.T. Schwartz, D. M. Bunce, R. G. Silberman , C. L. Stanitski, W. J. Statton and A.
P. Zipp, Chemistry in Context American Chemical Society, New York (1997).
6- R. Duran , L. P. Gold, C. G. Hass, and A. D. Norman Chemistry McGraw-Hill,
New York (2003).
7- I. A. Vogel , Text Book of Quantitative Chemical Analysis, 5th Edition,
Longman Press, England (1989).
8- D. A. Skoog, D.M. Weast, F. G. Holler and S.R. Crouch, Fundamental of
Analytical Chemistry, 8th Edition, Brooks Cole, Canda(2004).
9- I. A. Vogel , A Text Book of Macro and Semimicro Quantitative
Inorganic Analysis, 4th Edition, Longman Press, England (1974).
10- S. Prakash, G.D. Tuli,S.K. Basu and R.D. Madan,Advanced Inorganic Chemis-
try vol. II, Chand Company Ltd., New Delhi (2010).
11- J. E. Huheey Inorganic Chemistry Harper and Row Publisher, London (1997).
12- N. J. Tro , Introductory Chemistry 3rd Edition, Pearson Prentic Hall, New Jer-
sey (2009).
13- A. C. Wilbraham , D. D. Staley, M. S. Matta , and E. L.Waterman, Chemistry,
Pearson Prentice Hall, New Jersy (2008).
14- R. Chang , and B. C. Shank , Chemistry, Mc Graw - Hill , New York (2005).
15-K. J. Denniston , J. J.Topping , and R. L.Caret, General Organic and
Biochemistry, Mc-Graw- Hill, New York (2004).
16-S.S Zumdahle and S.A Zumdahle Chemistry Houjhton Mifflin Bosten (2003).
17- M.S. Silberberg , Chemistry , the Molecular Nature of Matter and Change Mc
Graw Hill, London (2003) .
318

-1

(.)1987

-2
(.)1984

-3
(.)1988

- 3 -
(. ) 2008-2009

- 4
(.)2006

319

320

You might also like